You are on page 1of 571

Prepared by Dr.

Sudhir Sindagi

Introduction to the Course

Geometry

Transverse
Seakeeping
Stability

Naval Longitudinal
Maneuvering
Architecture Stability

Propulsion Strength

By Resistance

Dr. Sudhir Sindagi 1


Prepared by Dr. Sudhir Sindagi

Introduction to the Faculty


Working in TMI since July 2011 as an Associate
Professor
Ph.D from IIT Madras
M.Tech. from IIT Kharagpur
Published more than 10 Research Papers in
International WoS / Scopus indexed Journals
Executed consultancy works for the industries across
the Globe
Received no. of awards including one from GOI
4 Yrs of work experience in Larsen and Toubro,
Mumbai as an Assistant Manager – Naval Architect.
https://www.linkedin.com/in/sudhir-sindagi-8aa47622/ 2
Prepared by Dr. Sudhir Sindagi

2019 End Sem Result

3
Prepared by Dr. Sudhir Sindagi

Introduction to the Course

Geometry

Transverse
Seakeeping
Stability

Naval Longitudinal
Maneuvering
Architecture Stability

Propulsion Strength

Resistance
4
Prepared by Dr. Sudhir Sindagi

Introduction to the Course


Naval Architecture -I
– Geometry, Lines plan, Coefficients of Form
– Flotation, FWA, DWA & load line
– Numerical Methods/Integration
– Calculation of CG & Angle of List
– Stability concepts - Small Angle of Heel
– Stability at large angles of Heel
– Longitudinal Stability – Trim
– Damaged Stability
– Strength of Ships
– IMO Rules related to Stability
– Critical stability during docking / Grounding
5
Prepared by Dr. Sudhir Sindagi

Introduction to the Course


Naval Architecture -II
– Resistance of the Ship
– Propulsion of the Ship
– Manoeuvring of the Ship
– Motion of Ships on waves

6
Prepared by Dr. Sudhir Sindagi

Lecture Plan
Class
Chapter Topic Marks
No
1 Detailed Introduction to the course
Representation of ship geometry: Body plan, Profile
2 Geometry, and Half breadth plan, Bonjean Curves. Definition of
Lines plan, offset and table of offsets.
7-12
3 Coefficinet of Areas - WPA & WSA, CG& CB, Coefficient of Forms
4 Forms Numericals on Coefficient of Forms
5 TPC, MCTC & Numericals
6 Numericals- DWT, GT, NT, TPC, MCTC
Hydrostatic Curves, Effect of Density, FB, Reserve
7
Flotation, FWA, Buoyancy
4-12
8 DWA & loadline FWA, DWA, Loadline
9 FWA, DWA, Loadline- Numericals 7
Prepared by Dr. Sudhir Sindagi

Lecture Plan
10 Trapezoidal Rule, Simpson’s 1st & 2nd rules, 5-8 1 Rule. Marks

11 Area of Water plane, LCF, TPC, MI - Numericals


12 Numerical Use of Second Rule, 1/2 ordinates
13 Methods Use of Second Rule, 1/2 ordinates
14
14 Calculation of Displacement, VCB
15 Use of TPC to calculate the displacement and VCB
16 Use of 5-8-1 and 3-8-1 rule
Find the change in CG when weight is added, removed or
17 Calculation Marks
shifted within the ship.
of CG &
18 Numericals
Angle of
19 Effect of suspended mass - Numericals 12
List 8
20 Calculation of angle of list - Numericals
Prepared by Dr. Sudhir Sindagi

Lecture Plan
Explain stability, criterion for positive, negative and neutral
21 Marks
Stability stability, righting and heeling moment.
22 concepts - BM= IT/V - Derivation
23 Small Numericals on GM= KB+BM-KG
24 Angle of Free Surface Effect
14-24
25 Heel Free Surface Effect - Numericals
26 Inclining Experiment
27 Inclining Experiment - Numericals
28 Derivation GZ= (GM+ BM/2 Tan^2θ)*sinθ - Wall Sided Formula
29 Stability at Angle of Loll
30 large Curve of Statical Stability - GZ Curve 7-12
31 angles of Curve of Statical Stability - GZ Curve - Numericals
32 Heel Curve of Statical Stability - KN tables & Curves- Numericals 9
33 Dynamical Stability
Prepared by Dr. Sudhir Sindagi

Lecture Plan
34 Trim, Concepts of IL, GML and MCTC. Marks
Find the drafts at the AP & FP and its trim. Change in mean
35
Longitudinal draft, trim and sinkage due to addition / removal of weight
36 Stability Numericals 14
Calculate the change in mean draft, trim and sinkage due to
37
change in water density.
Calculate the change in drafts & trim due to bilging by
38
Added weight and Lost Buoyancy methods.
Consideration of the permeability of compartment and
39 Damaged
stowage factors. Margin lines. 14
Stability
Calculate the damaged stability after flooding. Flooding
40
calculation. 10
41 Numericals
Prepared by Dr. Sudhir Sindagi

Lecture Plan
Ship as Girder, theory, Weight Curve, Buoyancy Curve, Load
42 Marks
curve, SFD, BMD
43 Properties of all curves with interlinking
44 Problems
45 Strength Stress on a Section, Problems
46 of ships Determining MI of Section - Problems
14-24
47 Determining MI of Section - Problems

48 Finding bending Stress in Decks

11
Prepared by Dr. Sudhir Sindagi

Lecture Plan
49 IMO Rules Stability requirements of Merchant vessels. Marks
related to Stability of Vessels subjected to Wind Loading and
50
Stability grain carrier
4-10
Floodable length curves, Factors of sub- division,
51 SOLAS
permissible length, compartment standard

12
Prepared by Dr. Sudhir Sindagi

Text Book and Ref books


Text Book: NIL
Reference Books:
– Buoyancy & Stability of Ships – by IR R F Scheltema De Heere & Bakker (George
Haarp & Co. Ltd. London)
– Ship Hydrostatics and Stability – by Adrian B Biran
– Principles of Naval Architecture -Vol I – by Edward V Lewis (SNAME)
– Ship Stability for Masters & Mates – by Derrett & Barrass
– Naval Architecture for Marine Engineers – Reeds Volume - 4
– Introduction to Naval Architecture – Eric Tupper
– Ship and Naval Architecture – R.Munro-Smith
– Ship Construction – D.J.Eyers
– Naval Architecture, Principles & Theory – B.Baxter
13
Prepared by Dr. Sudhir Sindagi

Evaluation Program
Internal Assessment as per guidelines issued by IMU

Sr.
Component Weightage Nature Date
No.
As per the Time Table
1 Class Test 20 Marks Written
declared by Exam
Teacher
2 10 Marks Observation Continuous
Assessment
Term End
3 70 Written To be announced
Examination

Note: There shall be a common minimum pass mark 50% in the External
14
Examinations and 50% overall.
Prepared by Dr. Sudhir Sindagi

15
Prepared by Dr. Sudhir Sindagi

Ship's Geometry and Flotation

By
Dr. Sudhir Sindagi 1
Prepared by Dr. Sudhir Sindagi

Chapter Content
Ship geometry, Definition of hull surface
Lines plan drawing of ships
Offset Table
Bonjean Curves
Archimedes principle, Displacements
Coefficients of form
Effect of Density and relative density of a liquid on drafts.
Meaning of buoyancy, reserve buoyancy
Center of Gravity and Center of Buoyancy of ship
TPC, FWA, DWA
Plimsoll line or Load line markings 2
Prepared by Dr. Sudhir Sindagi

Ship’s Geometry

3
Prepared by Dr. Sudhir Sindagi

Ship’s Geometry

4
Prepared by Dr. Sudhir Sindagi

Ship’s Geometry

5
Prepared by Dr. Sudhir Sindagi

Ship’s Geometry
A ship’s hull form helps determine most of its main attributes; its
stability characteristics; its resistance and therefore the power needed
for a given speed; its seaworthiness; its manoeuvrability and its load
carrying capacity.
It is important, therefore, that the hull shape should be defined with
some precision and unambiguously.
The after perpendicular is the perpendicular drawn to the Summer Load
Line - SLL (Design Water Line - DWL) at the after side of the rudder post,
where fitted, or the line passing through the centreline of the rudder
pintles.
The fore perpendicular is the perpendicular drawn to the to the Summer
Load Line - SLL (Design Water Line - DWL) at the intersection of the
6
forward side of the stem with the summer load line.
Prepared by Dr. Sudhir Sindagi

Ship’s Geometry

7
Prepared by Dr. Sudhir Sindagi

Ship’s Geometry
The length overall (LOA) is the horizontal distance between the extreme
points at the forward and aft end of the ship, measured parallel to the
SLL or DWL.
The Length on the waterline (LWL) is the length on the waterline between
the intersections of SLL or DWL at the bow and at the after end of the
ship.
The Length between perpendiculars (LBP) is the horizontal distance
measured between the aft perpendicular and the forward perpendicular
of the ship, measured parallel to the SLL or DWL.
The mid-point between the two perpendiculars is called amidships or
midship.
The transverse section of the ship at the midship is called the midship
8
section.
Prepared by Dr. Sudhir Sindagi

Ship’s Geometry
The draught of the ship at any point along its length is the vertical
distance from the keel to the waterline. If a moulded draught is quoted it
is measured from the inside of the keel plating.
Freeboard is the difference between the depth at side and the draught,
that is it is the height of the deck above the waterline.
The freeboard is usually greater at the bow and stern than at amidships.
This helps create a drier ship in waves.
Air draught is the vertical distance from the summer waterline to the
highest point in the ship, usually the top of a mast. This dimension is
important for ships that need to go under bridges in navigating rivers or
entering port.
9
Prepared by Dr. Sudhir Sindagi

Ship’s Geometry
The ship’s depth varies along the length but is usually quoted at midship. It
is the vertical distance measured between the upper surface of the main
deck at side and the base line of the ship.
At midship, if the depth at the centreline of the ship is greater than at the
side, then the ship is said to have the camber or the round of beam.
Camber is generally provided only to weather decks to drain the water at
side.
Camber can be defined as the curvature given to the main deck in the
transverse direction and is measured as the difference between the depth
of the ship at the centreline and at the side.
The curvature given to the main deck in the longitudinal direction is known
as the Sheer. It is measured as the difference between the depth of the ship
at the midship and/or at the forward perpendicular and/or at 10the aft
perpendicular.
Prepared by Dr. Sudhir Sindagi

Ship’s Geometry

11
Prepared by Dr. Sudhir Sindagi

Ship’s Geometry

12
Prepared by Dr. Sudhir Sindagi

Ship’s Geometry

13
Prepared by Dr. Sudhir Sindagi

Ship’s Geometry

14
Prepared by Dr. Sudhir Sindagi

Ship’s Geometry
The bottom of a ship, in the midships region, is usually flat but not
necessarily horizontal.
If the line of bottom is extended out to intersect the moulded breadth
line, the height of this intersection above the keel is called the rise of
floor or deadrise.
Many ships have a flat keel and the extent to which this extends
athwartships is termed the flat of keel or flat of bottom.
In some ships the sides are not vertical at amidships. If the upper deck
beam is less than that at the waterline it is said to have tumble home,
the value being half the difference in beams.
If the upper deck has a greater beam the ship is said to have flare. All
ships have flare at a distance from amidships 15
Prepared by Dr. Sudhir Sindagi

Ship’s Geometry
Inward curvature given to the side shell above the SLL is known as
tumble home and the outward curvature given to the side shell above
the SLL is known as Flare.

16
Prepared by Dr. Sudhir Sindagi

Ship’s Geometry
Reserve buoyancy is the volume of the enclosed spaces above the
waterline (SLL). It is a very important factor in which, minimum
freeboards are assigned to a ship to ensure that there is adequate
reserve buoyancy at all times especially when there is a case of
flooding.
Freeboard(F) = Depth(D) – Draft (T)

17
Prepared by Dr. Sudhir Sindagi

Ship’s Geometry- Line Plan Drawing


The hull shape is defined by its intersection with three sets of mutually
orthogonal planes famously known as Lines Plan Drawing of the ship.
Lines plan drawing basically consists of three different views created by
the intersection of these planes viz. Body Plan, Profile View and Half
Breadth Plan
The horizontal planes (Longitudinal Horizontal Sections) are known as
waterplanes and the lines of intersection are known as waterlines and
are represented in Half Breadth Plan.
The planes parallel to the middle line plane (Longitudinal vertical
Sections) cut the hull in buttock (or bow and buttock) lines, the middle
line plane itself defining the profile and are represented in Profile view.
The intersections of the athwartships planes define the transverse 18
sections and are represented in Body Plan.
Prepared by Dr. Sudhir Sindagi

Ship’s Geometry- Line Plan Drawing

19
Prepared by Dr. Sudhir Sindagi

Ship’s Geometry- Line Plan Drawing

Transverse Longitudinal vertical


Sections Sections- Buttocks

Longitudinal
Horizontal Sections
Waterplanes 20
Prepared by Dr. Sudhir Sindagi

Ship’s Geometry- Line Plan Drawing

Transverse sections

Waterlines

Buttocks

21
Prepared by Dr. Sudhir Sindagi

Ship’s Geometry- Line Plan Drawing

22
Prepared by Dr. Sudhir Sindagi

Ship’s Geometry- Line Plan Drawing

23
Prepared by Dr. Sudhir Sindagi

Ship’s Geometry- Line Plan Drawing

24
Prepared by Dr. Sudhir Sindagi

Ship’s Geometry- Line Plan Drawing

25
Prepared by Dr. Sudhir Sindagi

Ship’s Geometry- Line Plan Drawing

26
Prepared by Dr. Sudhir Sindagi

Ship’s Geometry- Line Plan Drawing


Stations: These are the longitudinal positions on the ship placed at an
equal intervals, wherein, transverse sections are taken, to generate the
Body Plan. Generally, the ship is divided into 10 equal parts
longitudinally, resulting in 11 in number stations, with numbering 0,1,2,3
…8,9,10. The station with number 5 will be placed at the midship.
To capture the abrupt changes in the shape of the hull at the aft and the
forward end, the half stations are inserted.

27
Prepared by Dr. Sudhir Sindagi

Ship’s Geometry- Line Plan Drawing


Waterlines: These are the vertical positions on the ship placed at an
equal intervals, wherein, Longitudinal Horizontal Sections (Waterplanes)
are taken to generate the half breadth plan.
Since the ship is symmetric about the centreline, hence in the half
breadth plan, only half side of the ship is shown.

28
Prepared by Dr. Sudhir Sindagi

Ship’s Geometry- Line Plan Drawing


Offsets (ordinates):
These are the
transverse positions
on the ship placed at
an equal intervals,
wherein, Longitudinal
Vertical Sections
(Buttocks) are taken
to generate the
Profile view.
Offset of any point on
any water line is the
transverse distance 29
from the center line
Prepared by Dr. Sudhir Sindagi

Ship’s Geometry- Line Plan Drawing


Body Plan: It is obtained by taking transverse sections of the ship at
various stations. Generally, the left side of the body plan represents the
sections of the ship taken at the aft end till midship.

While, the right side of


the body plan
represents the sections
of the ship taken at the
forward end from
midship

30
Prepared by Dr. Sudhir Sindagi

Ship’s Geometry- Line Plan Drawing


Half Breadth Plan: It is obtained by taking Longitudinal Horizontal
Sections (Waterplanes) of the ship taken at various waterlines.
Since the ship is symmetric about the centreline, hence in the half
breadth plan, only half side of the ship is shown.

31
Prepared by Dr. Sudhir Sindagi

Ship’s Geometry- Line Plan Drawing


Profile View: It is obtained by taking Longitudinal Vertical Sections
(Buttocks) of the ship taken at various offsets (ordinates).

32
Prepared by Dr. Sudhir Sindagi

Ship’s Geometry- Line Plan Drawing

33
Prepared by Dr. Sudhir Sindagi

Ship’s Geometry- Line Plan Drawing

34
Prepared by Dr. Sudhir Sindagi

Ship’s Geometry- Line Plan Drawing

35
Prepared by Dr. Sudhir Sindagi

Ship’s Geometry- Offset Table


Offsets: Offset of any intersection point on any waterline is its
transverse distance from the center line of the ship.
A collection of such offsets in the form of table for different transverse
sections taken at different stations is known as the offset table.
The offset data or Table must be measured and prepared at every
intersection points on each stations and waterlines including deck line,
chines, knuckles and bulwarks (if any).
Offset data also called as half breadth data, because it represents the
half breadth of the ship at every station and waterlines.
It is also a standard practice to indicate the data of height above for
deck, chine, bulwark, and knuckles lines. The height above base of
buttock lines may also be included whenever necessary. 36
Prepared by Dr. Sudhir Sindagi

Ship’s Geometry- Offset Table

37
Prepared by Dr. Sudhir Sindagi

Ship’s Geometry- Offset Table

38
Prepared by Dr. Sudhir Sindagi

Ship’s Geometry- Offset Ta

39
Prepared by Dr. Sudhir Sindagi

Ship’s Geometry- Bonjean Curves

Bonjean Curve is a curve obtained by joining points at different


waterlines, distance of which from the vertical line represents 40area of
that transverse section till the particular waterline.
Prepared by Dr. Sudhir Sindagi

Ship’s Geometry- Bonjean Curves


Collection of such curves prepared
for different stations is known as the
Bonjean Curves.
Bonjean curves are a representation
of the area of the transverse sections
at successive waterlines.
These are used in calculating the
volume of displacement and the
center of buoyancy at any waterline,
or angle of trim. They are used in
stability calculations, determining the
capacity of the ship, or in launching
calculations. 41
Prepared by Dr. Sudhir Sindagi

Ship’s Geometry- Bonjean Curves


Bonjean curves are
presented in two
ways
– With common
vertical axis
– With separate
vertical axes

42
Prepared by Dr. Sudhir Sindagi

Ship’s Geometry- Bonjean Curves


When plotted on a profile of the vessel, each Bonjean curve begins on
the base of the station it represents.
It rises forward on the profile, indicating the transverse area of the hull
at that location on the hull.

43
Prepared by Dr. Sudhir Sindagi

Coefficients of Form
Archimedes' principle: It states that the upward buoyant force that is
exerted on a body immersed in a fluid, whether fully or partially
submerged, is equal to the weight of the fluid that the body displaces.
Weight of the body = Weight of the water displaced
Volume of the water displaced = underwater volume of the ship
If Δ = Displacement or weight of the ship in tonnes= weight of the water
displaced by the ship
𝛁= Volumetric displacement of the ship = Volume of water displaced,
then
∆= 𝛁 ∗ 𝝆
Here 𝝆 is the density of water displaced.
ρSW = 1.025 tonnes/m3 & ρFW = 1.0 tonnes/m3 44
Prepared by Dr. Sudhir Sindagi

Coefficients of Form
Coefficients of form or coefficients of fineness are dimensionless
numbers that describe hull fineness and overall shape characteristics.
The coefficients are ratios of areas or volumes for the actual hull form
compared to prisms or rectangles defined by the ship’s length, breadth,
and draft.
Since length and breadth on the waterline as well as draft vary with
displacement, coefficients of form also vary with displacement.
Length between perpendiculars (LBP) is most often used, although some
designers prefer length on the waterline.
Coefficients of form can be used to simplify area and volume
calculations for stability or strength analyses.
As hull form approaches that of a rectangular barge, the coefficients
45
approach their maximum value of 1.0.
Prepared by Dr. Sudhir Sindagi

Coefficients of Form
Coefficients of form: These are broadly classified into two categories
Area Coefficients:
– Midship Coefficient (CM)
– Waterplane coefficient (CWP)
Volume Coefficients:
– Block Coefficient (CB)
– Prismatic Coefficient (CP)
• Longitudinal Prismatic Coefficient (CPL)
• Vertical Prismatic Coefficient(CPV)
– Volumetric Coefficient (CV)

46
Prepared by Dr. Sudhir Sindagi

Coefficients of Form
Midship Coefficient (CM)
It is the ratio of the area of the Midship Section till SLL or DWL and area
of a circumscribing rectangle whose sides are equal to the draught and
the breadth of ship

47
Prepared by Dr. Sudhir Sindagi

Coefficients of Form
Midship Coefficient (CM)
area of the Midship Section till SLL or DWL
𝐂𝐌 =
area of a circumscribing rectangle with sides equal to B & T
𝐀𝐌
𝐂𝐌 =
𝐁∗𝐓

Values of CM may range between


0.75 to 0.995 for normal ships.
In some cases vessels have
been built with bulges or
blisters below the design
waterline. If B is taken at the
SLL, then CM may be greater
48
than unity on such vessels
Prepared by Dr. Sudhir Sindagi

Coefficients of Form
Waterplane Coefficient (𝐂𝐖𝐏 ) or Coefficient of fineness:

49
Prepared by Dr. Sudhir Sindagi

Coefficients of Form
Waterplane Coefficient (𝐂𝐖𝐏 ) or Coefficient of fineness:
It is the ratio of the area of the waterplane taken at SLL or DWL and area
of a circumscribing rectangle whose sides are equal to the Length and
the breadth.
area of the waterplane taken at SLL or DWL
𝐂𝐖𝐏 =
area of a circumscribing rectangle with sides equal to L & B
𝐀𝐖𝐏
𝐂𝐖𝐏 =
𝐋 ∗𝐁
The values of CWP at the DWL range from about 0.65 to 0.95, depending
upon type of ship, speed, and other factors.

50
Prepared by Dr. Sudhir Sindagi

Coefficients of Form
Block Coefficient (CB)

51
Prepared by Dr. Sudhir Sindagi

Coefficients of Form
◼ Block Coefficient: CB =  / (L x B x T)

A Beam, B
P L (along DWL}

F
P

Hull Volume,
 (under Block
DWL) Volume, Draft, T
52
LxBxT
Prepared by Dr. Sudhir Sindagi

Coefficients of Form
Block Coefficient (CB):
It is the ratio of the volumetric displacement of the ship and the volume
of the circumscribing rectangular prism with dimensions Length,
Breadth and Draft of the ship.
𝐂𝐁 =
volumetric displacement of the ship
volume of the circumscribing rectangular prism with sides L,B & T

𝐂𝐁 =
𝐋 ∗𝐁 ∗𝐓

53
Prepared by Dr. Sudhir Sindagi

Coefficients of Form
Block Coefficient (CB):
The block coefficient indicates whether the form is full or fine and
whether the waterlines will have large angles of inclination to the middle
line plane at the ends.
Values of CB at design displacement may vary between 0.36 for a fine
high-speed vessel to about 0.92 for a slow and full Great Lakes bulk
carrier.
Large values signify large wave-making resistance at speed. A slow
ship can afford a relatively high block coefficient as its resistance is
predominately frictional.
A high value is good for cargo carrying and is often obtained by using a
length of parallel middle body, perhaps 15–20 per cent of the 54
total
length.
Prepared by Dr. Sudhir Sindagi

Coefficients of Form
Longitudinal Prismatic Coefficient (CP or CPL)

55
Prepared by Dr. Sudhir Sindagi

Coefficients of Form
Longitudinal Prismatic Coefficient (CP or CPL)
It is the ratio of the volumetric displacement of the ship and the volume
of the prism whose length equals the length of the ship and whose
cross section equals the midship section area till SLL remaining
constant throughout the length of the ship.
Volumetric displacement of the ship
𝐂𝐏𝐋 =
Volume of the prism with ( 𝐀𝐌∗𝐋)

𝐂𝐏𝐋 =
𝐀𝐌∗𝐋
Usual range of values is from about 0.50 to about 0.90.
A vessel with a low value of 𝐂𝐏𝐋 (or CB) is said to have a fine hull form,
while one with a high value of 𝐂𝐏𝐋 has a full hull form..
56
Prepared by Dr. Sudhir Sindagi

Coefficients of Form
Longitudinal Prismatic Coefficient (CP or CPL)
If two ships with equal length and displacement have different prismatic
coefficients, the one with the smaller value of CPL will have the larger
midship sectional area and hence a larger concentration of the volume
of displacement amidships. The ship with the smaller CPL is also
characterized by a protruding bulbous bow, which causes the swelling
in the sectional area curve right at the bow, and its extension forward of
Station 0.

57
Prepared by Dr. Sudhir Sindagi

Coefficients of Form
Vertical Prismatic Coefficient (CPV)

58
Prepared by Dr. Sudhir Sindagi

Coefficients of Form
Vertical Prismatic Coefficient (CPV)
It is the ratio of the volumetric displacement of the ship and the volume
of the prism whose depth equals the draft of the ship and whose vertical
cross section equals the waterplane area taken at SLL remaining
constant throughout the draft of the ship.
Volumetric displacement of the ship
𝐂𝐏𝐕 =
Volume of the prism with ( 𝐀𝐖𝐏∗𝐓)

𝐂𝐏𝐕 =
𝐀𝐖𝐏∗𝐓
Unless specifically mentioned, by default, it will be Longitudinal
Prismatic Coefficient is used everywhere.

59
Prepared by Dr. Sudhir Sindagi

Coefficients of Form
Volumetric Coefficient (𝐂𝐕 ) or fatness ratio:
This coefficient (or fatness ratio) is defined as the volume of
displacement divided by the cube of one tenth of the vessel's length
𝛁
𝐂𝐕 = 𝐋
(𝟏𝟎)𝟑

Ships with low volumetric coefficients might be said to be "thin", while


those with a high coefficient are "fat."
Values of the volumetric coefficient range from about 1.0 for light, long
ships like destroyers, to 15 for short heavy ships like trawlers.

60
Prepared by Dr. Sudhir Sindagi

Coefficients of Form
Ranges of different coefficients of form for different types of vessels

61
Prepared by Dr. Sudhir Sindagi

Coefficients of Form
Importance of coefficients of form
The angle at the bow is termed as the angle of entry and influences
resistance. As speed increases a designer will reduce the length of
parallel middle body to give a lower prismatic coefficient, keeping the
same midship area coefficient.
As speed increases still further the midship area coefficient will be
reduced, usually by introducing a rise of floor.
A low value of midship section coefficient indicates a high rise of floor
with rounded bilges.
A large value of vertical prismatic will indicate body sections of U-form;
a low value will indicate V-sections. These features will affect the
seakeeping performance. 62
Prepared by Dr. Sudhir Sindagi

Sectional Area Curve


It is curve obtained by joining points at each station, distance of which
from the horizontal line represents the area of the transverse for that
particular station section till SLL or DWL.

63
Prepared by Dr. Sudhir Sindagi

Sectional Area Curve


One of the fundamental hull form characteristics required to prepare the
hydrostatic curves are the immersed sectional areas at ordinate
stations.
A curve whose ordinates are areas of cross sections up to a given
waterline corresponding to each point in the length

64
Prepared by Dr. Sudhir Sindagi

Sectional Area Curve


Area under SAC= Volumetric displacement of the ship = 
Longitudinal location of centroid of SAC = LCB of the ship
Length of the parallel middle body, Entrance and Run are obtained from
the SAC

65
Prepared by Dr. Sudhir Sindagi

Problems on Coefficients of Form


Find the relationship between CB, CPL & CM
We know that:
𝐂𝐁 = 𝛁Τ𝐋∗𝐁∗𝐓
From 𝐂𝐏 = 𝛁ൗ𝐀𝐌∗𝐋 𝐩𝐮𝐭𝐭𝐢𝐧𝐠 𝛁 = 𝐂𝐏𝐋 ∗ 𝐀 𝐌 ∗ 𝐋 𝐢𝐧 𝐚𝐛𝐨𝐯𝐞 𝐞𝐪𝐮𝐚𝐭𝐢𝐨𝐧
𝐂𝐏𝐋 ∗𝐀𝐌 ∗𝐋
𝐂𝐁 = ൗ𝐋∗𝐁∗𝐓
𝐂𝐏𝐋 ∗𝐀𝐌
𝐂𝐁 = ൗ𝐁∗𝐓
𝐁𝐮𝐭 𝐂𝐌 = 𝐀𝐌ൗ𝐁∗𝐓
𝐂𝐁 = 𝐂𝐏𝐋 ∗ 𝐂𝐌

66
Prepared by Dr. Sudhir Sindagi

Problems on Coefficients of Form


Important Formulae:
∆= 𝛁 ∗ 𝛒
ρSW = 1.025 tonnes/m3 & ρFW = 1.0 tonnes/m3
𝐀 𝐀  
𝐂𝐌 = 𝐌 ∶ 𝐂𝐖𝐏 = 𝐖𝐏 : 𝐂𝐁 = : 𝐂𝐏𝐋 =
𝐁∗𝐓 𝐋 ∗𝐁 𝐋 ∗𝐁 ∗𝐓 𝐀𝐌∗𝐋
 𝛁
𝐂𝐏𝐕 = : 𝐂𝐕 = 𝐋
𝐀𝐖𝐏∗𝐓 (𝟏𝟎)𝟑

𝐂𝐁 = 𝐂𝐏𝐋 ∗ 𝐂𝐌
Form Merchant Vessels typical ranges are
Ratio of 𝐋𝐞𝐧𝐠𝐭𝐡 ∶ 𝐁𝐞𝐚𝐦 = 𝐛𝐞𝐭𝐰𝐞𝐞𝐧 𝟑. 𝟓 ~ 𝟏𝟎.
Ratio of 𝐋𝐞𝐧𝐠𝐭𝐡 ∶ 𝐃𝐫𝐚𝐟𝐭 = 𝐛𝐞𝐭𝐰𝐞𝐞𝐧 𝟏𝟎 ~ 𝟑𝟎.
67
Ratio of 𝐁𝐞𝐚𝐦 ∶ 𝐃𝐫𝐚𝐟𝐭 = 𝐛𝐞𝐭𝐰𝐞𝐞𝐧 𝟏. 𝟖 ~ 𝟓.
Prepared by Dr. Sudhir Sindagi

Problems on Coefficients of Form


Following data of a ship are given
L=200m, B=22m, T=7m, CPL=0.75, AWP=3500m2 Displacement in
SW=23000 tonnes. Calculate CB, CWP & CM
Given Data:
L=200m, B=22m, T=7m, CPL=0.75, AWP=3500m2 Δ=23000 tonnes.
CB=? CWP =? CM=?
Using equation ∆= 𝛁 ∗ 𝛒, 𝐜𝐚𝐥𝐜𝐮𝐥𝐚𝐭𝐞 𝛁 = 22439.02m3

Using equation: 𝐂𝐁 = Calculate: 𝐂𝐁 = 0.728
𝐋 ∗𝐁 ∗𝐓
𝐀𝐖𝐏
Using equation: 𝐂𝐖𝐏 = Calculate: 𝐂𝐖𝐏 = 0.795
𝐋 ∗𝐁

Using equation : 𝐂𝐏𝐋 = Calculate 𝐀𝐌 = 149.59m2 68
𝐀𝐌∗𝐋
Prepared by Dr. Sudhir Sindagi

Problems on Coefficients of Form


Following data of a ship are given
L=200m, B=22m, T=7m, CPL=0.75, AWP=3500m2 Displacement in
SW=23000 tonnes. Calculate CB, CWP & CM
Given Data:
L=200m, B=22m, T=7m, CPL=0.75, AWP=3500m2 Δ=23000 tonnes.
CB=? CWP =? CM=?
𝐀𝐌
Using equation: 𝐂𝐌 = Calculate: 𝐂𝐌 = 0.971
𝐁∗𝐓

69
Prepared by Dr. Sudhir Sindagi

Problems on Coefficients of Form


Following data of a ship are given
L=122m, B=20m, T=14m, CB=0.82, CPL=0.93, CWP=0.75. Calculate Midship
area and waterplane area of ship.
Given Data:
L=122m, B=20m, T=14m, CB=0.82, CPL=0.93, CWP=0.75
AM =? AWP =?

Using equation: 𝐂𝐁 = Calculate: = 28011.2 m3
𝐋 ∗𝐁 ∗𝐓

Using equation : 𝐂𝐏𝐋 = Calculate 𝐀𝐌 = 246.88m2
𝐀𝐌∗𝐋
𝐀𝐖𝐏
Using equation: 𝐂𝐖𝐏 = Calculate: 𝐀 𝐖𝐏 = 1830m2
𝐋 ∗𝐁
70
Prepared by Dr. Sudhir Sindagi

Problems on Coefficients of Form


A lifeboat, when fully laden, displaces 7.2 tonnes. Its dimensions are
7.5m, 2.5m, 1m, and its block coefficient 0.6. Find the percentage of its
volume under water when floating in fresh water.
Given Data:
L=7.5m, B=2.5m, T= 1m, CB=0.6, ∆= 7.2 tonnes
%of underwater volume=?
Using equation ∆= 𝛁 ∗ 𝛒, 𝐜𝐚𝐥𝐜𝐮𝐥𝐚𝐭𝐞
𝐀𝐜𝐭𝐮𝐚𝐥 𝐮𝐧𝐝𝐞𝐫𝐰𝐚𝐭𝐞𝐫 𝐯𝐨𝐥𝐮𝐦𝐞 𝛁𝐀 = 7.2m3

However, Using equation: 𝐂𝐁 = Calculate:
𝐋 ∗𝐁 ∗𝐓
𝐓𝐨𝐭𝐚𝐥 𝐯𝐨𝐥𝐮𝐦𝐞 T= 11.25 m3
𝐀𝐜𝐭𝐮𝐚𝐥 𝐮𝐧𝐝𝐞𝐫𝐰𝐚𝐭𝐞𝐫 𝐯𝐨𝐥𝐮𝐦𝐞 𝛁𝐀 71
%of underwater volume= = =64%
𝐓𝐨𝐭𝐚𝐥 𝐯𝐨𝐥𝐮𝐦𝐞 
Prepared by Dr. Sudhir Sindagi

Problems on Coefficients of Form


Two similar right circular cones are joined at their bases. Each one has
a height equal to the diameter of its base. The composite body floats so
that both apexes are in the water surfaces. Calculate the midship
section coefficient, prismatic coefficient and the waterplane area
coefficient.
Given Data:
Right circular cones floating with both the apexes in water surfaces
H=D.
CM=? CPL =? CWP=?
Since, Right circular cones floating with both the apexes in water
surfaces, hence
T= D/2 72
Prepared by Dr. Sudhir Sindagi

Problems on Coefficients of Form


AM= (π/4 D2)/2
𝐀𝐌
Using equation 𝐂𝐌 =
𝐁∗𝐓
Putting AM and B=D , T=D/2 and simplifying above equation, we get
CM =0.7853.

73
Prepared by Dr. Sudhir Sindagi

Problems on Coefficients of Form


𝟏 𝛑𝐑𝟐 𝐡 
Using  = ∗ ∗ 𝟐 and putting in the equation : 𝐂𝐏𝐋 =
𝟑 𝟐 𝐀𝐌∗𝐋
Putting B=D , T=D/2 and simplifying above equation, we get
𝐂𝐏𝐋 =0.3333
𝐀𝐖𝐏
Using equation: 𝐂𝐖𝐏 =
𝐋 ∗𝐁
Here, 𝐀 𝐖𝐏 =2*1/2*D*D, L=2D
Putting above and simplifying equation, we get
𝐂𝐖𝐏 =0.5

74
Prepared by Dr. Sudhir Sindagi

Problems on Coefficients of Form


A ship displaces 9450 tonnes and has a block coefficient of 0.7. The
area of immersed midship section is 106 sq. meter. Assume sea water
density =1.025 t/m3. If the beam =0.13 Length=2.1 draught, Calculate the
length of the ship and its prismatic coefficient.
Given Data:
AM= 106 m2, B=0.13L=2.1T, CB=0.7, ∆= 9450 tonnes
L= ? CPL=?
Using equation ∆= 𝛁 ∗ 𝛒 , Calculate: = 9219.51 m3

Using equation: 𝐂𝐁 = and putting the terms, we get
𝐋 ∗𝐁 ∗𝐓
9219.51
0.7= 𝟎.𝟏𝟑
𝐋 ∗𝟎.𝟏𝟑𝐋 ∗ 𝟐.𝟏 𝐋 75
Prepared by Dr. Sudhir Sindagi

Problems on Coefficients of Form


A ship displaces 9450 tonnes and has a block coefficient of 0.7. The
area of immersed midship section is 106 sq. meter. Assume sea water
density =1.025 t/m3. If the beam =0.13 Length=2.1 draught, Calculate the
length of the ship and its prismatic coefficient.
Given Data:
Simplifying and solving above equation for Length, we get
L= 117.84m

Using equation : 𝐂𝐏𝐋 = Calculate: 𝐂𝐏𝐋 =0.738
𝐀𝐌∗𝐋

76
Prepared by Dr. Sudhir Sindagi

Displacements
A ship’s displacement significantly influences its behaviour at sea.
A ship is said to be fully loaded, when it is floating at its minimum
statutory freeboard.
Freeboard is the vertical distance between the SLL or DWL or the load
line corresponding to the particular zone and the deckline.
Deckline is obtained by the intersection of the upper surface of the main
deck and the outer surface of the side shell.
It is a line having length of 300mm and thickness of 25 mm marked
amidships on both sides of the ship, which is a part of Plimsol line
(Load line).
The main deck of a ship is the uppermost continuous weatherproof
deck extending from bow to stern (running for the entire length). 77
Prepared by Dr. Sudhir Sindagi

Displacements

78
Prepared by Dr. Sudhir Sindagi

Displacements
Lightweight of the ship is the weight of the newly built ship. It includes
the weights of
– Hull and Superstructure
– Machinery and Equipment
– Piping and ducting
– Cables and furnishings
– Accommodation
Lightship or lightweight measures the actual weight of the ship with no
fuel, passengers, cargo, water.
Deadweight = fully loaded displacement – lightweight.
Deadweight tonnage (often abbreviated as DWT) is the displacement at
any loaded condition minus the lightship weight. It includes the crew,
79
passengers, cargo, fuel, water, provisions, and stores.
Prepared by Dr. Sudhir Sindagi

Effect of density on draft


For Box Barge, as the weight in both SW and FW remains same
∆𝐒𝐖 = ∆𝐅𝐖
∆𝐒𝐖 = 𝛁𝐒𝐖 ∗ 𝛒𝐒𝐖 = ∆𝐅𝐖 = 𝛁𝐅𝐖 ∗ 𝛒𝐅𝐖
L*B∗ 𝐓𝐒𝐖 * 𝛒𝐒𝐖 = L*B∗ 𝐓𝐅𝐖 ∗ 𝛒𝐅𝐖
𝐓𝐒𝐖 * 𝛒𝐒𝐖 = 𝐓𝐅𝐖 ∗ 𝛒𝐅𝐖
Note: This equation is applicable only for the box barge
Since, 𝛒𝐅𝐖 < 𝛒𝐃𝐖 < 𝛒𝐒𝐖 , hence 𝐓𝐅𝐖 > 𝐓𝐃𝐖 > 𝐓𝐒𝐖
Fresh Water Allowance (FWA) = 𝐓𝐅𝐖 − 𝐓𝐒𝐖
Dock Water Allowance (DWA) = 𝐓𝐃𝐖 − 𝐓𝐒𝐖

80
Prepared by Dr. Sudhir Sindagi

Effect of density on draft

81
Prepared by Dr. Sudhir Sindagi

Problems on Displacements
A ship is 150m long, has 20m beam, load draft 8m, light draft 3m. The
block coefficient at the load draft is 0.766, and at the light draft is 0.668.
Find the ship's deadweight.
Given Data:
L=150m, B= 20m, TSW=8m, TL=3m, CBFL=0.766, CBL=0.668, Deadweight=?

Using equation: 𝐂𝐁 = , we calculate
𝐋 ∗𝐁 ∗𝐓
FL= 18384 m3 and L= 6012 m3 putting the terms, we get
Deadweight = fully loaded displacement – lightweight
Using equation ∆= 𝛁 ∗ 𝛒

82
Prepared by Dr. Sudhir Sindagi

Problems on Displacements
A ship is 150m long, has 20m beam, load draft 8m, light draft 3m. The
block coefficient at the load draft is 0.766, and at the light draft is 0.668.
Find the ship's deadweight.
Deadweight = fully loaded displacement – lightweight.
Deadweight = (FL– L)* ρSW
Deadweight=12681.3 tonnes

83
Prepared by Dr. Sudhir Sindagi

Problems on Displacements
A ship 120m long 15m beam has a block coefficient of 0.700 and is
floating at the load draft of 7m in fresh water. Find how much more
cargo can be loaded if the ship is to float at the same draft in salt water.
Given Data:
L=120m, B= 15m, TFW=7m, CB=0.7, TSW=7m
To maintain same draft in SW how much more cargo to be added?
Since, 𝛒𝐅𝐖 < 𝛒𝐃𝐖 < 𝛒𝐒𝐖 , hence 𝐓𝐅𝐖 > 𝐓𝐃𝐖 > 𝐓𝐒𝐖
Hence, to maintain same draft ins SW, need to add weight.
Here, to maintain same draft, it will have similar underwater volume
both in SW and FW.
SW = FW
84
Prepared by Dr. Sudhir Sindagi

Problems on Displacements
A ship 120m long 15m beam has a block coefficient of 0.700 and is
floating at the load draft of 7m in fresh water. Find how much more
cargo can be loaded if the ship is to float at the same draft in salt water.

Using equation: 𝐂𝐁 = , we calculate
𝐋 ∗𝐁 ∗𝐓
FW= 8820 m3 = SW
ΔFW= FW* ρFW = 8820 tonnes
ΔSW=SW* ρSW = 9040.5 tonnes
Weight to be added = ΔSW- ΔFW
Weight to be added = 220.5 tonnes

85
Prepared by Dr. Sudhir Sindagi

Problems on Displacements
A homogeneous log of rectangular cross-section is 5m long, 60 cm
wide, 40 cm deep, and floats in fresh water at a draft of 30 cm. Find the
mass of the log and its relative density.
Given Data:
L=5m, B=0.6m, D=0.4 TFW=0.3m
Δ =? Relative density=?
Since it is the homogeneous rectangular log, hence its block coefficient
will be
𝐂𝐁 = 1.0
Mass of the log =  * ρFW
Mass of the log =𝐂𝐁 * LB TFW * ρFW
86
Mass of the log =0.9 tonnes
Prepared by Dr. Sudhir Sindagi

Problems on Displacements
A homogeneous log of rectangular cross-section is 5m long, 60 cm
wide, 40 cm deep, and floats in fresh water at a draft of 30 cm. Find the
mass of the log and its relative density.
𝐌𝐚𝐬𝐬 𝐨𝐟 𝐭𝐡𝐞 𝐛𝐨𝐝𝐲
Density of the body =
𝐓𝐨𝐭𝐚𝐥 𝐯𝐨𝐥𝐮𝐦𝐞 𝐨𝐟 𝐭𝐡𝐞 𝐛𝐨𝐝𝐲
𝟎.𝟗
Density of the body =
𝐋𝐁𝐃
Putting the value ,we get
Density of the body = 0.75 t/m3
Relative density is the ratio of the density of a substance to the density
of a standard, usually water for a liquid or solid, and air for a gas.
Relative Density of the body = 0.75
87
Prepared by Dr. Sudhir Sindagi

CG and CB of the ship


B - Center of Buoyancy: The geometric center of the ship's underwater
hull body. Centroid of the underwater volume is the CB of the ship.
It is the point at which all the forces of buoyancy may be considered to
act in a vertically upward direction.
The Center of Buoyancy will move as the shape of the underwater
portion of the hull body changes.
The centre of gravity of the ship is commonly denoted as point G.
It is the point on which whole mass of the ship is supposed to be acting.
When a ship is at equilibrium, the centre of buoyancy is vertically in line
with the centre of gravity of the ship

88
Prepared by Dr. Sudhir Sindagi

CG and CB of the ship

89
Prepared by Dr. Sudhir Sindagi

CG and CB of the ship


Longitudinal Center of Gravity (LCG): It is the Longitudinal distance of
the CG from the AP.
Vertical Center of Gravity (VCG of KG): It is the vertical distance of the
CG from the keel.
Transverse Center of Gravity (TCG): It is the transverse distance of the
CG from the centreline of the ship.
Longitudinal Center of Buoyancy (LCB):
Vertical Center of Buoyancy (VCB or KB):
Transverse Center of Buoyancy (TCB):
For the upright condition of the ship TCG=TCB=0 as both CG and CB of
the ship lie on centreline.
For Box barge, always KB=T/2 90
Prepared by Dr. Sudhir Sindagi

CG and CB of the ship

If TCG≠ 𝐓𝐂𝐁, 𝐭𝐡𝐞𝐧 𝐭𝐡𝐞 𝐬𝐡𝐢𝐩 𝐰𝐢𝐥𝐥 𝐋𝐢𝐬𝐭


If LCG≠ 𝐋𝐂𝐁, 𝐭𝐡𝐞𝐧 𝐭𝐡𝐞 𝐬𝐡𝐢𝐩 𝐰𝐢𝐥𝐥 𝐓𝐫𝐢𝐦 91
Prepared by Dr. Sudhir Sindagi

Problems on Displacements
The KB of a rectangular block which is floating in fresh water is 50 cm.
Find the KB in salt water.
Given Data:
KBFW=0.5m
KBSW=?
We know that, for Box Barge, as the weight in both SW and FW remains
same
𝐓𝐒𝐖 * 𝛒𝐒𝐖 = 𝐓𝐅𝐖 ∗ 𝛒𝐅𝐖
𝐓𝐒𝐖 =0.9756m
Moreover, it is also known to that, for Box barge, always KB=T/2
𝐊𝐁𝐒𝐖 = 𝐓𝐒𝐖 /2
92
𝐊𝐁𝐒𝐖 =0.4878m
Prepared by Dr. Sudhir Sindagi

Problems on Displacements
A box-shaped barge 55mX10mX6m is floating in fresh water on an even
keel at 1.5m draft. If 1800 tonnes of cargo is now loaded, find the
difference in the height of the centre of buoyancy above the keel.
Given Data:
L=55m, B= 10m, D= 6m, T OLD =TFW=1.5m, w= 1800 tonnes added.
𝐊𝐁𝐍𝐄𝐖 - 𝐊𝐁𝐎𝐋𝐃 = ?
ΔOLD= OLD* ρFW = LBT OLD * ρFW
ΔOLD= 825 tonnes
ΔNEW= ΔOLD +weight added = 2625 tonnes
ΔNEW= NEW* ρFW = LBT NEW * ρFW Solving it for T NEW we get
T NEW = 4.77m
93
KB NEW = T NEW/2 = 2.38m
Prepared by Dr. Sudhir Sindagi

Problems on Displacements
A box-shaped barge 55mX10mX6m is floating in fresh water on an even
keel at 1.5m draft. If 1800 tonnes of cargo is now loaded, find the
difference in the height of the centre of buoyancy above the keel.
KB OLD = T OLD/2 = 0.75m
𝐊𝐁𝐍𝐄𝐖 - 𝐊𝐁𝐎𝐋𝐃 = 1.63m

94
Prepared by Dr. Sudhir Sindagi

Problems on Displacements
A homogeneous log is 3m long, 60 cm wide, 60 cm deep, and has
relative density 0.9. Find the distance between the centres of buoyancy
and gravity when the log is floating in fresh water.
Given Data:
L=3m, B=0.6m, D=0.6, RD=0.9
KG-KB= ?
In case of solids density of the body=relative density=0.9 t/m3
Weight of the water displaced by the body = weight of the body
Weight of the body = Total volume of the body * density of the body
Δ = LBD *density of the body
Δ = 0.972 tonnes
95
Prepared by Dr. Sudhir Sindagi

Problems on Displacements
A homogeneous log is 3m long, 60 cm wide, 60 cm deep, and has
relative density 0.9. Find the distance between the centres of buoyancy
and gravity when the log is floating in fresh water.
Weight of the body=underwater volume of the body*density of the water
Δ= FW* ρFW = LBT FW * ρFW
Solving it for T FW
TFW = 0.54m.
KB = T/2 = 0.27m
Assuming KG is at the centroid of the body
KG= D/2= 0.3m
KG-KB= 0.03m
96
Prepared by Dr. Sudhir Sindagi

TPC, FWA and DWA


Tonnes per centimeter immersion (TPC): It is the weight required to be
added or to be removed from the ship to change the draft of a vessel by
1 cm.
Let AWP is the waterplane area of the vessel at any draft and is presumed
to remain constant for small change of draft.

97
Prepared by Dr. Sudhir Sindagi

TPC, FWA and DWA


TPC =AWP (m2) x 1 (cm) x ρSW
TPC = AWP (m2) x 1/100 (m) x ρSW
(note 1 is divided by 100 to convert centimeter to meter).
𝐀𝐖𝐏 ∗𝛒𝐒𝐖
𝐓𝐏𝐂 =
𝟏𝟎𝟎
The value of TPC will be different at different water density at the same
draft.
TPC is not constant for all the draft of a vessel. The water plane area of
a merchant vessel changes with the draft.
TPC is also not constant when the vessel is floating at same draft but in
water of different density
𝐓𝐏𝐂𝐃𝐖 𝛒𝐃𝐖
= 98
𝐓𝐏𝐂𝐒𝐖 𝛒𝐒𝐖
Prepared by Dr. Sudhir Sindagi

TPC, FWA and DWA


Derivation for Fresh Water Allowance (FWA = 𝐓𝐅𝐖 − 𝐓𝐒𝐖 )
Let us presume that the displacement of a vessel is Δ tonnes. The vessel is
floating in seawater, density ρSW (1.025 t/m3). It is presumed that a giant
crane lifts the vessel and put it in freshwater ρFW, (1.000 t/m3).
Volumetric change in displacement due to change in water density
d = (Δ/ρFW)– (Δ/ρSW)
d = Δ x (ρSW – ρFW) / (ρSW * ρFW)
If AWP is the area of the waterplane is assumed to remain constant for small
change of draft dT, then
d = AWP x dT = Δ x (ρSW – ρFW) / (ρSW * ρFW)
dT = FWA= Δ/ AWP x (ρSW – ρFW) / (ρSW * ρFW)
99
Prepared by Dr. Sudhir Sindagi

TPC, FWA and DWA


Derivation for Fresh Water Allowance (FWA = 𝐓𝐅𝐖 − 𝐓𝐒𝐖 )
FWA = Δ / AWP x (ρSW – ρFW) / (ρSW * ρFW)
Putting the values of ρSW=1.025 t/m3 and ρFW=1.000 t/m3 and multiplying and
dividing by 100
FWA = Δ / AWP x (1.025 – 1.0) / (ρSW *1.0*100/100
Rearranging the terms
FWA = Δ x 0.025 / (ρSW*AWP /100) *100
FWA = Δ x 0.025 / TPCsw *100
FWA = Δ/(4000*TPCsw) m
FWA = Δ/(40*TPCsw) cm
FWA =𝐓𝐅𝐖 − 𝐓𝐒𝐖 = Δ/(4*TPCsw) mm
100
Prepared by Dr. Sudhir Sindagi

TPC, FWA and DWA


Derivation for Dock Water Allowance (DWA = 𝐓𝐃𝐖 − 𝐓𝐒𝐖 )
𝐅𝐖𝐀 𝛒𝐒𝐖 −𝛒𝐃𝐖
𝐃𝐖𝐀 = 𝐓𝐃𝐖 −𝐓𝐒𝐖 = ∗
𝛒𝐃𝐖 𝛒𝐒𝐖 −𝛒𝐅𝐖

101
Prepared by Dr. Sudhir Sindagi

Problems on TPC, FWA and DWA


The waterplane area of the ship is 4525.3m2. If a mass of 100 tonnes is
added what will be increase in the draft (SW).
Given Data:
AWP=4525.3m2 w=100 tonnes added.
ΔTSW= ?
𝐀𝐖𝐏 ∗𝛒𝐒𝐖
Using equation 𝐓𝐏𝐂𝐒𝐖 = , we get
𝟏𝟎𝟎
𝐓𝐏𝐂𝐒𝐖 = 46.38 tonnes/cm
Change in draft due to the addition or the removal of weight
𝐖𝐞𝐢𝐠𝐡𝐭 𝐀𝐝𝐝𝐞𝐝 𝐨𝐫 𝐫𝐞𝐦𝐨𝐯𝐞𝐝
ΔT= 𝐜𝐦
𝐓𝐏𝐂
𝟏𝟎𝟎
ΔT= = 2.15 cm 102
𝟒𝟔.𝟑𝟖
Prepared by Dr. Sudhir Sindagi

Problems on TPC, FWA and DWA


A ship with draft of 8.2m in dock water of density 1.01t/m3. TPC in SW
=40. How much cargo the vessel will be able to load to bring her draft to
8.4m in the DW
Given Data:
TDW= 8.2m,, 𝐓𝐏𝐂𝐒𝐖 = 40 t/cm,
To have TDW= 8.4m, w=?
𝐓𝐏𝐂𝐃𝐖 𝛒𝐃𝐖
Using equation, = , 𝐰𝐞 𝐠𝐞𝐭
𝐓𝐏𝐂𝐒𝐖 𝛒𝐒𝐖
𝐓𝐏𝐂𝐃𝐖 =39.41 t/cm
Change in draft expected = 0.2m
Weight to be added = (ΔT) in cm* TPCDW
Weight to be added = 788.29 tonnes. 103
Prepared by Dr. Sudhir Sindagi

Problems on TPC, FWA and DWA


A ship if floating in fresh water at a draft of 6.8m. Its maximum allowable
draft in the fresh water is 7m. Its TPC in the SW at the corresponding
draft of 6.8m is 40. Find the available dead weight tonnage.
Given Data:
TFW= 6.8m (TFW)max=7.0m 𝐓𝐏𝐂𝐒𝐖 = 40 t/cm
Available deadweight=?
Available deadweight = (ΔT) in cm* TPCFW
Change in draft expected = 0.2m
𝐓𝐏𝐂𝐅𝐖 𝛒𝐅𝐖
Using equation, = , 𝐰𝐞 𝐠𝐞𝐭
𝐓𝐏𝐂𝐒𝐖 𝛒𝐒𝐖
𝐓𝐏𝐂𝐅𝐖 =39.0243 t/cm
Available deadweight = 780.48 tonnes 104
Prepared by Dr. Sudhir Sindagi

Problems on TPC, FWA and DWA


A ship of 10000tonnes displacement is floating in salt water. The ship
has to proceed to a berth where density of water is 1008kg/m3. Find how
much cargo must be discharged, if same salt water draft is to be
maintained. FWA= 200mm.
Given Data:
ΔSW=10000 tonnes 𝛒𝐃𝐖 1.008 t/m3 ,TSW= TDW , FWA= 200mm
Weight to be removed = ?
Change in draft expected by moving the ship from DW to SW = DWA
𝐅𝐖𝐀 𝛒𝐒𝐖 −𝛒𝐃𝐖
Using equation 𝐃𝐖𝐀 = 𝐓𝐃𝐖 −𝐓𝐒𝐖 = ∗
𝛒𝐃𝐖 𝛒𝐒𝐖 −𝛒𝐅𝐖
DWA = 134.92mm
Weight to be removed = DWA in cm * TPCDW 105
Prepared by Dr. Sudhir Sindagi

Problems on TPC, FWA and DWA


A ship of 10000tonnes displacement is floating in salt water. The ship
has to proceed to a berth where density of water is 1008kg/m3. Find how
much cargo must be discharged, if same salt water draft is to be
maintained. FWA= 200mm.
We know that, FWA =𝐓𝐅𝐖 − 𝐓𝐒𝐖 = Δ/(4*TPCSW) mm , we get
TPCSW = 12.5 t/cm
𝐓𝐏𝐂𝐃𝐖 𝛒𝐃𝐖
Using equation, = , 𝐰𝐞 𝐠𝐞𝐭
𝐓𝐏𝐂𝐒𝐖 𝛒𝐒𝐖
𝐓𝐏𝐂𝐃𝐖 =12.2926 t/cm
Weight to be removed = DWA in cm* TPCDW
Weight to be removed =165.85 tonnes.
106
Prepared by Dr. Sudhir Sindagi

Problems on TPC, FWA and DWA


A vessel of FWA 200mm goes from water of RD 1.018 to a water of RD
1.006. Find the change in draft and state whether there will be sinkage
or rise?
Given Data:
FWA= 200mm, 𝛒𝐃𝐖𝟏 =1.018 t/m3 , 𝛒𝐃𝐖𝟐 = 1.006 t/m3
ΔT= ? When moved from 𝛒𝐃𝐖𝟏 to 𝛒𝐃𝐖𝟏
Since, 𝛒𝐃𝐖𝟐 < 𝛒𝐃𝐖𝟏 < 𝛒𝐒𝐖 , hence 𝐓𝐃𝐖𝟐 > 𝐓𝐃𝐖𝟏 > 𝐓𝐒𝐖
ΔT When moved from 𝛒𝐃𝐖𝟏 to 𝛒𝐃𝐖𝟏 = DWA2- DWA1

107
Prepared by Dr. Sudhir Sindagi

Problems on TPC, FWA and DWA


A vessel of FWA 200mm goes from water of RD 1.018 to a water of RD
1.006. Find the change in draft and state whether there will be sinkage
or rise?
𝐅𝐖𝐀 𝛒𝐒𝐖 −𝛒𝐃𝐖
Using equation 𝐃𝐖𝐀 = 𝐓𝐃𝐖 −𝐓𝐒𝐖 = ∗
𝛒𝐃𝐖 𝛒𝐒𝐖 −𝛒𝐅𝐖
𝐅𝐖𝐀 𝛒𝐒𝐖 −𝛒𝐃𝐖𝟏
𝐃𝐖𝐀𝟏 = 𝐓𝐃𝐖𝟏 −𝐓𝐒𝐖 = ∗
𝛒𝐃𝐖 𝟏 𝛒𝐒𝐖 −𝛒𝐅𝐖
𝐅𝐖𝐀 𝛒𝐒𝐖 −𝛒𝐃𝐖𝟐
𝐃𝐖𝐀𝟐 = 𝐓𝐃𝐖𝟐 −𝐓𝐒𝐖 = ∗
𝛒𝐃𝐖 𝛒𝐒𝐖 −𝛒𝐅𝐖
𝟐

𝐃𝐖𝐀𝟏 = 55.009mm
𝐃𝐖𝐀𝟐 = 151.093mm
ΔT When moved from 𝛒𝐃𝐖𝟏 to 𝛒𝐃𝐖𝟐 = DWA2- DWA1
ΔT When moved from 𝛒𝐃𝐖𝟏 to 𝛒𝐃𝐖𝟐 =96.084mm 108
Prepared by Dr. Sudhir Sindagi

Problems on TPC, FWA and DWA


A vessel arrives at a port X at the mouth of a river. Her displacement at
that time was 12000tonnes and arrival draft is 5.77m in water with RD of
1.02. She has to cross a bar upriver before entering port Y. The depth of
water over the bar is 6m and RD 1.005. If her TPC in SW is 25, find the
minimum quantity of cargo to be offloaded at port X so that she can
cross the bar with an under-keel clearance of 0.5m
Given Data:
Δ= 12000 tonnes, 𝐓𝐃𝐖𝟏 = 5.77m 𝛒𝐃𝐖𝟏 =1.02 t/m3 , 𝛒𝐃𝐖𝟐 = 1.005 t/m3
Depth of water at Y port = 6m, 𝐓𝐏𝐂𝐒𝐖 =25, UKC at port Y =0.5m
Weight to be removed=?

109
Prepared by Dr. Sudhir Sindagi

Problems on TPC, FWA and DWA


We know that, ΔT When moved from 𝛒𝐃𝐖𝟏 to 𝛒𝐃𝐖𝟏 = DWA2- DWA1
Draft to be maintained in river Y = Depth of water in river Y – UKC
Draft to be maintained in river Y =6-0.5 =5.5m
Again, final draft in river Y = draft in river X + ΔT When moved from
𝛒𝐃𝐖𝟏 to 𝛒𝐃𝐖𝟏
Final draft in river Y = draft in river X + (DWA2- DWA1)

110
Prepared by Dr. Sudhir Sindagi

Problems on TPC, FWA and DWA


𝐅𝐖𝐀 𝛒𝐒𝐖 −𝛒𝐃𝐖
Using equation 𝐃𝐖𝐀 = 𝐓𝐃𝐖 −𝐓𝐒𝐖 = ∗
𝛒𝐃𝐖 𝛒𝐒𝐖 −𝛒𝐅𝐖
𝐅𝐖𝐀 𝛒𝐒𝐖 −𝛒𝐃𝐖𝟏
𝐃𝐖𝐀𝟏 = 𝐓𝐃𝐖𝟏 −𝐓𝐒𝐖 = ∗
𝛒𝐃𝐖 𝟏 𝛒𝐒𝐖 −𝛒𝐅𝐖
𝐅𝐖𝐀 𝛒𝐒𝐖 −𝛒𝐃𝐖𝟐
𝐃𝐖𝐀𝟐 = 𝐓𝐃𝐖𝟐 −𝐓𝐒𝐖 = ∗
𝛒𝐃𝐖 𝛒𝐒𝐖 −𝛒𝐅𝐖
𝟐

𝐃𝐖𝐀𝟏 = 23.5mm
𝐃𝐖𝐀𝟐 = 95.5mm
ΔT When moved from 𝛒𝐃𝐖𝟏 to 𝛒𝐃𝐖𝟏 = DWA2- DWA1
ΔT When moved from 𝛒𝐃𝐖𝟏 to 𝛒𝐃𝐖𝟏 =72mm
Final draft in river Y = draft in river X + (DWA2- DWA1)
Final draft in river Y = 5.77+0.072 = 5.842m
111
Expected draft in River Y = 5.5m
Prepared by Dr. Sudhir Sindagi

Problems on TPC, FWA and DWA


Expected change in draft = 5.842- 5.5 =0.342m
Weight to be removed = Expected change in draft in cm* TPCDW2
𝐓𝐏𝐂𝐃𝐖𝟐 𝛒𝐃𝐖𝟐
Using equation, = , 𝐰𝐞 𝐠𝐞𝐭
𝐓𝐏𝐂𝐒𝐖 𝛒𝐒𝐖
𝐓𝐏𝐂𝐃𝐖 =24.512 t/cm
Weight to be removed = Expected change in draft in cm* TPCDW2
Weight to be removed =838.31 tonnes

112
Prepared by Dr. Sudhir Sindagi

Load Line or Plimsol Line


Load line is a special marking positioned amidships which depicts the
draft of the vessel to which the ship can be loaded.
The fundamental purpose of a Load Line is to allot a maximum legal
limit up to which a ship can be loaded by cargo.
By prescribing such limits, the risk of having the vessel sailing with
inadequate freeboard and buoyancy can be limited.
A vessel should be having sufficient freeboard at all times, any
exceptions made will result in insufficient stability and excessive stress
on the ship’s hull.
However, since the buoyancy and immersion of the vessel largely
depend on the type of water and its density, it is not practical to define a
standard freeboard limit for the ship at all times. 113
Prepared by Dr. Sudhir Sindagi

Load Line or Plimsol Line


For this reason, the load line convention has put regulations which
divides the world into different geographical zones each having
different prescribed load line.
For example, A vessel sailing in Winter on North Atlantic Ocean will
have a greater freeboard than on a voyage in Tropical Zones and Fresh
waters.
All vessels of 24 meters and more are required to have this Load line
marking at the centre position of the length of summer load water line.
There are two types of Load line markings:-
– Standard Load Line marking – This is applicable to all types of vessels.
– Timber Load Line Markings – This is applicable to vessels carrying timber cargo

114
Prepared by Dr. Sudhir Sindagi

Load Line or Plimsol Line


Load lines are horizontal lines extending forward and aft from a vertical
line placed at a distance of 540mm from the centre of the disc.
They measure 230mm by 25mm.
The upper surfaces of the load lines indicate the maximum depths to
which the ships maybe submerged in different seasons and
circumstances.

115
Prepared by Dr. Sudhir Sindagi

Load Line or Plimsol Line


S – Summer :- It is the basic freeboard line at the same level as the
Plimsol Line. Other load lines are marked based on this Summer
freeboard line.
T – Tropical :- It is 1/48th of summer draft marked above the Summer
load line.
W – Winter :- It is 1/48th of summer draft marked below the SLL.
WNA – Winter North Atlantic :- It is marked 50mm below the Winter load
line. It applies to voyages in North Atlantic (above 36 degrees of
latitude) during winter months.
F – Fresh Water :- It is the summer fresh water load line. The distance
between S and F is the Fresh Water Allowance (FWA).
TF – Tropical Fresh Water :- It is the fresh water load line in Tropical.
116 It is
marked above the T at an amount equal to 1/48th of summer draft.
Prepared by Dr. Sudhir Sindagi

Load Line or Plimsol Line

117
Prepared by Dr. Sudhir Sindagi

Load Line or Plimsol Line

118
Prepared by Dr. Sudhir Sindagi

Problems on Load line


Calculation of Freeboard in various zones
FSW = Minimum Statutory Freeboard in SW ( Summer Freeboard)
𝐓𝐒𝐖
Freeboard in Tropical Zone = FT = 𝐅𝐒𝐖 − mm
𝟒𝟖
𝐓𝐒𝐖
Freeboard in Winter Zone = FW = 𝐅𝐒𝐖 + mm
𝟒𝟖
Freeboard in Winter North Atlantic Zone = FWNA = 𝐅𝐖 + 𝟓𝟎 𝐦𝐦
Freeboard in Fresh water Zone = FFW = 𝐅𝐒𝐖 − 𝐅𝐖𝐀 mm
𝐓𝐒𝐖
Freeboard in Tropical Fresh water Zone = FTF = 𝐅𝐒𝐖 − 𝐅𝐖𝐀 − mm
𝟒𝟖

119
Prepared by Dr. Sudhir Sindagi

Problems on Load line


Calculation of Drafts in various zones
TSW = Draft in SW ( Summer Freeboard)
𝐓𝐒𝐖
Draft in Tropical Zone = TT = 𝐓𝐒𝐖 + mm
𝟒𝟖
𝐓
Draft in Winter Zone = TW = 𝐓𝐒𝐖 − 𝐒𝐖 mm
𝟒𝟖
Draft in Winter North Atlantic Zone = TWNA = 𝐓𝐒𝐖 − 𝟓𝟎 𝐦𝐦
Draft in Fresh water Zone = TFW = 𝐓𝐒𝐖 + 𝐅𝐖𝐀 mm
𝐓𝐒𝐖
Draft in Tropical Fresh water Zone = TTF = 𝐓𝐒𝐖 + 𝐅𝐖𝐀 + mm
𝟒𝟖

120
Prepared by Dr. Sudhir Sindagi

Problems on Load line


A ship of 4477 tonnes displacement is less than 100m long. It floats at a
summer load line draft of 7m. Find the drafts of it in Winter, WNA,
Tropical, FW, Tropical FW load line markings. Area of waterplane is
622m2
Given Data:
Δ= 4477 tonnes, , T𝐒𝐖 =7m, 𝑨𝐖𝐏 =622m2
𝐀𝐖𝐏 ∗𝛒𝐒𝐖
𝐓𝐏𝐂𝐒𝐖 = = 637.55 t/cm
𝟏𝟎𝟎
FWA =𝐓𝐅𝐖 − 𝐓𝐒𝐖 = Δ/(4*TPCsw) mm
FWA =175mm

121
Prepared by Dr. Sudhir Sindagi

Problems on Load line


Calculation of Drafts in various zones
TSW = Draft in SW ( Summer Freeboard) =7m
𝐓𝐒𝐖
Draft in Tropical Zone = TT = 𝐓𝐒𝐖 + mm = 7.1458m
𝟒𝟖
𝐓
Draft in Winter Zone = TW = 𝐓𝐒𝐖 − 𝐒𝐖 mm =6.8541m
𝟒𝟖
Draft in Winter North Atlantic Zone = TWNA = 𝐓𝐒𝐖 − 𝟓𝟎 𝐦𝐦 =6.8041m
Draft in Fresh water Zone = TFW = 𝐓𝐒𝐖 + 𝐅𝐖𝐀 mm= 7.175m
𝐓𝐒𝐖
Draft in Tropical Fresh water Zone = TTF = 𝐓𝐒𝐖 + 𝐅𝐖𝐀 + mm
𝟒𝟖
TTF = 7.32m

122
Prepared by Dr. Sudhir Sindagi

Problems on Load line


A vessel is lying in a river berth of RD 1.01 with her summer load line
mark 20mm above the water on the starboard side and 50mm above on
the port side. Find how much cargo she can load to bring her to her “S”
line markings in SW, if her summer displacement is 15000 tonnes and
TPC 25.
Given Data:
𝛒𝐃𝐖 = 1.01 t/m3 Δ= 15000 tonnes, 𝐓𝐏𝐂𝐒𝐖 = 25
Weight to be added = ? When draft in SW will matching with S line.
If we assume mean WL in DW, then it will be 35mm below the S line.
Moreover, when ship moves from DW to SW, then draft will reduce by
the amount DWA.
Total expected change in draft in SW= DWA+35mm 123
Prepared by Dr. Sudhir Sindagi

Problems on Load line


𝐅𝐖𝐀 𝛒𝐒𝐖 −𝛒𝐃𝐖
Using equation 𝐃𝐖𝐀 = 𝐓𝐃𝐖 −𝐓𝐒𝐖 = ∗
𝛒𝐃𝐖 𝛒𝐒𝐖 −𝛒𝐅𝐖
𝐃𝐖𝐀 = 89.1mm
Total expected change in draft in SW= DWA+35mm = 124.1

124
Prepared by Dr. Sudhir Sindagi

Problems on Load line


FWA =𝐓𝐅𝐖 − 𝐓𝐒𝐖 = Δ/(4*TPCsw) mm
FWA =150mm
Weight to be added = ΔT in cm* TPCSW
Weight to be added =310.25 tonnes

125
Prepared by Dr. Sudhir Sindagi

Summary
Ship geometry, Definition of hull surface
Lines plan drawing of ships
Offset Table
Bonjean Curves
Archimedes principle, Displacements
Coefficients of form
Effect of Density and relative density of a liquid on drafts.
Meaning of buoyancy, reserve buoyancy
Center of Gravity and Center of Buoyancy of ship
TPC, FWA, DWA
Plimsoll line or Load line markings 126
Prepared by Dr. Sudhir Sindagi

Numerical Integration in Naval Architecture

By
Dr. Sudhir Sindagi 1
Prepared by Dr. Sudhir Sindagi

Chapter Content
Introduction
Use of integration in finding various properties
Rules of Integration
– Trapezoidal Rule
– Simpson’s (1/3)rd rule
– Simpson’s (3/8)th rule
Numerical
Simpson’s 5-8-(-1) rule
Simpson’s 3-10-(-1) rule

2
Prepared by Dr. Sudhir Sindagi

Introduction
As previously stated, the shape of a ship's hull cannot usually be
described by mathematical equations.
In order to calculate fundamental geometric properties of the hull, naval
architects use numerical methods.
This chapter describes two methods, the trapezoidal and Simpson’s
rules. The latter yields better approximations, but imposes a condition
on the input.
The following different properties of a hull form are estimated
– Area of Waterplane, centroid of waterplane: Center of Flotation, LCF & TPC
– Moment of Inertia of the Waterplane about Longitudinal Axis: Transverse MI (IT)
– Moment of Inertia of the Waterplane about Transverse Axis passing through
centroid: Longitudinal MI (IL)
3
– Underwater volume and its centroid (Center of Buoyancy), KB & LCB
Prepared by Dr. Sudhir Sindagi

Use of Integration
In order to find the various properties of an
area under the curve y=F(x)
Consider a small elemental area dxdy
Let 𝐱ത is the distance of the centroid of the
small elemental area from the Y axis and
𝐲ത is the distance of the centroid of the small
elemental area from the X axis
Let, x and y is the distance of any point A on
the curve from the x & y axis as shown
dA= area of small elemental area = ydx
𝐋
Total Area under the curve will be (A) = ‫𝐱𝐝𝐲 𝟎׬‬
4
Prepared by Dr. Sudhir Sindagi

Use of Integration
Position of Centroid from Y axis
ഥ = 𝐅𝐢𝐫𝐬𝐭 𝐌𝐨𝐦𝐞𝐧𝐭 𝐨𝐟 𝐀𝐫𝐞𝐚 𝐚𝐛𝐨𝐮𝐭 𝐚𝐱𝐢𝐬 𝐎𝐘
𝐗
𝐀𝐫𝐞𝐚
𝐋
‫𝐱𝐝𝐲𝐱 𝟎׬‬
ഥ=
𝐗 𝐋
‫𝐱𝐝𝐲 𝟎׬‬

Position of Centroid from X axis


ത = 𝐅𝐢𝐫𝐬𝐭 𝐌𝐨𝐦𝐞𝐧𝐭 𝐨𝐟 𝐀𝐫𝐞𝐚 𝐚𝐛𝐨𝐮𝐭 𝐚𝐱𝐢𝐬 𝐎𝐗
𝐘
𝐀𝐫𝐞𝐚
𝟏 𝐋 𝟐
∗‫𝐱𝐝 𝐲 ׬‬
ത=
𝐘 𝟐 𝟎
𝐋
‫𝐱𝐝𝐲 𝟎׬‬

5
Prepared by Dr. Sudhir Sindagi

Use of Integration
Moment of Inertia of the area about Longitudinal Axis: Transverse MI (IT)
IT= Second moment of area about OX
𝟏 𝐋 𝟑
IT= ∗ ‫𝐱𝐝 𝐲 𝟎׬‬
𝟑
Moment of Inertia of the area about Transverse Axis : Longitudinal MI (IL)
IL= Second moment of area about OY
𝐋 𝟐
IL= ‫𝐱𝐝𝐲 𝐱 𝟎׬‬

6
Prepared by Dr. Sudhir Sindagi

Numerical Integration
What does an integral represent?
d b
 f ( x)dxdy = volume
b
 a
f ( x )dx = area c a

Basic definition of an integral:


n f(x)
f ( x )dx = lim f ( xk )x
b

a n →
k =1

b−a
where x =
n

x
7
Prepared by Dr. Sudhir Sindagi

Trapezoidal Rule

fp

8
Prepared by Dr. Sudhir Sindagi

Trapezoidal Rule
Function f approximately by function fp. Then,

 fdx   f p dx
where fp is a linear polynomial interpolation, that is

fp =
(x − b ) f (a) + (x − a ) f (b)
(a − b ) (b − a )

f p dx =  f (a ) + f (b)
h
 fdx   2

where h =b−a 9
Prepared by Dr. Sudhir Sindagi

Trapezoidal Rule

fp

10
Prepared by Dr. Sudhir Sindagi

Trapezoidal Rule
For two interval, we can use summation operation
to derive the formula of two interval trapezoidal that
is

 fdx  2  f ( x0 ) + f ( x1 )+ 2  f ( x1 ) + f ( x2 )
h h

=  f ( x0 ) + 2 f ( x1 ) + f ( x2 )
h
2
where h = x2 − x0
2 11
Prepared by Dr. Sudhir Sindagi

Trapezoidal Rule

fp

12
Prepared by Dr. Sudhir Sindagi

Trapezoidal Rule
Similar to two interval trapezoidal, we can derive three interval
trapezoidal formula that is

 fdx  2  f ( x0 ) + 2 f ( x1 ) + 2 f ( x2 ) + f ( x3 )
h

x3 − x0
where h=
3
Thus, for n interval we have
h n −1

 fdx   f ( x0 ) + 2 f ( xi ) + f ( xn )
2 i =1 

where xn − x0
h=
n
Prepared by Dr. Sudhir Sindagi

Simpson’s 1/3rd Rule

fp

14
Prepared by Dr. Sudhir Sindagi

Simpson’s 1/3rd Rule


Function f approximately by function fp. Then,

 fdx   f p dx
where fp is a quadratic polynomial interpolation, that is

fp =
( x − x1 )(x − x2 )
f ( x0 ) +
( x − x0 )(x − x2 )
f ( x1 ) +
( x − x0 )(x − x1 )
f ( x2 )
(x0 − x1 )(x0 − x2 ) (x1 − x0 )(x1 − x2 ) (x2 − x0 )(x2 − x1 )

f p dx =  f ( x0 ) + 4 f ( x1 ) + f ( x2 )
h
 fdx   3
x2 − x0
where h= 15
2
Prepared by Dr. Sudhir Sindagi

Simpson’s 1/3rd Rule

fp

16
Prepared by Dr. Sudhir Sindagi

Simpson’s 1/3rd Rule


For 4 subinterval we have

 fdx  3  f ( x0 ) + 4 f ( x1 ) + 2 f ( x2 ) + 4 f ( x3 ) + f ( x4 )
h

x4 − x0
where h=
4
Thus, for n subinterval we have

h n −1 n−2 
 fdx   f ( x0 ) + 4  f ( xi ) + 2  f ( x j ) + f ( xn )
3 i =1, 3, 5 j = 2, 4, 6 
xn − x0
where h= 17
n
Prepared by Dr. Sudhir Sindagi

Simpson’s 3/8th Rule

fp
f

18
Prepared by Dr. Sudhir Sindagi

Simpson’s 3/8th Rule


Similar to 1/3 Simpson’s method, f approximately by function fp
where fp is a cubic polynomial interpolation, that is

fp =
( x − x1 )(x − x2 )(x − x3 )
f ( x0 ) +
( x − x0 )( x − x2 )( x − x3 )
f ( x1 )
(x0 − x1 )(x0 − x2 )(x0 − x3 ) (x1 − x0 )(x1 − x2 )(x1 − x3 )
+
(x − x0 )(x − x1 )(x − x3 ) f ( x ) + (x − x0 )(x − x1 )(x − x2 ) f ( x )
(x2 − x0 )(x2 − x1 )(x2 − x3 ) 2 (x3 − x0 )(x3 − x1 )(x3 − x2 ) 3

f p dx =  f ( x0 ) + 3 f ( x1 ) + 3 f ( x2 ) + f ( x3 )
3h
 fdx   8
x3 − x0
where h=
3
Prepared by Dr. Sudhir Sindagi

Simpson’s 3/8th Rule

Thus, for n subinterval we have


𝑛−1 𝑛−2
3ℎ
න𝑓𝑑𝑥 ≅ 𝑓(𝑥0 ) + 3 ෍ 𝑓(𝑥𝑖 ) + 2 ෍ 𝑓(𝑥𝑗 ) + 𝑓(𝑥𝑛 )
8
𝑖=1,2,4,5,7 𝑗=3,6,9..

where xn − x0
h=
n

20
Prepared by Dr. Sudhir Sindagi

Formulae

Trapezoidal Rule
h n −1

 fdx  2  f ( x0 ) + 2
i =1
f ( xi ) + f ( xn )
 h=
xn − x0
n
Simpson’s 1/3rd Rule

h n −1 n−2 
 fdx   f ( x0 ) + 4  f ( xi ) + 2  f ( x j ) + f ( xn )
3 i =1, 3, 5 j = 2, 4, 6 
Simpson’s 3/8th Rule
𝑛−1 𝑛−2
3ℎ
න𝑓𝑑𝑥 ≅ 𝑓(𝑥0 ) + 3 ෍ 𝑓(𝑥𝑖 ) + 2 ෍ 𝑓(𝑥𝑗 ) + 𝑓(𝑥𝑛 )
8 21
𝑖=1,2,4,5,7 𝑗=3,6,9..
Prepared by Dr. Sudhir Sindagi

Formulae

Trapezoidal Rule xn − x0
ℎ h=
න𝑓𝑑𝑥 ≅ 𝑓 𝑥0 + 𝟐 ∗ R + 𝑓(𝑥𝑛 ) n
2
Simpson’s 1/3rd Rule Trapezoidal multipliers


න𝑓𝑑𝑥 ≅ 𝑓 𝑥0 + 𝟒 ∗ Odd + 𝟐 ∗ Even + 𝑓(𝑥𝑛 )
3

Simpson’s 3/8th Rule Simpson’s multipliers


3ℎ
න𝑓𝑑𝑥 ≅ 𝑓 𝑥0 + 𝟐 ∗ Triplet + 3 ∗ R + 𝑓(𝑥𝑛 )
8
22
Prepared by Dr. Sudhir Sindagi

Applicability of Rules
Following conditions are necessary in using different rules
Sr Condition for the
Method Remark
No applicability
Trapezoidal Can be used for any number Least Accurate
1
Rule of ordinates method of all
Must be used for any odd More accurate
Simpson’s 1/3 rd
2 number of ordinates method than
Rule
N=1,3,5,7…. trapezoidal method
Most accurate
Must be used when number
Simpson’s 3/8th method, if in clash
of ordinates = 3*n+1, where
3 Rule with any method,
n=1,2,3…
then this method23to
No of ordinates = 4,7,10,13
be used.
Prepared by Dr. Sudhir Sindagi

Problems
A ship of 180m long has ½ widths of waterplane as follows
1, 7.5, 12, 13.5, 14, 14, 13.5, 12, 7 and 0m respectively. Calculate
waterplane area, LCF, TPC & waterplane area coefficient.
Given Data:
No of ordinates = 10
Hence using Simpson’s 3/8th rule.
𝐋𝐞𝐧𝐠𝐭𝐡 𝐨𝐟 𝐭𝐡𝐞 𝐬𝐡𝐢𝐩
𝐒𝐩𝐚𝐜𝐢𝐧𝐠 𝐛𝐞𝐭𝐰𝐞𝐞𝐧 𝐭𝐰𝐨 𝐬𝐭𝐚𝐭𝐢𝐨𝐧𝐬 = 𝐡 =
𝐧𝐮𝐦𝐛𝐞𝐫 𝐨𝐟 𝐨𝐫𝐝𝐢𝐧𝐚𝐭𝐞𝐬 −𝟏
𝟏𝟖𝟎
𝐡= = 20m
𝟏𝟎−𝟏
Use tabular method to solve such problems, as it will be very easy to
solve problems.
24
Prepare table as shown in the next slide
Prepared by Dr. Sudhir Sindagi

Problems
As shown, the first station will be AP and the last will be FP.
Since, it is half breadth plan, the values obtained through calculations
shall be multiplied by the factor of 2.
In between station 4 and 5, the midship lies with the biggest section.

25
Prepared by Dr. Sudhir Sindagi

Problems
Center of Flotation (F or CF): It is centroid of the water plane area taken
at the SLL. It is the point about which the ship trims.
Longitudinal distance of the F from the AP is known as the LCF

26
Prepared by Dr. Sudhir Sindagi

Problems
Col(1) Col (2) Col (3) Col(4)= (2)X(3) Col(5) Col(6)=(4)X(5)

Simpson’s Lever from


Station ½ Widths For Area For Moment @ AP
Multiplier AP
0(AP) 1
1 7.5
2 12
3 13.5
4 14
5 14
6 13.5
7 12
8 7
9 (FP) 0 27

ΣA=00m2 ΣM=00m3
Prepared by Dr. Sudhir Sindagi

Problems
Col(1) Col (2) Col (3) Col(4)= (2)X(3) Col(5) Col(6)=(4)X(5)

Simpson’s Lever from


Station ½ Widths For Area For Moment @ AP
Multiplier AP
0(AP) 1 1 1 0 0
1 7.5 3 22.5 1 22.5
2 12 3 36 2 72
3 13.5 2 27 3 81
4 14 3 42 4 168
5 14 3 42 5 210
6 13.5 2 27 6 162
7 12 3 36 7 252
8 7 3 21 8 168
9 (FP) 0 1 0 9 028
ΣA=254.5m2 ΣM=1135.5m3
Prepared by Dr. Sudhir Sindagi

Problems
As per the Simpson’s 3/8th rule
𝟑𝐡
Total Area of the waterplane=A=2* * ΣA
𝟖
A = 3817.5 m2
We know that, Position of Centroid from Y axis
ഥ = 𝐅𝐢𝐫𝐬𝐭 𝐌𝐨𝐦𝐞𝐧𝐭 𝐨𝐟 𝐀𝐫𝐞𝐚 𝐚𝐛𝐨𝐮𝐭 𝐚𝐱𝐢𝐬 𝐎𝐘
𝐗
𝐀𝐫𝐞𝐚
𝟑𝐡
1st Moment of the area of the waterplane about the AP= 2*h* * ΣM
𝟖
1st Moment of the area about the AP = 340650 m3
𝐅𝐢𝐫𝐬𝐭 𝐌𝐨𝐦𝐞𝐧𝐭 𝐨𝐟 𝐀𝐫𝐞𝐚 𝐚𝐛𝐨𝐮𝐭 𝐀𝐏
LCF =
𝐀𝐫𝐞𝐚
LCF =89.23m from AP
29
Prepared by Dr. Sudhir Sindagi

Problems
One more way to get location of centroid
𝐅𝐢𝐫𝐬𝐭 𝐌𝐨𝐦𝐞𝐧𝐭 𝐨𝐟 𝐀𝐫𝐞𝐚 𝐚𝐛𝐨𝐮𝐭 𝐀𝐏 2∗h∗𝟑𝐡 ∗ ΣM ΣM
LCF = = 𝟖
=h∗
𝐀𝐫𝐞𝐚 2∗ 𝟖 ∗ ΣA
𝟑𝐡
ΣA
ΣM
LCF = h∗
ΣA
LCF =89.23m from AP
𝐀𝐖𝐏 ∗𝛒𝐒𝐖
𝐔𝐬𝐢𝐧𝐠, 𝐓𝐏𝐂 =
𝟏𝟎𝟎
TPC= 39.1293 t/cm
𝐀𝐖𝐏
Using equation: 𝐂𝐖𝐏 =
𝐋 ∗𝐁
Here, B = 14*2 = 28m
30
𝐂𝐖𝐏 = 0.7475
Prepared by Dr. Sudhir Sindagi

Problems
A ship is floating on an even keel at 6m draft. The areas of the
waterplane are as follows. Find the ship’s KB at this draft.
Draft(m) 0 1 2 3 4 5 6
AWP(m2) 5000 5600 6020 6025 6025 6025 6025

Given Data:
No of ordinates = 7
Using Simpson’s 1/3rd rule.
𝐡= 1m
When waterplanes are
provided, then we will get
31
underwater Volume and KB
Prepared by Dr. Sudhir Sindagi

Problems
Col(1) Col (2) Col (3) Col(4)= (2)X(3) Col(5) Col(6)=(4)X(5)

Simpson’s Lever from For Moment @


Draft AWP For Volume
Multiplier Keel Keel
0 5000
1 5600
2 6020
3 6025
4 6025
5 6025
6 6025
ΣV=00m3 ΣM=00m4

32
Prepared by Dr. Sudhir Sindagi

Problems
Col(1) Col (2) Col (3) Col(4)= (2)X(3) Col(5) Col(6)=(4)X(5)

Simpson’s Lever from For Moment @


Draft AWP For Volume
Multiplier Keel Keel
0 5000 1 5000 0 0
1 5600 4 22400 1 22400
2 6020 2 12040 2 24080
3 6025 4 24100 3 72300
4 6025 2 12050 4 48200
5 6025 4 24100 5 120500
6 6025 1 6025 6 36150
ΣV= 105715 ΣM= 323630

33
Prepared by Dr. Sudhir Sindagi

Problems
As per the Simpson’s 1/3rd rule
𝐡
Total Volume=V= * ΣV
𝟑
V = 35238.33 m3
We know that, Position of Centroid from Keel
ഥ = 𝐅𝐢𝐫𝐬𝐭 𝐌𝐨𝐦𝐞𝐧𝐭 𝐨𝐟 𝐭𝐡𝐞 𝐕𝐨𝐥𝐮𝐦𝐞 𝐚𝐛𝐨𝐮𝐭 𝐭𝐡𝐞 𝐊𝐞𝐞𝐥
𝐗
𝐕𝐨𝐥𝐮𝐦𝐞
𝐡
1st Moment of the volume about the keel= h* * ΣM
𝟑
1st Moment of the volume about the keel = ??
𝐅𝐢𝐫𝐬𝐭 𝐌𝐨𝐦𝐞𝐧𝐭 𝐨𝐟 𝐭𝐡𝐞 𝐯𝐨𝐥𝐮𝐦𝐞 𝐚𝐛𝐨𝐮𝐭 𝐭𝐡𝐞 𝐤𝐞𝐞𝐥 ΣM
KB = = =h∗
𝐕𝐨𝐥𝐮𝐦𝐞 ΣV
KB =3.06m from Keel 34
Prepared by Dr. Sudhir Sindagi

Problems
A 200m long vessel has half ordinates of a waterplane as below
commencing from AP. Calculate
– Waterplane area & LCF from AP
– TPC in SW
– MI of the waterplane area @ transverse axis through LCF
– MI of the waterplane area @ longitudinal axis (CL)
Station 0(AP) 1 2 3 4 5 6 7 8 9 10
½ Ord 0 10 13 14 14.2 14.2 14.1 14 11.5 6.2 0

Given Data:
No of ordinates = 11, hence using using Simpson’s 1/3rd rule.
𝐋𝐞𝐧𝐠𝐭𝐡 𝐨𝐟 𝐭𝐡𝐞 𝐬𝐡𝐢𝐩
𝐒𝐩𝐚𝐜𝐢𝐧𝐠 𝐛𝐞𝐭𝐰𝐞𝐞𝐧 𝐭𝐰𝐨 𝐬𝐭𝐚𝐭𝐢𝐨𝐧𝐬 = 𝐡 =
𝐧𝐮𝐦𝐛𝐞𝐫 𝐨𝐟 𝐨𝐫𝐝𝐢𝐧𝐚𝐭𝐞𝐬 −𝟏 35
𝐡 = 20m
Prepared by Dr. Sudhir Sindagi

Problems
(1) (2) (3) (4)= (2)X(3) (5) (6)=(4)X(5) (7)=(5)*(6) (8)=(2)3 (9)=(3)*(8)

Station ½ Ord SM For Area Lever For 1st M@AP For IL For y3 For IT
0(AP) 0
1 10
2 13
3 14
4 14.2
5 14.2
6 14.1
7 14
8 11.5
9 6.2
10 (FP) 0
36
ΣA=00m2 ΣM1=00m3 ΣM2=00m4 ΣIT=00m4
Prepared by Dr. Sudhir Sindagi

Problems
(1) (2) (3) (4)= (2)X(3) (5) (6)=(4)X(5) (7)=(5)*(6) (8)=(2)3 (9)=(3)*(8)

Station ½ Ord SM For Area Lever For 1st M@AP For IL For IT
0(AP) 0 1 0 0 0 0 0 0
1 10 4 40 1 40 40 1000 4000
2 13 2 26 2 52 104 2197 4394
3 14 4 56 3 168 504 2744 10976
4 14.2 2 28.4 4 113.6 454.4 2863.288 5726.576
5 14.2 4 56.8 5 284 1420 2863.288 11453.15
6 14.1 2 28.2 6 169.2 1015.2 2803.221 5606.442
7 14 4 56 7 392 2744 2744 10976
8 11.5 2 23 8 184 1472 1520.875 3041.75
9 6.2 4 24.8 9 223.2 2008.8 238.328 953.312
10 (FP) 0 1 0 10 0 0 0 0
37
ΣA=339.2 ΣM =1626 ΣM =9762.4 ΣI =57127
Prepared by Dr. Sudhir Sindagi

Problems
As per the Simpson’s 1/3rd rule
𝐡
Total Area=A=2* * ΣA
𝟑
A = 4522.66m2
𝐅𝐢𝐫𝐬𝐭 𝐌𝐨𝐦𝐞𝐧𝐭 𝐨𝐟 𝐭𝐡𝐞 𝐚𝐫𝐞𝐚 𝐚𝐛𝐨𝐮𝐭 𝐀𝐏 ΣM1
LCF= = =h∗
𝐀𝐫𝐞𝐚 ΣA
LCF=95.87m from AP
𝐀𝐖𝐏 ∗𝛒𝐒𝐖
𝐔𝐬𝐢𝐧𝐠, 𝐓𝐏𝐂 =
𝟏𝟎𝟎
TPC= 46.357 t/cm
Moment of Inertia of the area about Transverse Axis : Longitudinal MI
(IL) =IL= Second moment of area about AP
𝐋 38
IL= ‫𝐱𝐝𝐲 𝟐 𝐱 𝟎׬‬
Prepared by Dr. Sudhir Sindagi

Problems
𝐡
IL@ AP = 2*h2* * ΣM2
𝟑
IL@ AP =52066133.33m4
However, we need to estimate the MI about an axis passing through
center of flotation.
Hence, using parallel axis theorem
MI about any axis = MI about F +AWP*d2
MI about F = MI about AP - AWP*d2
Here d= LCF= 95.87m
IL@ F =10498107.93m4

39
Prepared by Dr. Sudhir Sindagi

Problems
Moment of Inertia of the area about Longitudinal Axis: Transverse MI (IT)
IT= Second moment of area about centreline
𝟏 𝐋
IT= ∗ ‫𝐱𝐝 𝟑 𝐲 𝟎׬‬
𝟑
𝟏 𝐡
IT = 2* ** * ΣIT
𝟑 𝟑
IT = 253897.77m4

40
Prepared by Dr. Sudhir Sindagi

Problems
The TPC values for a ship at 1.2m intervals of draught commencing at
the keel are 8.2, 16.5, 18.7, 19.4, 20.0, 20.5 and 21.1 respectively.
Calculate displacement at 7.2m draught
Given Data:
No of ordinates = 7
Hence using Simpson’s 3/8th rule.
𝐡 =1.2m
𝐀 ∗𝛒
When TPC values are provided, u𝐬𝐢𝐧𝐠 𝐓𝐏𝐂 = 𝐖𝐏 𝐒𝐖, one needs to
𝟏𝟎𝟎
calculate AWP at different drafts.
And then using those values of , one calculate the displacement and
KB, if required, as per the procedure mentioned in problem 2.
41
Prepared by Dr. Sudhir Sindagi

Problems
Col(5)=
Col(1) Col (2) Col (3) Col (4) Col(6) Col(7)=(5)X(6)
(3)X(4)
Simpson’s Lever from For Moment @
Draft TPC AWP For Volume
Multiplier Keel Keel
0 8.2
1 16.5
2 18.7
3 19.4
4 20.0
5 20.5
6 21.1
ΣV=00m3 ΣM=00m4

42
Prepared by Dr. Sudhir Sindagi

Problems
Col(1) Col (2) Col (3) Col (4) Col(5)= (3)X(4) Col(6) Col(7)=(5)X(6)

Simpson’s Lever from For Moment @


Draft TPC AWP For Volume
Multiplier Keel Keel
0 8.2 800 1 800 0 0
1 16.5 1609.75 3 4829.25 1 4829.25
2 18.7 1824.39 3 5473.17 2 10946.34
3 19.4 1892.68 2 3785.36 3 11356.08
4 20.0 1951.21 3 5853.63 4 23414.52
5 20.5 2000 3 6000 5 30000
6 21.1 2058.53 1 2058.53 6 12351.18
ΣV=28799.94m3 ΣM=92897.37m4

43
Prepared by Dr. Sudhir Sindagi

Problems
As per the Simpson’s 3/8th rule
𝟑𝐡
Total Volume=V= * ΣV
𝟖
V = 12959.97 m3
Δ=ρSW *V
Δ=13283.97 tonnes
𝐅𝐢𝐫𝐬𝐭 𝐌𝐨𝐦𝐞𝐧𝐭 𝐨𝐟 𝐭𝐡𝐞 𝐯𝐨𝐥𝐮𝐦𝐞 𝐚𝐛𝐨𝐮𝐭 𝐭𝐡𝐞 𝐤𝐞𝐞𝐥 ΣM
KB = = =h∗
𝐕𝐨𝐥𝐮𝐦𝐞 ΣV
KB =3.87m from Keel

44
Prepared by Dr. Sudhir Sindagi

Problems
A ship is floating upright in SW on an even keel at 7m draft. The TPCs
are as follows.
Draft(m) 1 2 3 4 5 6 7
TPC in
60 60.3 60.5 60.5 60.5 60.5 60.5
Tonnes
The volume between the outer bottom and 1m draft is 3044m3 and
centroid is 0.5m above the keel. Find the Ship’s KB
Given Data:
No of ordinates = 7
Hence using Simpson’s 3/8th rule.
𝐡 =1m
45
Initially, follow the similar procedure as mentioned in previous problem.
Prepared by Dr. Sudhir Sindagi

Problems

46
Prepared by Dr. Sudhir Sindagi

Problems
Col(5)=
Col(1) Col (2) Col (3) Col (4) Col(6) Col(7)=(5)X(6)
(3)X(4)
Simpson’s Lever from For Moment @
Draft TPC AWP For Volume
Multiplier WL at 1m WL at 1m
1 60
2 60.3
3 60.5
4 60.5
5 60.5
6 60.5
7 60.5
ΣV=00m3 ΣM=00m4

47
Prepared by Dr. Sudhir Sindagi

Problems
Col(1) Col (2) Col (3) Col (4) Col(5)= (3)X(4) Col(6) Col(7)=(5)X(6)

Simpson’s Lever from For Moment @


Draft TPC AWP For Volume
Multiplier WL at 1m WL at 1m
0 60 5853.659 1 5853.659 0 0
1 60.3 5882.927 3 17648.78 1 17648.78
2 60.5 5902.439 3 17707.32 2 35414.63
3 60.5 5902.439 2 11804.88 3 35414.63
4 60.5 5902.439 3 17707.32 4 70829.27
5 60.5 5902.439 3 17707.32 5 88536.59
6 60.5 5902.439 1 5902.439 6 35414.63
ΣV=94331.71m3 ΣM=283258.54m4

48
Prepared by Dr. Sudhir Sindagi

Problems
As per the Simpson’s 3/8th rule
𝟑𝐡
Total Volume=V= * ΣV
𝟖
V = 35374.39 m3
Total underwater volume = VT= V+ Additional volume of appendages
Total underwater volume = VT= 35374.39 +3044 = 38418.39 m3
𝐅𝐢𝐫𝐬𝐭 𝐌𝐨𝐦𝐞𝐧𝐭 𝐨𝐟 𝐭𝐡𝐞 𝐯𝐨𝐥𝐮𝐦𝐞 𝐚𝐛𝐨𝐮𝐭 𝐭𝐡𝐞 𝐤𝐞𝐞𝐥 ΣM
KB = = h∗
𝐕𝐨𝐥𝐮𝐦𝐞 ΣV
KB =3.002m from WL at 1m
But actual KB =3.002 +1 = 4.002m from Keel
𝐕∗𝐊𝐁+𝟑𝟎𝟒𝟒∗𝟎.𝟓
KB for total volume =
VT
49
KB for total volume = 3.724m from Keel
Prepared by Dr. Sudhir Sindagi

Problems
A 300m long vessel has a waterplane with half ordinates commencing
from AP are as below.
Station 0 ½ 1 2 3 4 5 5½ 6
½ ord 0.1 7.5 10 12 12.3 11.4 8 5.2 1.0
Find the area of the waterplane if there are appendages forward and aft
with a total area of 2.8m2 to be added to the main area
Given Data:
No of stations = 7.
𝐋𝐞𝐧𝐠𝐭𝐡 𝐨𝐟 𝐭𝐡𝐞 𝐬𝐡𝐢𝐩
𝐡=
𝐧𝐮𝐦𝐛𝐞𝐫 𝐨𝐟 𝐨𝐫𝐝𝐢𝐧𝐚𝐭𝐞𝐬 −𝟏
𝐡 = 50m
50
Prepared by Dr. Sudhir Sindagi

Problems
It is the problem with half stations, which are included at the forward
and the aft end. In such cases, needed to divide stations and ordinates
as per the length between stations.
The 1st group will contain 0, ½ and 1 stations, the 2nd group will contain
1,2,3,4&5 stations, the 3rd group will contain 5,5 ½ and 6 stations.
After divisions of stations in the subgroups, need to check how many
stations are included in the groups and then accordingly one can use
particular method to estimate.
Here, in this particular problem, all 3 groups have odd number of
ordinates, hence we can use Simpson’s 1/3rd rule for all of them.
For the group 1 and 3, the Simpson's multiplier will be multiplied with ½
and for the common stations, SMs will be added
51
Levers will be similar to that of stations.
Prepared by Dr. Sudhir Sindagi

Problems
Col(1) Col (2) Col (3) Col(4)= (2)X(3) Col(5) Col(6)=(4)X(5)

Simpson’s Lever from


Station ½ Widths For Area For Moment @ AP
Multiplier AP
0 0.1
½ 7.5 Group 1
1 10
2 12
3 12.3 Group 2
4 11.4
5 8
5½ 5.2 Group 3
6 1.0
ΣA=00m2 ΣM=00m3 52
Prepared by Dr. Sudhir Sindagi

Problems
Col(1) Col (2) Col (3) Col(4)= (2)X(3) Col(5) Col(6)=(4)X(5)

Simpson’s Lever from


Station ½ Widths For Area For Moment @ AP
Multiplier AP
0 0.1 1*½ =½
½ 7.5 4*½ =2
1 10 1*½ +1=1 ½
2 12 4
3 12.3 2
4 11.4 4
5 8 1*½ +1= 1 ½
5½ 5.2 4*½ =2
6 1.0 1*½ =½
ΣA=0m2 ΣM=0m3 53
Prepared by Dr. Sudhir Sindagi

Problems
Col(1) Col (2) Col (3) Col(4)= (2)X(3) Col(5) Col(6)=(4)X(5)

Simpson’s Lever from


Station ½ Widths For Area For Moment @ AP
Multiplier AP
0 0.1 1*½ =½ 0.05 0 0
½ 7.5 4*½ =2 15 0.5 7.5
1 10 1*½ +1=1 ½ 15 1 15
2 12 4 48 2 96
3 12.3 2 24.6 3 73.8
4 11.4 4 45.6 4 182.4
5 8 1*½ +1= 1 ½ 12 5 60
5½ 5.2 4*½ =2 10.4 5.5 57.2
6 1.0 1*½ =½ 0.5 6 3
ΣA=171.15m2 ΣM=494.9m543
Prepared by Dr. Sudhir Sindagi

Problems
As per the Simpson’s 1/3rd rule
𝐡
Total Area of the waterplane=A=2* * ΣA +2.8
𝟑
A = 5707.8 m2
ΣM
LCF from AP =h∗ ( Here, the effect of additional area is neglected, as
ΣA
distance of its centroid from AP is not provided).
LCF =144.58m from AP

55
Prepared by Dr. Sudhir Sindagi

Problems
The Water plane of a ship at a particular draught has the ordinates, 20
meters apart, of magnitude 1.6m, 13.6m, 26.5m, 25.1m, 10.6m, and 2.1m.
There is also an intermediate ordinate, midway between the first two
ordinates, of magnitude 9.4m. Find the area of the water plane
Given Data:
This problem is exactly similar to the previous problem, except there is
only one half station inserted between stations 1 and 2.
h=20m.
Here there will be only two groups: The 1st group will contain 0, ½ and 1
stations, the 2nd group will contain 1,2,3,4&5 stations.
As both the groups have the odd number of ordinates, hence we can
use Simpson’s 1/3rd rule for them. 56
Levers will be similar to that of stations.
Prepared by Dr. Sudhir Sindagi

Problems
Col(1) Col (2) Col (3) Col(4)= (2)X(3) Col(5) Col(6)=(4)X(5)

Simpson’s Lever from


Station ½ Widths For Area For Moment @ AP
Multiplier AP
0 1.6
½ 9.4 Group 1
1 13.6
2 26.5
3 25.1 Group 2
4 10.6
5 2.1
ΣA=00m2 ΣM=00m3

57
Prepared by Dr. Sudhir Sindagi

Problems
Col(1) Col (2) Col (3) Col(4)= (2)X(3) Col(5) Col(6)=(4)X(5)

Simpson’s Lever from


Station ½ Widths For Area For Moment @ AP
Multiplier AP
0 1.6 1*½ =½
½ 9.4 4*½ =2
1 13.6 1*½ +1=1 ½
2 26.5 4
3 25.1 2
4 10.6 4
5 2.1 1
ΣA=0m2 ΣM=0m3

58
Prepared by Dr. Sudhir Sindagi

Problems
Col(1) Col (2) Col (3) Col(4)= (2)X(3) Col(5) Col(6)=(4)X(5)

Simpson’s Lever from


Station ½ Widths For Area For Moment @ AP
Multiplier AP
0 1.6 1*½ =½ 0.8 0 0
½ 9.4 4*½ =2 18.8 0.5 9.4
1 13.6 1*½ +1=1 ½ 20.4 1 20.4
2 26.5 4 106 2 212
3 25.1 2 50.2 3 150.6
4 10.6 4 42.4 4 169.6
5 2.1 1 2.1 5 10.5
ΣA=240.7m2 ΣM=572.5m3

59
Prepared by Dr. Sudhir Sindagi

Problems
As per the Simpson’s 1/3rd rule
𝐡
Total Area of the waterplane=A=2* * ΣA
𝟑
A = 3209.33 m2
ΣM
LCF from AP =h∗
ΣV
LCF =47.569m from AP

60
Prepared by Dr. Sudhir Sindagi

Problems
The areas of water plane, 2.5 metres apart, of a Tanker is given below:-
Calculate the volume of displacement and the position of VCB.
Water
1 2 3 4 5 5½ 6
plane
AWP(m2) 4010 4000 3800 3100 1700 700 200

Given Data:
Answers are
Underwater volume= 36716.667m3
Δ= 37634.58 tonnes.
KB= 7.65m from keel.
61
Prepared by Dr. Sudhir Sindagi

Simpson’s 5-8-(-1) Rule


Simpson's 3rd rule
Also known as the 5–8–1 rule, Simpson's third rule is used to find the
area between two consecutive ordinates when three consecutive
ordinates are known.
The following equation estimates the area in the left half of the figure
between ordinates y0 and y1.
𝐡
Area= ∗ (𝟓𝐲𝟎 + 𝟖𝐲𝟏 − 𝐲𝟐)
𝟏𝟐

62
Prepared by Dr. Sudhir Sindagi

Simpson’s 3-10-(-1) Rule


Simpson’s 3-10-(-1) Rule
It is used to find moment of the area between two consecutive ordinates
about an axis passing through the first ordinate, when three consecutive
ordinates are known.
The following equation estimates moment of the area between ordinates
y0 and y1 about the first ordinate
Moment of Area @ y0=
𝐡𝟐
∗ (𝟑𝐲𝟎 + 𝟏𝟎𝐲𝟏 − 𝐲𝟐)
𝟐𝟒

63
Prepared by Dr. Sudhir Sindagi

Problems
A vessel has the following waterplane areas at the given drafts.
Draft
1 2 3 4 5 6
(m)
AWP(m2) 350 2500 3450 3960 4000 4030
When in light weight condition, the vessel floats at draft of 2m and when
in fully loaded condition, it floats at the draft of 6m. Find its lightweight
and total displacement in tonnes. Also calculate its load carrying
capacity
Given Data:
No of ordinates are 6, hence cannot be solved by earlier Simpson's
rules. For the first two WLs 3rd rule can be applied to get volumetric
displacement in lightship condition and for next 5 WLs 1st rule 64 can
applied to get volumetric displacement corresponding to deadweight.
Prepared by Dr. Sudhir Sindagi

Simpson’s 5-8-(-1) Rule


Applying Simpson's 3rd rule for 1st two WLs
𝐡
Volume between WL1 and WL2 = ∗ (𝟓𝐲𝟎 + 𝟖𝐲𝟏 − 𝐲𝟐)
𝟏𝟐
Volume between WL1 and WL2 = 1525m3.
Applying Simpson’s 1st rule for next five WLs
𝐡
Volume between WL2 and WL6 = ∗ [𝟐𝟓𝟎𝟎 + 𝟒𝟎𝟑𝟎 + 𝟐 ∗ 𝟑𝟗𝟔𝟎 + 𝟒 ∗
𝟑
𝟑𝟒𝟓𝟎 + 𝟒𝟎𝟎𝟎 ]
Volume between WL2 and WL6 =14750m3.
Total displacement = Total volume* ρSW
Total displacement= 16681.875 tonnes
Load carrying capacity = deadweight= 14750* ρSW =16681.875 -1525* ρSW
65
Load carrying capacity = deadweight=15118.75tonnes
Prepared by Dr. Sudhir Sindagi

Problems
The half ordinates of a midship section of a ship at 5 waterlines at 4m
intervals are. WL 1 2 3 4 5
½ Breadth
6.4 10.9 12.8 13.6 14.2
(m)
Find the area between waterlines 4&5 and height of the centroid of this
layer above the base
Given Data:
Here both rules 5-8-1 and 3-10-1 rule
can be used

66
Prepared by Dr. Sudhir Sindagi

Simpson’s 5-8-(-1) Rule


Applying Simpson's 3rd rule between WL4 and WL5
𝐡
Area between WL4 and WL5 =2* ∗ (𝟓𝐲𝟓 + 𝟖𝐲𝟒 − 𝐲𝟑)
𝟏𝟐
Area between WL4 and WL5 = 111.33m2.
Applying Simpson’s 3-10-1 rule
𝐡𝟐
Moment of Area @ WL5 = 𝟐 ∗ ∗ (𝟑𝐲𝟓 + 𝟏𝟎𝐲𝟒 − 𝐲𝟑)
𝟐𝟒
Moment of Area @ WL5 = 221.06m3.
We know that, Position of Centroid from any axis
𝐅𝐢𝐫𝐬𝐭 𝐌𝐨𝐦𝐞𝐧𝐭 𝐨𝐟 𝐀𝐫𝐞𝐚 𝐚𝐛𝐨𝐮𝐭 𝐭𝐡𝐞 𝐚𝐱𝐢𝐬 𝟐𝟐𝟏.𝟎𝟔

𝐗= = =1.98m from WL5
𝐀𝐫𝐞𝐚 𝟏𝟏𝟏.𝟑𝟑
Distance of he centroid from the keel = 4*4-1.98
Distance of he centroid from the keel = 14.02m 67
Prepared by Dr. Sudhir Sindagi

Problems
The ½ ordinates of a waterplane 120 m long are as follows.
Calculate
– waterplane area
– Distance of centroid from midship.
– Second moment of area of waterplane about centroid
Draft(
AP 0.5 1 1.5 2 3 4 5 6 7 8 8.5 9 9.5 FP
m)
½ Ord
1.2 3.5 5.3 6.8 8 8.3 8.5 8.5 8.5 8.4 8.2 7.9 6.2 3.5 0
(m)

68
Prepared by Dr. Sudhir Sindagi

Summary
Introduction
Use of integration in finding various properties
Rules of Integration
– Trapezoidal Rule
– Simpson’s (1/3)rd rule
– Simpson’s (3/8)th rule
Numerical
Simpson’s 5-8-(-1) rule
Simpson’s 3-10-(-1) rule

69
Prepared by Dr. Sudhir Sindagi

CG and angle of list of the ship

By
Dr. Sudhir Sindagi 1
Prepared by Dr. Sudhir Sindagi

Chapter Content
Introduction
CG and CB of the ship: List and Trim
Estimation of the CG of the ship
Shift in CG of the ship
– Due to shifting in existing weight
– Addition of weight
– Removal of existing of weight
– Effect of suspended weight
Angle of List

2
Prepared by Dr. Sudhir Sindagi

Introduction
The location of a ship's vertical center of gravity (LCG, TCG and VCG) is
an important measurement needed to estimate the initial stability of a
vessel.
Centre of gravity is the point of a body at which all the mass of the body
may be assumed to be concentrated.
The force of gravity acts vertically downwards from this point with a
force equal to the weight of the body.
B - Center of Buoyancy: The geometric center of the ship's underwater
hull body. Centroid of the underwater volume is the CB of the ship.
It is the point at which all the forces of buoyancy may be considered to
act in a vertically upward direction.
The Center of Buoyancy will move as the shape of the underwater 3
portion of the hull body changes.
Prepared by Dr. Sudhir Sindagi

CG and CB of the ship


When a ship is at
equilibrium, the centre of
buoyancy is vertically in
line with the centre of
gravity of the ship

4
Prepared by Dr. Sudhir Sindagi

CG and CB of the ship


Longitudinal Center of Gravity (LCG): It is the Longitudinal distance of
the CG from the AP.
Vertical Center of Gravity (VCG of KG): It is the vertical distance of the
CG from the keel.
Transverse Center of Gravity (TCG): It is the transverse distance of the
CG from the centreline of the ship.

5
Prepared by Dr. Sudhir Sindagi

CG and CB of the ship


For the upright condition of the ship TCG=TCB=0 as both CG and CB of
the ship lie on centreline.
If TCG≠ 𝐓𝐂𝐁, 𝐭𝐡𝐞𝐧 𝐭𝐡𝐞 𝐬𝐡𝐢𝐩 𝐰𝐢𝐥𝐥 𝐋𝐢𝐬𝐭

6
Prepared by Dr. Sudhir Sindagi

CG and CB of the ship


For the upright condition of the ship TCG=TCB=0 as both CG and CB of
the ship lie on centreline.
If TCG≠ 𝐓𝐂𝐁, 𝐭𝐡𝐞𝐧 𝐭𝐡𝐞 𝐬𝐡𝐢𝐩 𝐰𝐢𝐥𝐥 𝐋𝐢𝐬𝐭
Heel. A ship is said to be heeled when
she is inclined by an external force. For
example, when the ship is inclined by
the action of the waves or wind.
List. A ship is said to be listed when
she is inclined by forces within the
ship. For example, when the ship is
inclined by shifting a weight
transversely within the ship. This is a
7
fixed angle of heel.
Prepared by Dr. Sudhir Sindagi

CG and CB of the ship

If LCG≠ 𝐋𝐂𝐁, 𝐭𝐡𝐞𝐧 𝐭𝐡𝐞 𝐬𝐡𝐢𝐩 𝐰𝐢𝐥𝐥 𝐓𝐫𝐢𝐦


8
Prepared by Dr. Sudhir Sindagi

Estimation of the CG of the ship


Consider a ship as shown in the figure, consisting of three major
weights resulting in CG of the ship located at G.
w1, w2 and w3 are the weights placed at a distances of d1, d2 and d3
distances from the AP.
The total displacement of the ship = Δ = w1+w2+w3
Let, LCG = Longitudinal Center of Gravity from AP

9
Prepared by Dr. Sudhir Sindagi

Estimation of the CG of the ship


Taking moment about AP, we get
LCG *( w1+w2+w3)= w1*d1+w2*d2+w3*d3
w ∗d +w ∗d +w ∗d w ∗d +w ∗d +w ∗d
LCG= 1 1 2 2 3 3 = 1 1 2 2 3 3
w1+w2+w3 Δ
To generalize it,
𝐅𝐢𝐫𝐬𝐭 𝐌𝐨𝐦𝐞𝐧𝐭 𝐨𝐟 𝐰𝐞𝐢𝐠𝐡𝐭𝐬 𝐚𝐛𝐨𝐮𝐭 𝐀𝐏
LCG=
𝐓𝐨𝐭𝐚𝐥 𝐰𝐞𝐢𝐠𝐡𝐭 𝐨𝐟 𝐭𝐡𝐞 𝐬𝐡𝐢𝐩
If d1, d2 and d3 distances from the Keel
Taking moment about Keel, we get
VCG * (w1+w2+w3)= w1*d1+w2*d2+w3*d3
w1∗d1+w2∗d2+w3∗d3
VCG=KG=
Δ 10
Prepared by Dr. Sudhir Sindagi

Estimation of the CG of the ship


𝐅𝐢𝐫𝐬𝐭 𝐌𝐨𝐦𝐞𝐧𝐭 𝐨𝐟 𝐰𝐞𝐢𝐠𝐡𝐭𝐬 𝐚𝐛𝐨𝐮𝐭 𝐊𝐞𝐞𝐥
VCG=KG=
𝐓𝐨𝐭𝐚𝐥 𝐰𝐞𝐢𝐠𝐡𝐭 𝐨𝐟 𝐭𝐡𝐞 𝐬𝐡𝐢𝐩
If d1, d2 and d3 distances from the center of the ship, then
Taking moment about center line, we get
TCG *(w1+w2+w3)= w1*d1+w2*d2+w3*d3
w1∗d1+w2∗d2+w3∗d3
TCG=
Δ
𝐅𝐢𝐫𝐬𝐭 𝐌𝐨𝐦𝐞𝐧𝐭 𝐨𝐟 𝐰𝐞𝐢𝐠𝐡𝐭𝐬 𝐚𝐛𝐨𝐮𝐭 𝐂𝐋
TCG=
𝐓𝐨𝐭𝐚𝐥 𝐰𝐞𝐢𝐠𝐡𝐭 𝐨𝐟 𝐭𝐡𝐞 𝐬𝐡𝐢𝐩

11
Prepared by Dr. Sudhir Sindagi

Problems
A ship of 6000 tonnes displacement is composed of masses of 300,
1200 & 2000 tonnes at a distances of 60, 35 & 11m aft of amidships and
masses of 1000, 1000 & 500 tonnes at distances of 15, 30 & 50 m
forward of amidships. Calculate the LCG of the ship from midship.
Solution:
When number weight are provided with their locations, then it is
recommended to use the tabular method to solve such problems

12
Prepared by Dr. Sudhir Sindagi

Problems
Weight Distance from Midship Moment

ΣW=00 tonnes ΣM= 00 t-m

13
Prepared by Dr. Sudhir Sindagi

Problems
Weight Distance from Midship Moment
300 -60 -18000
1200 -35 -42000
2000 -11 -22000
1000 15 15000
1000 30 30000
500 50 25000
ΣW=6000 tonnes ΣM=-12000 t-m

𝐅𝐢𝐫𝐬𝐭 𝐌𝐨𝐦𝐞𝐧𝐭 𝐨𝐟 𝐰𝐞𝐢𝐠𝐡𝐭𝐬 𝐚𝐛𝐨𝐮𝐭 𝐌𝐢𝐝𝐬𝐡𝐢𝐩


LCG from midship=
𝐓𝐨𝐭𝐚𝐥 𝐰𝐞𝐢𝐠𝐡𝐭 𝐨𝐟 𝐭𝐡𝐞 𝐬𝐡𝐢𝐩
−𝟏𝟐𝟎𝟎𝟎
LCG from midship= =-2m from the midship. It means CG is142m aft
𝟔𝟎𝟎𝟎
of the midship.
Prepared by Dr. Sudhir Sindagi

Problems
A vessel of displacement 12500 tonnes has KG= 9.6m. On completion of
loading, she is required to have a KG of 9.5m. Cargo of 1000 & 850
tonnes are loaded at KG 5.5m & 13.6m respectively. Find the KG at
which to load further 1600 tonnes to produce the required final KG.
Given Data
Δold= 12500 tonnes
KGold= 9.6m
KGfinal=9.5

15
Prepared by Dr. Sudhir Sindagi

Problems
Distance from Distance from
Weight Moment Weight Moment
Keel Keel
12500 9.6 12500 9.6 120000
1000 5.5 1000 5.5 5500
850 13.6 850 13.6 11560
1600 x 1600 x 1600x
ΣW=000 ΣM=-000 t- ΣW=15950 ΣM=137060+
tonnes m tonnes 1600x

ΣM
Final KG= = 9.5m
ΣW
137060+1600x
9.5 =
𝟏𝟓𝟗𝟓𝟎
X=9.046m from the keel 16
Prepared by Dr. Sudhir Sindagi

Problems
A vessel displacing 6200 tonnes KG=8m. Distribute 9108 tonnes of
cargo between spaces KG 0.59m and 11.45m, so that the vessels
completes loading with KG 7.57m.
Given Data
Δold= 6200 tonnes
KGold= 8m
KGfinal=7.57m

17
Prepared by Dr. Sudhir Sindagi

Problems
Distance from Distance
Weight Moment Weight Moment
Keel from Keel
6200 8 6200 8 49600
w 0.59 w 0.59 0.59w
9108-w 11.45 9108-w 11.45 104286.6-11.45w
ΣW=000 ΣM=-000 t- ΣW=15308 ΣM=-153886.6-
tonnes m tonnes 10.86w

ΣM
Final KG= = 7.57m
ΣW
153886.6−10.86w
9.5 =
𝟏𝟓𝟑𝟎𝟖
W= 3499.54 tonnes and the other weight = 5608.46tonnes 18
Prepared by Dr. Sudhir Sindagi

Shift in CG of the ship


Shift in CG of the ship due to removal of existing weight:
Consider a rectangular plank as shown:
Now cut the length of plank of mass ‘w’ kg whose CG is ‘d’ meters away
from CG of the plank.
Note that a resultant moment of ‘w x d’ kg-m has been created in an
anti-clockwise direction about ‘G’.
The CG of the new plank shifts from ‘G’ to ‘G1’.

19
Prepared by Dr. Sudhir Sindagi

Shift in CG of the ship


Shift in CG of the ship due to removal of existing weight:
The new mass (W-w) kg now creates a tilting moment of (Δ-w) x GG1
about G. Since both are referring to the same moment,
(Δ-w) x GG1 = w x d
Shift in CG due to the removal of weight = GG1
wxd wxd
GG1 = =
Δ−w Final mass
w : the mass removed
d : the distance between the CG of the mass removed and the original
location of the CG of the body.
When a weight is removed from a body, the CG shifts directly away from
the CG of the mass removed. 20
Prepared by Dr. Sudhir Sindagi

Shift in CG of the ship


Shift in CG of the ship due to the addition or loading weight:
Equating the tilting moments created due to the added weight, which
must again be equal:
(W + w) x GG1 = w x d
wxd wxd
GG1 = =
Δ+w Final mass
d : the distance between the CG of the mass added and the original CG
of the body.
To generalize,
Shift in CG of the ship due to the addition or removal of weight=
wxd wxd
GG1 = =
Δ±w Final mass 21
When a weight is added, the CG shifts towards the CG of the mass.
Prepared by Dr. Sudhir Sindagi

Shift in CG of the ship


Shift in CG of the ship due to the addition or removal of weight
wxd wxd
GG1 = =
Δ±w Final mass

22
Prepared by Dr. Sudhir Sindagi

Shift in CG of the ship


Shift in CG of the ship due to the shifting of existing weight:
To calculate the height of the ship’s center of gravity after a vertical
weight shift, the following equation is used:
wxd wxd
GG1 = =
Δ Total weight
w = weight shifted
d = The distance by which the weight is
shifted
Shifting of weight, no matter where
onboard it is, will always cause the ship’s
center of gravity to move in the same
23
direction as the weight shifted.
Prepared by Dr. Sudhir Sindagi

Shift in CG of the ship


wxd wxd
GG1 = =
Δ Total weight
Shift in CG of the ship will be in the
transverse direction

24
Prepared by Dr. Sudhir Sindagi

Shift in CG of the ship


Effect of suspended weights
As we are aware that, the CG of a body is the point at which the force of
gravity may be considered to act vertically downwards.
For a suspended weight, whether the vessel is upright or inclined, the
point through which the force of gravity may be considered to act
vertically downwards is g, the POINT OF SUSPENSION.

25
Prepared by Dr. Sudhir Sindagi

Shift in CG of the ship


Important points:
The CG of a body will move directly TOWARDS the CG of any weight ADDED.
The CG of a body will move directly AWAY from the CG of any weight
DISCHARGED.
The CG of a body will move PARALLEL to the shift of the CG of any weight
MOVED within the body.
When a weight is SUSPENDED, CG of the weight is considered to be at the
POINT OF SUSPENSION.
wxd wxd
GG1 = =
Δ±w Final mass
d : the distance between the CG of the mass added and the original CG of the body.
wxd wxd
GG1 = =
Δ Total weight
26
w = weight shifted and d = The distance by which the weight is shifted
Prepared by Dr. Sudhir Sindagi

Problems
A heavy lift derrick is used to discharge a 100t package from a ship of
displacement 8000 tonnes with KG at 8.2m. If the KG of the weight while
on board is 3m and if the derrick head is 25m above the keel, find the
KG of ship
– While discharging
– After discharge
Given Data:

27
Prepared by Dr. Sudhir Sindagi

Problems
While Discharging:
To shift the weight, the moment
weight is lifted by the derrick, the
weight will act at the point of
suspension, which is derrick head.
Hence, it will be typical case of shift
in weight vertically upwards wherein,
d= 25-3= 22m
Shift in CG of the ship, due to vertical
shift in weight
wxd wxd
GG1 = =
Δ Total weight 28
Prepared by Dr. Sudhir Sindagi

Problems
While Discharging:
GG1 =0.275m upwards
New KG =Old KG + GG1 = 8.475m
After Discharging:
This can be solved by two ways
– A. Considering the weight is removed from the derrick head
– B. Considering the weight is removed from the deck at Kg= 3m
A. Considering the weight is removed from the derrick head
wxd
GG1 =
Δ− w
d : the distance between the CG of the mass removed and the original
CG of the body 29

d = 25-8.475 = 16.525m
Prepared by Dr. Sudhir Sindagi

Problems
After Discharging:
wxd
GG1 = = 0.2091m downwards
Δ− w
New KG =Old KG - GG1 = 8.475 – 0.2091 = 8.265m
B. Considering the weight is removed from the deck at Kg= 3m
Here, the CG of the ship will be at 8.2 and the Kg of the weight is at 3m
d : the distance between the CG of the mass removed and the original
CG of the body
d = 8.2-3 = 5.2m
wxd
GG1 = = 0.065m upwards
Δ− w
New KG =Old KG + GG1 = 8.2 + 0.065 = 8.265m 30
Prepared by Dr. Sudhir Sindagi

Problems
A vessel of 8000 tonnes displacement has 75 tonnes of cargo on the
deck. It is lifted by a derrick whose head is 10.5m above the CG of the
cargo and be placed in the lower hold 9m below the deck and 14m
forward of its original position. Calculate the shift of vessel’s CG from
its original position when the cargo is
– Just clear of the deck
– In its final position
Given Data

31
Prepared by Dr. Sudhir Sindagi

Problems
Just Clear of the deck:
To shift the weight, the moment weight is lifted by the derrick i.e. when
the weight is Just Clear of the deck, the weight will act at the point of
suspension, which is derrick head.
Hence, it will be typical case of shift in weight vertically upwards
wherein,
d= 10.5m
Shift in CG of the ship
wxd
GG1 =
Δ
GG1 = 0.098m upwards
32
Prepared by Dr. Sudhir Sindagi

Problems
In its final position:
Here the weight is not only moved downwards, it also being moved
forward
For the downward movement
This can be solved by two ways
– A. Considering the weight is shifted from the derrick head
– B. Considering the weight is shifted from the main deck
A. Considering the weight is shifted from the derrick head
In this case, d= 19.5m
wxd
G1G 2 = =0.1828m downwards
Δ
33
Prepared by Dr. Sudhir Sindagi

Problems
In its final position:
B. Considering the weight is shifted from the main deck
In this case, d= 9m
wxd
GG2 = =0.084m downwards
Δ
This can also be obtained by subtracting the shift in the CG of the ship
as follows
GG2 = G1G2 - GG1
GG2 = 0.1828-0.098
GG2 = 0.1828-0.098
GG2 = 0.084m downwards
34
Prepared by Dr. Sudhir Sindagi

Problems
In its final position:
For the forward movement
In this case, d= 14m
wxd
G2G 3 =
Δ
G2G3 = 0.131m forward

35
Prepared by Dr. Sudhir Sindagi

Problems
A ship of 4000 tonne displacement has its centre of gravity 1.5m aft of
midships and 4m above keel. 200 tonne of cargo are now added 45m
forward of midships and 12m above the keel. Calculate the new position
of the centre of gravity and the angle in which the centre of gravity
moves relative to the horizontal
Given Data

36
Prepared by Dr. Sudhir Sindagi

Problems
It is the typical case of weight addition, wherein, weight added =200
tonnes
wxd
GG1 =
Δ+ w
d : the distance between the CG of the mass addedd and the original
CG of the body
For the horizontal movement of CG of the ship
d = 45+1.5 = 46.5m
GG1 =2.21m forward
For the vertical movement of CG of the ship
d = 12-4= 8m
G1G2 =0.3809m upwards 37
Prepared by Dr. Sudhir Sindagi

Problems
For the new position of the ship
KG2 = KG+G1G2
KG2 =4.3809m above the keel
LCG=2.21-1.5 =0.71m forward of midship
For the angle of in which the centre of gravity moves relative to the
horizontal
G1G2
tan θ =
GG1
Ɵ= 9.76 degrees

38
Prepared by Dr. Sudhir Sindagi

Angle of List
The angle of list is the degree to which a vessel heels (leans or tilts) to
either port or starboard at equilibrium—with no external forces acting
upon it.
Listing is caused by the off-centerline distribution of weight onboard
due to uneven loading or due to flooding.
By contrast, roll is the dynamic movement from side to side caused by
waves

39
Prepared by Dr. Sudhir Sindagi

Angle of List
Let an existing weight on board a ship is shifted to starboard side
transversely as shown in the figure.
This causes a shift in the CG of the ship from G to G1

40
Prepared by Dr. Sudhir Sindagi

Angle of List
In this new position, if one considers no shift in the CB of the ship, then
buoyancy force and gravity force acting at CB and CG of the ship
respectively would create a clockwise moment and the ship will not be
in equilibrium.
To achieve the equilibrium condition with zero moment acting on the
ship, slowly CB of the ship will start moving towards starboard side in
such a way that, it will set itself exactly below the CG of the ship.
This is typically a equilibrium condition and the ship will remain in that
state unless any weight is being shifted/removed/added.
This condition/state of the ship is known as the listed condition.
In the listed condition, the new line of action of buoyancy will intersect
the center line of the ship at M, which is known as the Transverse 41
Metacentre of the ship.
Prepared by Dr. Sudhir Sindagi

Angle of List
Transverse Metacentre is the point about which the ship
heels/lists/rolls.
The vertical distance between the original CG of the ship and the
metacentre is known as the metacentric height (GM).
The angle of list can found by
GG1 TCG
𝐭𝐚𝐧 𝛟 = =
GM GM
Here, GG1 is known as the TCG of the ship, which is transverse distance
of the CG of the ship from the centreline and can be found using
wxd
TCG= GG1= if the existing weight is being shifted
Δ
wxd
TCG= GG1= if the weight is added/removed 42
Δ±𝐰
Prepared by Dr. Sudhir Sindagi

Problems
A vessel of 6500 tonnes displacement has a KM 7.2m and KG 6.8m. A
weight of 100 tonnes is shifted 2.3m to starboard and 3.9m upwards. If
the vessel is initially upright, calculate the resulting list.
Given data:
Δ= 6500 tonnes
w=100 tonnes
dh= 2.3m
dv= 3.9m
Here, since there are two movements of the weight, one in transverse
direction and the other being vertically upwards, hence accordingly, the
CG of the ship will also move in the similar direction.
43
Prepared by Dr. Sudhir Sindagi

Problems
In the first case, dh= 2.3m
w x dh
GGH =
Δ
GGH = 0.03538m
In the second case, dv= 3.9m
w x dv
G HG V =
Δ
GHGV = 0.06m vertically upwards
The angle of list can found by
GGH TCG
𝐭𝐚𝐧 𝛟 = =
New GM Old GM−GHGV
Φ= 5.9407 degrees
44
When there is a vertical shift in CG of ship, GM value changes.
Prepared by Dr. Sudhir Sindagi

Problems
A ship 8000 Tonnes displacement has KM =8.7 meters KG= 7.6 meters.
The following weights are either loaded or discharged as mentioned.
– Load 250 T cargo, VCG 6.1 m, TCG 7.6 m to Stbd side
– Load 300 T Fuel Oil, VCG 0.60 m, TCG 6.1 m to Port side
– Discharge 50 T of Ballast, VCG 1.2 m, TCG 4.6 m to Port side.
Find the Final List of the ship after completion of these activities.
Here, it is recommended to use tabular
method

45
Prepared by Dr. Sudhir Sindagi

Problems
Weight KG Moment Weight TCG Moment
8000 7.6 8000
250 6.1 250
300 0.6 300
-50 1.2 -50
ΣW=000 ΣM=-000 t- ΣW=000 ΣM=-000 t-
tonnes m tonnes m

46
Prepared by Dr. Sudhir Sindagi

Problems
Weight KG Moment Weight TCG Moment
8000 7.6 60800 8000 0 0
250 6.1 1525 250 7.6 1900
300 0.6 180 300 -6.1 -1830
-50 1.2 -60 -50 -4.6 +230
ΣW=8500 ΣM=- ΣW=8500 ΣM=300 t-
tonnes 62445 t-m tonnes m

ΣM 62445
Final KG= =
ΣW 8500
Final KG=7.346m

47
Prepared by Dr. Sudhir Sindagi

Problems
ΣM 300
Final TCG= =
ΣW 8500
Final TCG=0.035m on starboard side.
The angle of list can found by
GGH TCG
𝐭𝐚𝐧 𝛟 = =
New GM 𝐊𝐌 −𝐧𝐞𝐰 𝐊𝐆
0.035
𝐭𝐚𝐧 𝛟 =
𝟖.𝟕−𝟕.𝟑𝟒𝟔
Φ = 1.49 degrees on starboard side.

48
Prepared by Dr. Sudhir Sindagi

Problems
A ship of 12500 tonne displacement and 15m beam has a metacentric
height of 1.10m. A mass of 80tonne is lifted from its position in the
centre of lower hold by one of ship’s derricks and placed on the quay
2m away from the ship’s side. The ship heels to a maximum angle of 3.5
deg when the mass is being moved.
– a) Does the GM alter during operation?
– b) Calculate the height of the derrick head above the original centre of gravity of
the mass

49
Prepared by Dr. Sudhir Sindagi

Problems

50
Prepared by Dr. Sudhir Sindagi

Problems
In this case, as shown in the second figure, the ship will list with angle
3.5 degree, wherein, using derrick, the weight will be shifted in both
horizontal and vertical distances causing both the horizontal and
vertical shifts in the CG of the ship, respectively.
Since there is vertical shift in the CG of the ship, hence GM of the ship
will alter.
GGH due to dh and GHGV due to dV
Here, dh is known but dV is unknown.
w x dh
GGH =
Δ
here dh = 9.5m
GGH = 0.0608m 51
Prepared by Dr. Sudhir Sindagi

Problems
GGH GGH
𝐭𝐚𝐧 𝛟 = =
New GM Old GM−GHGV
Putting the values, one can obtain the value of GHGV
0.0608
𝐭𝐚𝐧 𝟑. 𝟓 =
1.1−GHGV
GHGV =0.10593m
Putting it in the below equation, we can calculate the value of dv
w x dv
G HG V =
Δ
dV = 16.5515m

52
Prepared by Dr. Sudhir Sindagi

Problems
A ship of 13750 tonnes displacement, GM = 0.75 m, is listed 2.5 degrees
to starboard and has yet to load 250 tonnes of cargo. There is space
available in each side of No. 3 between deck (centre of gravity, 6.1 m out
from the centreline). Find how much cargo to load on each side if the
ship is to be upright on completion of loading. Assume, no vertical shift
in CG of ship due to addition of weight.

53
Prepared by Dr. Sudhir Sindagi

Problems
Since there is no vertical shift in CG of the ship, hence GM remains
same.
Using below equation, one can find the value of GGH
GGH
𝐭𝐚𝐧 𝛟 =
New GM
GGH = 0.032m
Since, it is the case of weight addition, hence
w1 x d𝟏−w2 x d2
GGH =
Δ+𝐰𝟏+𝐰𝟐
w x 6.1−(250−w1)∗ 6.1
0.032 = 1
13750+250
Solving this equation, we get 54
w1 =88.04 tonnes and w2 =161.97tonnes
Prepared by Dr. Sudhir Sindagi

Summary
Introduction
CG and CB of the ship: List and Trim
Estimation of the CG of the ship
Shift in CG of the ship
– Due to shifting in existing weight
– Addition of weight
– Removal of existing of weight
– Effect of suspended weight
Angle of List

55
Prepared by Dr. Sudhir Sindagi

Transverse stability of the ship

By
Dr. Sudhir Sindagi 1
Prepared by Dr. Sudhir Sindagi

Chapter Content
Introduction – Types of Equilibrium
Conditions for the stability of the ship and submarine
Stiff and Tender Ship
Stability for the small angles of heel
– Derivation for the Metacentric radius
– Free Surface Effect
– Inclining Experiment
Stability at the large angles of heel
– Derivation for the wall sided formula
– Angle of Loll
– GZ Curve, Cross curves of Stability (KN Curves)
– Dynamical Stability of the ship 2
– Requirements of the IMO for the GZ Curve
Prepared by Dr. Sudhir Sindagi

Introduction
The body is said to be in equilibrium when the summation of forces and
moments acting on it is zero.
Normally two forces act on a floating body, one is gravitational force
and the other one is the buoyancy force.
The gravitational force acts through the center of gravity. Similarly the
buoyancy force also acts through the center of buoyancy.
The position of the center of gravity depends on the distribution and
magnitude of the weights.
But the center of buoyancy depends only on the shape of the immersed
portion of the vessel.
It is the centroid of the under water portion of the floating body.
3
Prepared by Dr. Sudhir Sindagi

Introduction
The conditions of equilibrium for floating bodies are:
The buoyancy force is equal to the gravitational force.
The center of buoyancy and the center of gravity lie in the same vertical
line.
The common line of action of these forces will be perpendicular to the
new water line.

4
Prepared by Dr. Sudhir Sindagi

Introduction
MT

Wind

Water
Resistance

When a ship is affected by outside forces (Upsetting Forces), it will


alter its state of equilibrium. The forces of wind- and the opposing
force of the water below the waterline- will cause an external5
moment couple about the ship’s center of flotation.
Prepared by Dr. Sudhir Sindagi

Introduction
The ship reacts to this
external moment MT
couple by pivoting Wind
about F, causing a
shift in the center of
buoyancy. The center
of buoyancy will shift Water
B Resistance
because the submerged
volume shifts

Note that there is no change in weight or it’s distribution so there is


6
NO change in the location of G!
Prepared by Dr. Sudhir Sindagi

Introduction
Because the location of
B changes, the location
MT
of where the FB is
applied also changes. s
Because G does not
move, the location of
the Δs force does not F
B
change.
FB
The weight force and the buoyancy
are no longer aligned. The heeling As a result, the ship is now
over causes the creation of an back into equilibrium, even
internal moment couple known as as it heels over due to the
7
the Righting Moment (RM) wind force.
Prepared by Dr. Sudhir Sindagi

Introduction
Upsetting forces of stability:
Beam winds, with or without of rolling,
Lifting of heavy weights over the side
Free swinging of weights,
High speed of turns,
Grounding and docking,
Strain on mooring lines,
Towline pull of tugs,
Entrapped water on deck.

8
Prepared by Dr. Sudhir Sindagi

Types of Equilibrium
Stable equilibrium
A ship is said to be in stable equilibrium if, when
inclined, she tends to return to the initial position.
For this to occur the centre of gravity must be
below the metacentre, that is, the ship must have
positive metacentric height (M above G & GM>0)
If moments are taken about G there is a moment
to return the ship to the upright (GM>0).
This moment is referred to as the Moment of
Statical Stability or Righting Moment and is equal
to the product of the force 'Δ' and the lever GZ.
Moment of Statical Stability = Δ x GZ (tonnes- 9
metres).
Prepared by Dr. Sudhir Sindagi

Types of Equilibrium

10
Prepared by Dr. Sudhir Sindagi

Types of Equilibrium
Stable equilibrium
The lever GZ is referred to as the righting lever
and is the perpendicular distance between the
centre of gravity and the vertical through the new
centre of buoyancy.
At a small angle of heel (less than 100)
GZ = GM sin ϕ
Moment of Statical Stability =
RM= Δ x GM sin ϕ

11
Prepared by Dr. Sudhir Sindagi

Types of Equilibrium
Neutral Equilibrium
When G coincides with M i.e. GM=0 as shown in
Figure, the ship is said to be in neutral
equilibrium, and if inclined to a small angle she
will tend to remain at that angle of heel until
another external force is applied.
The ship has zero GM. Note that KG = KM.
Moment of Statical Stability = Δ x GZ, but in this
case GZ = 0;
Moment of Statical Stability = 0.
Therefore there is no moment to bring the ship
back to the upright or to heel her over still further 12
Prepared by Dr. Sudhir Sindagi

Types of Equilibrium

13
Prepared by Dr. Sudhir Sindagi

Types of Equilibrium
Unstable Equilibrium
When a ship which is inclined to a small
angle tends to heel over still further,
she is said to be in unstable
equilibrium.
For this to occur the ship must have a
negative GM. G is above M.
The moment of statical stability=Δ*GZ,
is clearly a capsizing moment which will
tend to heel the ship still further.

14
Prepared by Dr. Sudhir Sindagi

Types of Equilibrium

15
Prepared by Dr. Sudhir Sindagi

Types of Equilibrium
When a ship in unstable or neutral equilibrium, which can be made
stable by lowering the effective centre of gravity of the ship.
To do this one or more of the following methods may be employed:
– Weights already in the ship may be lowered,
– Weights may be loaded below the centre of gravity of the ship,
– Weights may be discharged from positions above the centre of gravity,
– Free surfaces within the ship may be removed

16
Prepared by Dr. Sudhir Sindagi

Stability of Submarine
For the submarine which is fully submerged in water, as shape of the
submerged volume doesn't alter, the position of center buoyancy
remains unaltered and it remains on the center line of the ship.
Hence, it is interesting to know the condition to generate the righting
moment, in turn, the submarine to remain stable.
The condition for the stability of the submarine is G lies below B.

Unstable Stable
submarine submarine

17
Prepared by Dr. Sudhir Sindagi

Metacentric Height

18
Prepared by Dr. Sudhir Sindagi

Metacentric Height

19
Prepared by Dr. Sudhir Sindagi

Metacentric Height
The centre of gravity of a body `G' is the point through which the force
of gravity is considered to act vertically downwards with a force equal
to the weight of the body. KG is VCG of the ship.
The centre of buoyancy `B' is the point through which the force of
buoyancy is considered to act vertically upwards with a force equal to
the weight of water displaced. It is the centroid of the underwater
volume. KB is VCB of the ship.
To float at rest in still water, a vessel must displace her own weight of
water, and the centre of gravity must be in the same vertical line as the
centre of buoyancy.
KM = KB + BM Also KM = KG + GM
GM= KB+ BM – KG 20
BM= Metacentric Radius and GM = Metacentric Height
Prepared by Dr. Sudhir Sindagi

Metacentric radius
The metacentric radius of a ship is the vertical distance between its
center of buoyancy and metacenter.

21
Prepared by Dr. Sudhir Sindagi

Metacentric Height
GM is crucial to ship stability.
The table below shows typical working values for GM for several ship-
types all at fully-loaded drafts.

Ship type GM at fully-loaded condition


General cargo ships 0.30–0.50m
Oil tankers 0.50–2.00m
Double-hull supertankers 2.00–5.00m
Container ships 1.50–2.50m
Ro-Ro vessels 1.50 m approximately
Bulk ore carriers 2–3m
22
Prepared by Dr. Sudhir Sindagi

Stiff Ship
The Rolling time period of a ship is the time taken by the ship to roll
from one side to the other and back again to the initial position.
When a ship has a comparatively large GM, for example 2 m to 3 m, the
righting moments at small angles of heel will also be comparatively
large.
It will thus require larger moments to incline the ship. When inclined
she will tend to return more quickly to the initial position.
The result is that the ship will have a comparatively short time period,
and will roll quickly – and perhaps violently – from side to side.
A ship in this condition is said to be ‘stiff’, and such a condition is not
desirable.
The time period could be as low as 8 seconds. The effective centre 23 of
gravity of the ship should be raised within that ship.
Prepared by Dr. Sudhir Sindagi

Tender Ship
When the GM is comparatively small, for example 0.16 m to 0.20 m the
righting moments at small angles of heel will also be small.
The ship will thus be much easier to incline and will not tend to return
so quickly to the initial position.
The time period will be comparatively long and a ship, for example 25 to
35 seconds, in this condition is said to be ‘tender’.
As before, this condition is not desirable and steps should be taken to
increase the GM by lowering the effective centre of gravity of the ship.
The officer responsible for loading a ship should aim at a happy
medium between these two conditions whereby the ship is neither too
stiff nor too tender.
A time period of 15 to 25 seconds would generally be acceptable 24 for
those on board a ship at sea.
Prepared by Dr. Sudhir Sindagi

Derivation for BM
The metacentric radius of a ship is the vertical distance between its
center of buoyancy and metacenter.
This parameter can be visualized as the length of the string of a
swinging pendulum of the center of gravity of the pendulum coincides
the center of buoyancy of the ship.
In other words, the ship behaves as a pendulum swinging about its
metacenter.
It is a different fact that, the metacenter of the ship changes itself, every
moment. Because with every angle of heel, the transverse shift in center
of buoyancy will vary, therefore creating a new metacenter.
For small angle of heel, this vertical shift in M is neglected.
The importance of this parameter can be realised when 25 the
mathematical expression of metacentric radius is investigated.
Prepared by Dr. Sudhir Sindagi

Derivation for BM

26
Prepared by Dr. Sudhir Sindagi

Derivation for BM
Consider a ship as shown in the figure, is initially floating upright at
waterline WL.
Due to the external force or the heeling moment by wind or wave, let the
ship heels by an angle ϕ and floats at new waterline W1L1.
In this case, a triangular wedge on port side will come out of the water,
known as emerging wedge and a wedge with similar volume on
starboard side immerses into the water, known as immerses wedge.
This is simply a case of shifting of volume from port to the starboard,
causing a shift in the center of buoyancy of the ship.
Consider a small elementary length of the wedge dx along the length of
the ship.
Volume of the elementary wedge (v) = area *length of the wedge 27
Prepared by Dr. Sudhir Sindagi

Derivation for BM
𝟏
Volume of the elementary wedge (v) = 𝐲 ∗ 𝐲 tan φ 𝐝𝐱
𝟐
𝟏 𝟐
Volume of the elementary wedge (v) = 𝐲 tan φ 𝐝𝐱
𝟐
Here, the distance by which the volume of wedge is shifted = distance
by which the centroid is shifted from g to g1
𝟐 𝟐 𝟒
d= 𝐲+ 𝐲 = 𝐲
𝟑 𝟑 𝟑
As mentioned earlier, this is case of the shift in volume, hence there is a
shift in the center buoyancy of the ship.
Small shift in the center buoyancy of the ship due to the shift in Volume
of the elementary wedge =
𝐕𝐨𝐥𝐮𝐦𝐞 𝐨𝐟 𝐰𝐞𝐝𝐠𝐞 ∗ 𝐝𝐢𝐬𝐭𝐚𝐧𝐜𝐞 𝐛𝐲 𝐰𝐡𝐢𝐜𝐡 𝐭𝐡𝐞𝐢𝐫 𝐜𝐞𝐧𝐭𝐫𝐨𝐢𝐝 𝐬𝐡𝐢𝐟𝐭𝐬 𝐯∗𝐝
= 28
𝐯𝐨𝐥𝐦𝐞𝐭𝐫𝐢𝐜 𝐝𝐢𝐬𝐩𝐥𝐚𝐜𝐞𝐦𝐞𝐧𝐭 𝐨𝐟 𝐭𝐡𝐞 𝐬𝐡𝐢𝐩 𝛁
Prepared by Dr. Sudhir Sindagi

Derivation for BM
𝟏 𝟐 𝟒
𝐲 tan φ 𝐝𝐱∗ 𝐲
Small shift in the center buoyancy of the ship = 𝟐 𝟑
𝛁
𝟐 𝟑
𝐲 𝐝𝐱
Small shift in the center buoyancy of the ship = 𝟑
∗ tan φ
𝛁
Total shift in the center of buoyancy of the ship = BB1 which will be
obtained by integrating above equation for the entire length
𝟐 𝐋 𝟑 tan φ
BB1 = ‫𝐱𝐝 𝐲 𝟎׬‬ ∗
𝟑 𝛁
𝟐 𝐋 𝟑
Here, the term ‫𝐱𝐝 𝐲 ׬‬ = IT = Transverse Moment of inertia of the
𝟑 𝟎
ship’s waterplane area taken at SLL about the longitudinal axis passing
through the centerline.
29
Prepared by Dr. Sudhir Sindagi

Derivation for BM
tan φ
BB1 = IT
𝛁
From the triangle MBB1, we can write
BB1
tan φ =
BM
Putting this in above equation and after simplification, we get
IT
BM =
𝛁

30
Prepared by Dr. Sudhir Sindagi

Derivation for BM
Assumptions in the derivation:
The angle of heel is assumed to be lesser than equal to 7 or 10 degrees.
Since, the angle of heel is assumed to be smaller, hence the vertical
shift in the center of buoyancy of the ship due to the vertical shift in the
centroid of the wedge is neglected.
BB1 is perpendicular to the center line of the ship.
The side walls (shells) of the ship are assumed to be vertical.

31
Prepared by Dr. Sudhir Sindagi

Derivation for BM
Derive the expression for the Transverse Metacentric Radius (BMT)and
Longitudinal Metacentric Radius (BML) of the rectangular barge floating
at the draft T.

32
Prepared by Dr. Sudhir Sindagi

Derivation for BM
IT = Transverse Moment of inertia of the ship’s waterplane area taken at
SLL about the longitudinal axis passing through the centerline.
𝐋𝐁 𝟑
IT =
𝟏𝟐
IL = Longitudinal Moment of inertia of the ship’s waterplane area taken at
SLL about the Transverse axis passing through the centroid of the
waterplane.
𝐁𝐋𝟑
IL =
𝟏𝟐
Volumetric displacement = 𝛁 = 𝐋𝐁𝐓
𝐋𝐁 𝟑
IT
Transverse Metacentric Radius = BMT = = 𝟏𝟐
𝛁 𝐋𝐁𝐓
33
Prepared by Dr. Sudhir Sindagi

Derivation for BM
𝐁𝟐
Transverse Metacentric Radius BMT =
𝟏𝟐𝐓
𝐁𝐋𝟑
IL
Longitudinal Metacentric Radius = BML = = 𝟏𝟐
𝛁 𝐋𝐁𝐓
𝐋𝟐
Longitudinal Metacentric Radius = BML =
𝟏𝟐𝐓

34
Prepared by Dr. Sudhir Sindagi

Problems
The second moment of area of the water plane about the centre line is
20000 m4. The displacement of the ship is 7000 tonnes, while floating in
a water of density 1.008 tonnes/m3. KB=1.9m. KG=3.2m. Calculate the
initial metacentric height of the ship.
Given Data
Δ=7000 tonne, GM=?, KB=1.9m, IT= 20000 m4. KG=3.2m
We know that,
GM=KB + BM – KG
IT
But BM =
𝛁
Using equation ∆= 𝛁 ∗ 𝛒, 𝐜𝐚𝐥𝐜𝐮𝐥𝐚𝐭𝐞 𝛁 = 6944.44m3
Putting in the equation of BM, we get BM= 2.88m 35
GM=1.58m
Prepared by Dr. Sudhir Sindagi

Problems
A ship of 12000 tonne displacement has a metacentric height of 0.6m
and a centre of buoyancy of 4.5 m above the keel. The second moment
of area of the water plane about the centre line is 42.5x1000 m4.
Calculate the height of centre of gravity above the keel.
Given Data
Δ=12000 tonne, GM=0.6m, KB=4.5m, IT= 42.5x1000 m4. KG=?
We know that,
GM=KB + BM – KG
IT
But BM =
𝛁
Using equation ∆= 𝛁 ∗ 𝛒, 𝐜𝐚𝐥𝐜𝐮𝐥𝐚𝐭𝐞 𝛁 = 11707.317m3
Putting in the equation of BM, we get BM= 3.63m 36
KG=7.53m
Prepared by Dr. Sudhir Sindagi

Problems
A rectangular barge 100mX16mX6m depth is floating in SW at draft of
4m. Calculate its KB, BM & GM of the vessel if it’s KG=5.9m. Also
comment on KG of ship at which the ship will become unstable.
Given Data
T= 4m, KG=5.9m
KB=?, BM=? GM= ?& KG=?, when ship becomes unstable
∆= 𝟏𝟎𝟎 ∗ 𝟏𝟔 ∗ 𝟒 ∗ 𝟏. 𝟎𝟐𝟓= 6560 tonnes.
T
KB = =2m
𝟐
IT 𝐁𝟐
BMT = =
𝛁 𝟏𝟐𝐓
BMT = 5.33m 37
Prepared by Dr. Sudhir Sindagi

Problems
KM= KB+BM = 7.33m
GM= KB+BM-KG = KM-KG
GM=1.433m
Ship becomes unstable when GM becomes negative and is possible
when KG>KM
KG>7.33m

38
Prepared by Dr. Sudhir Sindagi

Problems
A vessel of constant triangular cross section has a depth of 12m and
breadth at the main deck of 15m. Calculate the draft at which the vessel
becomes unstable if the KG is equal to 6.675m
Given Data
D=12m, B=15m, KG= 6.675m, T=? When vessel becomes unstable.
Vessel become unstable when GM<0,
GM=KB + BM – KG <0
KB + BM < KG
In this case,
KB + BM < 6.675m

39
Prepared by Dr. Sudhir Sindagi

Problems
For 𝐈𝐓 , take the water plane at the SLL and obtain its second moment of
area of the waterplane about longitudinal axis.
Assuming length of the prism =L
𝐋𝐛𝟑
𝐈𝐓 =
𝟏𝟐
𝛁= Area of the section*Length of the section
𝟏
𝛁= 𝐛 𝐓*L
𝟐

40
Prepared by Dr. Sudhir Sindagi

Problems
IT
But BM =
𝛁
𝐋𝐛𝟑
BM = 𝟏 𝟏𝟐

𝟐
𝐛 𝐓∗L

Simplifying above equation, we get


BM=0.2604T
KB= 2T/3
Putting all these values in the equation,
KB + BM < 6.675m
2T/3 + 0.2604T < 6.675m
T< 7.2m. 41
Ship becomes unstable when the draft < 7.2m
Prepared by Dr. Sudhir Sindagi

Problems
A Conical buoy has a displacement of 0.73 tonnes when floating with its
vertex down in sea water. Vertex angle is 60 degrees and second
𝛑𝐃𝟒
moment of inertia of the circular waterplane is .
The CG of the ship is
𝟔𝟒
1.22 from the vertex. Find the metacentric height and the depth of water
from the vertex.
Given Data
𝛑𝐃𝟒
Δ = 0.73 tonnes, 𝐈𝐓 = ,
𝟔𝟒
KG= 1.22m
GM= ? T=?

42
Prepared by Dr. Sudhir Sindagi

Problems
Using equation ∆= 𝛁 ∗ 𝛒,
𝟏
0.73 = 𝛑𝐑𝟐 𝐓 ∗ 𝛒
𝟑
From the geometry, we get relationship between R & T
𝐑
tan 𝟑𝟎 =
𝐓
Putting R= tan 𝟑𝟎 ∗ 𝐓 in above equation, we get
𝟏
0.73 = 𝛑(tan 𝟑𝟎 ∗ 𝐓)𝟐 𝐓 ∗ 𝟏. 𝟎𝟐𝟓
𝟑
Solving this equation for the T, we get
T= 1.27m
D= 2R=2 * tan 𝟑𝟎 ∗ 𝐓
D= 1.4664m 43
Prepared by Dr. Sudhir Sindagi

Problems
Metacentric height is calculated using
GM= KB + BM – KG
Here, KB= 2T/3
IT 𝛑𝐃𝟒
But BM = , 𝐈𝐓 = , 𝛁 = Δ/ ρ = 0.73/1.025
𝛁 𝟔𝟒
Putting all these values in the above equation, we get
IT
GM= 2T/3 + – 1.22
𝛁
𝛑𝐃𝟒
GM= 2T/3 + 𝟔𝟒
– 1.22
𝟎.𝟕𝟑/ρ
Putting values of T and D, We get, GM
GM= -0.0583m and the buoy is unstable. 44
Prepared by Dr. Sudhir Sindagi

Problems
A pontoon raft 10m long is formed by two cylinders 0.75m radius
spaced 2m apart between centers and is planked over wood forming a
platform 10m x 3m. When laden, the raft floats with cylinders half
immersed in river water and its centre of gravity when laden is 1m
above the waterline. Calculate the transverse and longitudinal
metacentric heights.
Given Data
L=10m, D=0.75m, T=D/2

45
Prepared by Dr. Sudhir Sindagi

Problems
KG= 1m above the waterline
KG=1+0.75=1.75m
𝟒𝐑
Distance of center of buoyancy from the center for semicircle=
𝟑𝛑
𝟒𝐑
KB= R -
𝟑𝛑
KB= 0.4316m
IT
BM =
𝛁
𝛁=volume of two half cylinders
𝛁= 𝛑𝐑𝟐 𝐋
𝛁= 17.6714 m3
46
Prepared by Dr. Sudhir Sindagi

Problems
IT = MI of outer rectangle – MI of inner rectangle
𝟏𝟎∗𝟑.𝟓𝟑 𝟏𝟎∗𝟎.𝟓𝟑
IT = −
𝟏𝟐 𝟏𝟐
IT =35.625 m4
IT
BM =
𝛁
BM = 2.0159m
GM=KB + BM – KG
GM=0.6975m

47
Prepared by Dr. Sudhir Sindagi

Free Surface Effect


When any tank is not fully filled and the ship heels, the liquid in the tank
moves across the tank in the direction of heel, causing CG of the ship to
move away from the centerline reducing the righting lever, GM and in
turn, the stability of the ship reduces. This effect is known as the free
surface effect.
The free surface effect can become a problem in tankers, wherein, cargo
holds are partially full, fuel tanks, or water tanks (especially if they span
the full breadth of the ship), or from accidental flooding.
If a compartment or tank is either empty or full, there is no change in the
craft's center of mass as it rolls from side to side. However, if the
compartment is only partially full (Filled less than 90% or filled more
than 10%), the liquid in the compartment will respond to the vessel's
heave, pitch, roll, surge, sway or yaw. 48
Prepared by Dr. Sudhir Sindagi

Free Surface Effect

49
Prepared by Dr. Sudhir Sindagi

Free Surface Effect


Let there be a tank which contains a liquid of density ρl and iT is the
transverse moment of inertia of the tank about its longitudinal axis.
Due to shift of the liquid in the tank, a horizontal shift in the CG from G
to G1 occurs. Here it is assumed that, for a small angle of heel up to 70,
the vertical shift in the CG of the ship is neglected.
The line of action of gravity force intersects the centerline of the ship at
GV, resulting in a vertical shift in the CG from G to GV.
As it has been observed in the past, FSE causes a vertical shift in the
CG of the ship reducing stability, hence, it was concluded that, this
vertical shift from G to GV is the FSE
GGV= FSE= FSC = Free Surface Correction
GVM= Fluid GM which is considering FSE 50

GM= Solid GM without considering FSE


Prepared by Dr. Sudhir Sindagi

Free Surface Effect


GVM= Fluid GM = Solid GM - FSE
GM= Solid GM without considering FSE
If v is the volume of the liquid inside the tank shifted
w x d (vρl) d ρl(vd)
GG1 = = =
Δ Δ Δ
𝟏 𝟐 𝟒
(vd) = 𝐲 tan φ 𝐝𝐱 𝐲
𝟐 𝟑
For the entire length, it will be
𝟐 𝐋 𝟑
(vd) = ( ‫) 𝐱𝐝 𝐲 𝟎׬‬tan 𝝋
𝟑
𝟐 𝐋
Here, iT = ( ‫)𝐱𝐝 𝟑 𝐲 𝟎׬‬
𝟑
ρl(vd) ρl(iT tan 𝝋) 51
GG1 = =
Δ Δ
Prepared by Dr. Sudhir Sindagi

Free Surface Effect


𝐆𝐆𝟏
But tan φ =
𝐆𝐆𝐕
ρl(iT𝐆𝐆
𝐆𝐆
)
𝟏

GG1= 𝐕
Δ
iT∗ρl
FSE=FSC=GGV=
Δ
FSE does not depend upon
– Weight of the liquid inside the tank
– Location of the tank

52
Prepared by Dr. Sudhir Sindagi

Free Surface Effect


Reducing FSE by inserting transverse bulkhead at the center of length
Since density and displacement of the ship remains same for the
particular loading, FSE depends on iT of the tank
Here, iT = iT1+ iT2
(𝐋/𝟐)𝐛𝟑 (𝐋/𝟐)𝐛𝟑
New iT = +
𝟏𝟐 𝟏𝟐
𝐋𝐛𝟑
New iT = = Old iT
𝟏𝟐
No change in the iT is observed.
Hence no change in FSE with
this modified configuration
53
Prepared by Dr. Sudhir Sindagi

Free Surface Effect


Reducing FSE by inserting longitudinal bulkhead at the center of
Breadth
Since density and displacement of the ship remains same for the
particular loading, FSE depends on iT of the tank
Here, iT = iT1+ iT2
𝐋(𝐛/𝟐)𝟑 𝐋(𝐛/𝟐)𝟑
New iT = +
𝟏𝟐 𝟏𝟐
𝐋𝐛𝟑 𝟏 𝐋𝐛𝟑 𝟏
New iT = = = Old iT
𝟒𝟖 𝟒 𝟏𝟐 𝟒
𝟏
New iT = Old iT for one BHD
𝟐𝟐
𝟏
New iT = Old iT for two BHDs
𝟑𝟐 54
𝟏
New i = Old i for three BHDs
Prepared by Dr. Sudhir Sindagi

Free Surface Effect


Reducing FSE by inserting longitudinal bulkhead at the center of length
𝟏
New iT = Old iT for n in number longitudinal bulkheads inserted
(𝒏+𝟏)𝟐
𝟏 iT∗ρl
FSE = where n is number longitudinal bulkheads inserted
(𝒏+𝟏)𝟐 Δ
𝟏 iT∗ρl
FSE = where n is number longitudinal compartments created
(𝒏)𝟐 Δ

55
Prepared by Dr. Sudhir Sindagi

Problems
A vessel displacing 10000 tonnes, KG=8.9m, KM=9.4. The vessel loads
Ballast water of RD 1.01 into a rectangular tank 30mX20mX2m up to 1m
depth of the tank. The tank has a single centerline division. KG of BW is
0.5. Find the fluid GM of the vessel. Assume KM remains constant.
Given Data
Δ = 10000 tonnes, KG=8.9m, KM=9.4 ρl= 1.01 tonnes/m3 KG of BW=0.5m
Fluid GM=?
Weight added into the tank= volume of the tank* ρl=
w= 30*20*1*1.01= 606 tonnes
Shift in CG of the ship due to the addition of weight
wxd
GG1 =
Δ±w 56
Prepared by Dr. Sudhir Sindagi

Problems
Here d : the distance between the CG of the mass added and the
original location of the CG of the ship
w x d 606 x (𝟖.𝟗 −𝟎.𝟓)
GG1 = =
Δ±w 10606
GG1 =0.4799m (downwards)
FSE can be calculated in two ways
𝟏 iT∗ρl
FSE = where n is number longitudinal compartments created
(𝒏)𝟐 Δ
Since there is one centerline division creating two compartments, hence
n=2.
𝟏 30∗203∗1.01
FSE = 𝟐
(𝟐) 10606∗12 57
FSE =0.4761m upwards
Prepared by Dr. Sudhir Sindagi

Problems
FSE also can be calculated for individual compartments
Since both compartments are identical, hence Total FSE will be twice of
the FSE for the individual tank
iT∗ρl
FSE = *2
Δ
30∗103∗1.01
FSE = *2
10606 ∗12
FSE =0.4761m (upwards)
Fluid GM= KM-KG + downward shift in CG of ship-FSE
Fluid GM=9.4-8.9+0.4799-0.4761
Fluid GM=0.503m
58
Prepared by Dr. Sudhir Sindagi

Problems
A vessel of 10000 tonnes displacement, KM=9.3, KG=7.3 has two
rectangular identical deep tanks, port and starboard, each 15m long,
10m wide and 8m deep. The starboard tank is full of SW while the port
deep tank is empty. Calculate the fluid GM of the vessel when one
quarter of the water in the starboard deep tank is transferred to the port
deep tank.
Given Data
Δ = 10000 tonnes, KG=7.3m, KM=9.3 ρl= 1.025 tonnes/m3
Fluid GM=?
This is the typical problem of shifting of weight with FSE in both tanks
Weight shifted= volume of the liquid* ρl
w= 15*10*2*1.025= 307.5 tonnes 59
Prepared by Dr. Sudhir Sindagi

Problems
Shift in CG of the ship due to shifting of weight
wxd
GG1 =
Δ
307.5x 6
GG1 =
10000
GG1 =0.1845m (downwards)
Since both compartments are identical
Total FSE will be twice FSE of each tank
iT∗ρl
FSE = *2
Δ
15∗103∗1.025
FSE = *2
10000∗12
60
FSE =0.2562m (upwards)
Prepared by Dr. Sudhir Sindagi

Problems
Fluid GM= Solid GM-FSE
Fluid GM= KM-KG + downward shift in CG of ship-FSE
Fluid GM=9.3-7.3+0.1845-0.2562
Fluid GM=1.92825m

61
Prepared by Dr. Sudhir Sindagi

Problems
A ship of 8000 tonnes displacement has KM=7.5m, KG=7.0m. A double
bottom tank is 12m long, 15m wide and 1m deep. The tank is divided
longitudinally at the center line and both sides are full of salt water.
Calculate the list if one side is pumped out until it is half empty.
Given Data
Δ = 8000 tonnes, KG=7.0m, KM=7.5 ρl= 1.025 tonnes/m3
Angle of List=?
This is the typical problem of removal of weight with FSE in only one
tank
Weight removed= volume of the liquid* ρl
w= 12*7.5*0.5*1.025= 46.125 tonnes
62
Prepared by Dr. Sudhir Sindagi

Problems
Shift in CG of the ship due to the removal of weight
Here d : the distance between the CG of the mass added and the
original location of the CG of the ship
For horizontal shift d= 3.75m
w x d 46.125∗3.75
GGH = =
Δ−w 8000−46.125
GGH = 0.0217m (Port side)
For vertical shift d= 6.25m
w x d 46.125∗6.25
GGV = =
Δ−w 8000−46.125
GGV = 0.03624m (upwards)
63
Prepared by Dr. Sudhir Sindagi

Problems
Since there is only one tank which is half empty, hence Total FSE will be
for only one tank
iT∗ρl
FSE =
Δ
12∗7.53∗1.025
FSE =
(8000−46.125)∗12
FSE =0.0543m (upwards)
Fluid GM= Solid GM-FSE
Fluid GM=0.50-0.03624-0.0543
Fluid GM=0.40912m

64
Prepared by Dr. Sudhir Sindagi

Problems
GGH
𝐭𝐚𝐧 𝛟 =
Fluid GM
0.02174
𝐭𝐚𝐧 𝛟 =
𝟎.𝟒𝟎𝟗𝟏𝟐
Φ = 3.05 degrees on port side.

65
Prepared by Dr. Sudhir Sindagi

Inclining Experiment
Purpose:
To Find KG or GM of a vessel when it is as near to the completion as
possible, that is as near to the light condition as possible. The purpose
of this procedure is to achieve a satisfactory accuracy in the
determination of the lightship weight and of the coordinates of its CG.
Reason:
The above are initial conditions which must be known before the
stability of a ship in any particular condition of loading can be
determined. For example, when dealing with height of CG above the
keel, the initial position of the CG must be known to decide the final
one. Also since the KG is very great comparing to the GM, it must be
accessed very accurately if GM is to be found with reasonable
66
accuracy.
Prepared by Dr. Sudhir Sindagi

Inclining Experiment
l= Length of the pendulum
y= reading on the batton
𝒚
sin 𝝋 =
𝒍
wxd
But GG1 =
Δ
GG1
Also, tan 𝝋 =
𝐆𝐌
From here, we get GM
Now,
GM=KM- KG
KG = KB + BM - GM
67
Prepared by Dr. Sudhir Sindagi

Inclining Experiment
Procedure:
During the experiment, the ship is forcibly inclined by shifting weights
at fixed distance across the deck under controlled condition and finding
the resultant angle of heel.
The weights are usually concrete blocks and inclination is measured by
using long pendulum one forward and one aft of the midship section.
If third pendulum to be used, it is usually placed amidships.
The movement of the pendulum is made across battens which lie
perfectly horizontal when the ship is upright.
The pendulums are fixed at a height of about 10m above the battens at
the centerline of the ship.
The pendulum bob may be immersed in water or oil to damp 68 their

motion.
Prepared by Dr. Sudhir Sindagi

Inclining Experiment
Preparations before execution of an Inclining Experiment
The experiment should be carried out in calm water & nice weather. No
wind, no heavy rain, no tides.
It is essential that the ship be free to incline (mooring ropes should be
as slack as possible, but be careful.)
The vessel should be upright prior to the inclining. However, an initial
list of the ship not exceeding 0.5° is permissible.
Prior to the inclining test, lists of all items which are to be added,
removed, or relocated should be prepared. These weights and their
locations should be accurately recorded.
All objects should be secured in their regular positions. All weights
which may swing or shift must be secured in their known position.69
Prepared by Dr. Sudhir Sindagi

Inclining Experiment
Preparations before execution of an Inclining Experiment
Normally, the total value of missing weights should not exceed 2
percent and surplus weights, excluding liquid ballast, not exceed 4
percent of the light ship displacement
State of all tanks to be noted carefully.
Drafts to be accurately read amidships and on both sides of the ship.
The ship should be cleared of residues of cargo, tools, debris,
scaffolding and snow. Icing of the inner and outer surfaces, the
underwater hull included, is not permitted
Density of water is measured at a number of positions and depths
around the ship.
All cross connections between tanks are to be closed 70
Prepared by Dr. Sudhir Sindagi

Inclining Experiment
Preparations before execution of an Inclining Experiment
Preferably, all tanks should be either full or empty. The number of tanks
containing liquids should be kept to a minimum.
Soundings and density of liquids in tanks should be taken. Shapes of
tanks which are partly filled are to be known in order to determine the
free liquid surface effect
Minimum number of people on board and should remain on a specified
positions during each reading.
All service tanks and machinery plant pipings are to be filled as for the
working condition.
The angle of inclination should be small enough with the range of
validity of the theory. 71

The ship in experiment should not have a large trim.


Prepared by Dr. Sudhir Sindagi

Inclining Experiment
Procedure:
During the experiment, the ship is forcibly inclined by shifting weights
at fixed distance across the deck under controlled condition and finding
the resultant angle of heel.
The weights are usually concrete blocks and inclination is measured by
using long pendulum one forward and one aft of the midship section.
If third pendulum to be used, it is usually placed amidships.
The movement of the pendulum is made across battens which lie
perfectly horizontal when the ship is upright.
The pendulums are fixed at a height of about 10m above the battens at
the centerline of the ship.
The pendulum bob may be immersed in water or oil to damp 72 their

motion.
Prepared by Dr. Sudhir Sindagi

Problems
The inclining experiment is carried out on a vessel. The following data
is noted
– Displacement when inclined: 9550 tonnes
– Mass of inclining weight: 10 tonnes
– Distance of Transverse shift of weight: 18m
– Length of Pendulum line: 9.5m
– Mean deflection of pendulum line: 100mm
– KG of inclining weight: 12.5m
– KM=8.35.
Calculate the GM of the vessel. A tank containing 150tonnes of FW is
full at the time of experiment. KG of water is 1m. Calculate the lightship
KG of the ship.
73
Prepared by Dr. Sudhir Sindagi

Problems
Given Data
Δ = 9550 tonnes, w=10 tonnes, d= 18m, l= 9.5m, y=0.1m, KG of
weight=12.5m, KM=8.35m
GM=? FW weight = 150 tonnes and its KG=1m, KG light=?
𝒚
sin 𝝋 =
𝒍
Φ = 0.603 degrees.
wxd
But GG1 =
Δ
GG1 =0.01884m
GG1
tan 𝝋 =
𝐆𝐌
74
From here, we get GM = 1.787m
Prepared by Dr. Sudhir Sindagi

Problems
Now, GM=KM- KG
KGold= KM - GM
KGold= 6.5596m
To calculate the KG in lightship condition, the inclining weights and FW
are to be removed from the ship as they are not part of the lightweight
condition. Hence using the tabular method

Weight Distance from Keel Moment


ΣM
9550 6.5596 62644.18 Final KG=
ΣW
-10 12.5 -125
-150 1 -150 Lightship KG= 6.642m
ΣW=9390 ΣM=- 75
tonnes 62369.18 t-m
Prepared by Dr. Sudhir Sindagi

Problems
While loading a cargo of timber on a deck at KG=12m, it is noted that a
sling of timber weighing 8 tonnes moved from one side of the ship to
the other side by 16m and inclined the ship by 1 deg. KM at the draft
was 10.5m. Calculate the approx. KG. Displacement =13000 tonnes. How
much more cargo would be safe to load on the deck, if the GM was not
to be less than 0.5m.
Given Data
Δ = 13000 tonnes, w=8 tonnes, d= 16m, KG of weight=12.5m, KM=10.5m,
Φ = 01 degrees
If GM=0.5m How much more cargo can be loaded=?
wxd
But GG1 =
Δ 76
GG1 =0.009846m
Prepared by Dr. Sudhir Sindagi

Problems
GG1
tan 𝝋 =
𝐆𝐌
From here, we get GM = 0.564m
Now, GM=KM- KG
KGold= KM - GM
KGold= 9.935m
Now to have GM>0.5
KM- KG >0.5
KG <10m
GGV= KGNEW-Kgold
wxd
GGV=0.0641m = =
Δ+𝐰 77

W=416.65 tonnes
Prepared by Dr. Sudhir Sindagi

Problems
In an inclining experiment, a mass of 12.5 tonnes was moved 10 metres
across the deck and caused a plumb line, 12 metres long to move out
320 mm. A double bottom tank in the ship was full of water, during the
experiment. Mass of water in the tank is 450 tonnes and had its centre
of gravity 0.9 metres above the keel, without which the ship would have
been in the light condition. If the ship’s displacement at the time of
experiment was 3750 tonnes and her KM was 9.0 metres, find:
a) The KG at the time of experiment.
b) The light KG
Given Data
Δ = 3750 tonnes, w=12.5 tonnes, d=10m, l=12m, y=0.32m, KG of
weight=12.5m, KM=9m, BW=450 tonnes with KG=0.9m 78
KG= ? At the experiment and at the lightship condition
Prepared by Dr. Sudhir Sindagi

Problems
𝒚
sin 𝝋 =
𝒍
Φ = 1.528 degrees.
wxd
But GG1 =
Δ
GG1 =0.033m
GG1
tan 𝝋 =
𝐆𝐌
From here, we get GM = 1.237m
Now, GM=KM- KG
KGold= KM - GM
KGold= 7.762m
79
Prepared by Dr. Sudhir Sindagi

Problems
To calculate the KG in lightship condition, the inclining weights and BW
are to be removed from the ship as they are not part of the lightweight
condition. However, KG of inclining weights are not provided here,
hence assuming they are part of the lightship condition.
Hence using the tabular method

Weight Distance from Keel Moment


ΣM
3750 7.762 29107.5 Final KG=
ΣW
-450 0.9 -405
ΣW=3300 ΣM=-28702.5 Lightship KG= 8.697m
tonnes t-m
80
Prepared by Dr. Sudhir Sindagi

Empirical expressions for KB

81
Prepared by Dr. Sudhir Sindagi

Stability at the Large Angles of Heel


At the large angles of heel (greater than 7 degrees) the centre of
buoyancy will move further out and the low side and the force of
buoyancy can no longer be considered to act vertically upwards though
fixed position of M, the initial metacentre.
M will no longer remain in one position and it is no longer of direct
relevance to the calculation. By definition, M remains the point at which
successive lines of buoyancy intersect as the ship heels.
The ship will, in general, trim as well as heel. This means that the CB
moves fore and aft as well as transversely.
The deck edge and the turn of bilge will become immersed or exposed.
The metacentre moves as per: when B moves up, M moves down; and
when B moves down, M moves up. 82
Prepared by Dr. Sudhir Sindagi

Stability at the Large Angles of Heel

83
Prepared by Dr. Sudhir Sindagi

Stability at the Large Angles of Heel

84
Prepared by Dr. Sudhir Sindagi

Stability at the Large Angles of Heel


𝐁𝐌
Derivation for the Wall sided formula (𝐆𝐙 = (𝐆𝐌 + tan𝟐 𝛗) sin φ))
𝟐

85
Prepared by Dr. Sudhir Sindagi

Stability at the Large Angles of Heel


𝐁𝐌
Derivation for the Wall sided formula (𝐆𝐙 = (𝐆𝐌 + tan𝟐 𝛗) sin φ))
𝟐
In the case of large angles of heel, the vertical shift in the CB of the ship
due to the vertical shift in the centroid of the wedge cannot be
neglected.
This vertical shift in the CB causes vertical shift in the metacenter from
M to M’, causing an increase in the metacentric height from GM to GM’
resulting in an increase in the righting lever from GZ to GZ’.
New Righting Lever = GZ’ = GZ+ ZZ’
New Righting Lever = GM sin ϕ+ NB2
New Righting Lever = GM sin ϕ+ B1B2 sinϕ
New Righting Lever = (GM + B1B2)sinϕ
86
Prepared by Dr. Sudhir Sindagi

Stability at the Large Angles of Heel


𝟏
Volume of the elementary wedge (v) = 𝐲 ∗ 𝐲 tan φ 𝐝𝐱
𝟐
𝟏 𝟐
Volume of the elementary wedge (v) = 𝐲 tan φ 𝐝𝐱
𝟐
Here, the distance by which the volume of wedge is shifted vertically
upwards = distance by which the centroid is shifted from g to g1
𝟏 𝟏 𝟐
d= 𝐲 tanϕ+ 𝐲 tanϕ = 𝐲 tanϕ
𝟑 𝟑 𝟑
As mentioned earlier, this is case of the shift in volume, hence there is a
shift in the center buoyancy of the ship.
Small shift in the center buoyancy of the ship due to the shift in Volume
of the elementary wedge =
𝐕𝐨𝐥𝐮𝐦𝐞 𝐨𝐟 𝐰𝐞𝐝𝐠𝐞 ∗ 𝐝𝐢𝐬𝐭𝐚𝐧𝐜𝐞 𝐛𝐲 𝐰𝐡𝐢𝐜𝐡 𝐭𝐡𝐞𝐢𝐫 𝐜𝐞𝐧𝐭𝐫𝐨𝐢𝐝 𝐬𝐡𝐢𝐟𝐭𝐬 𝐯∗𝐝
= 87
𝐯𝐨𝐥𝐦𝐞𝐭𝐫𝐢𝐜 𝐝𝐢𝐬𝐩𝐥𝐚𝐜𝐞𝐦𝐞𝐧𝐭 𝐨𝐟 𝐭𝐡𝐞 𝐬𝐡𝐢𝐩 𝛁
Prepared by Dr. Sudhir Sindagi

Stability at the Large Angles of Heel


𝟏 𝟐
𝐲 tan φ 𝐝𝐱∗
𝟐
𝐲 tanϕ
Small shift in the center buoyancy of the ship = 𝟐 𝟑
𝛁
𝟏 𝟑
𝐲 𝐝𝐱
Small shift in the center buoyancy of the ship = 𝟑
∗ tan𝟐 𝝋
𝛁
Total shift in the center of buoyancy of the ship = BB1 which will be
obtained by integrating above equation for the entire length
𝟏 𝐋 𝟑 tan𝟐 𝝋
B1B2 = ‫𝐱𝐝 𝐲 𝟎׬‬ ∗
𝟑 𝛁
𝟏 𝐋 𝟑 IT
Here, the term = =Transverse Moment of inertia of the
‫𝐱𝐝 𝐲 𝟎׬‬
𝟑 𝟐
ship’s waterplane area taken at SLL about the longitudinal axis passing
through the centerline.
IT tan𝟐 𝝋 BM 88
B1B2 = ∗ = 𝟐
∗ tan 𝝋
Prepared by Dr. Sudhir Sindagi

Stability at the Large Angles of Heel


New Righting Lever = (GM + B1B2)sinϕ
BM
Righting Lever (GZ) = (GM + ∗ tan𝟐 𝝋)sinϕ
𝟐
This equation is applicable when angle of heel is greater than 70.
For small angles of heel below 70 Righting Lever (GZ) = GMsinϕ
BM
Increase in the Righting lever = ZZ’ = ∗ tan𝟐 𝝋sinϕ
𝟐
BM
Vertical shift in the Metacenter is MM’ = B1B2 = ∗ tan𝟐 𝝋
𝟐

89
Prepared by Dr. Sudhir Sindagi

Stability at the Large Angles of Heel


Angle of Loll: When a vessel has negative metacentric height (GM) i.e.,
is in unstable equilibrium, any external force applied to the vessel will
cause it to start heeling.
As it heels, the moment of inertia of the vessel's waterplane increases,
which increases the vessel's BM. Since there is relatively little change
in KB of the vessel, the KM of the vessel increases.
At some angle of heel (say 10°), KM will increase sufficiently equal to
KG, thus making GM of vessel equal to zero. When this occurs, the
vessel goes to neutral equilibrium, and the angle of heel at which it
happens is called angle of loll.
In other words, when an unstable vessel heels over towards a
progressively increasing angle of heel, at a certain angle of heel, the
90
centre of buoyancy (B) may fall vertically below the centre of gravity (G).
Prepared by Dr. Sudhir Sindagi

Stability at the Large Angles of Heel

91
Prepared by Dr. Sudhir Sindagi

Stability at the Large Angles of Heel


Angle of Loll
tan φ
BB1 = IT
𝛁
From the triangle MBB1, we can write
BB1
tan φ =
BM
Putting this in above equation and after simplification, we get
IT
BM =
𝛁

92
Prepared by Dr. Sudhir Sindagi

Stability at the Large Angles of Heel


Angle of Loll
The angle at which a ship with a negative initial metacentric height will
achieve the equilibrium condition is the angle of Loll.
In a seaway, such a ship will oscillate between the angle of loll on the
starboard and the on Port side.
Depending upon external forces such as wind and waves a ship may
suddenly flop over from PS to SB and then back again to PS.
Such abrupt oscillation, different from a continuous roll, is
characteristic for negative metacentric heights.
An angle of loll can be corrected only by lowering the centre of gravity,
not by moving loads transversely. This can be done by moving weight
downwards, adding water ballast in double bottom tanks or removing
93
weight above the ship vertical centre of gravity.
Prepared by Dr. Sudhir Sindagi

Stability at the Large Angles of Heel


Derivation for the Angle of Loll:
BM
We know the Righting Lever (GZ) = (GM + ∗ tan𝟐 𝛗)sinϕ
𝟐
However, for the Angle of Loll, the Value of GZ=0 as it is in the
equilibrium condition and more so GM= -ve.
BM
0= (-GM + ∗ tan𝟐 𝛗)sinϕ
𝟐
This will yield either sinϕ = 0 giving ϕ=0 for the equilibrium
condition or
BM
-GM + ∗ tan𝟐 𝛗 =0
𝟐
BM
∗ tan𝟐 𝛗 = GM 94
𝟐
Prepared by Dr. Sudhir Sindagi

Stability at the Large Angles of Heel


Derivation for the Angle of Loll:
BM
∗ tan𝟐 𝛗 = GM
𝟐
2GM
tan𝟐 𝛗 =
𝐁𝐌

2GM
𝛗= ± tan−𝟏
𝐁𝐌
This is the expression by which one can estimate the angle of Loll for
the initial unstable ship.

95
Prepared by Dr. Sudhir Sindagi

Problems
A ship 12000 t displacement has second moment of area about the
centerline 72x103 m4. If the metacentric height is -0.05. Calculate the
Angle of Loll. At one point during Voyage, the above vessel is found to
have angle of Loll 130. Calculate the initial Metacentric height.
Given Data
Δ=12000 tonne, GM=-0.05m, IT= 72x103m3, 𝛗 = ?
If Angle of Loll (𝛗) = 130. then GM=?
2GM
Angle of Loll 𝛗 = ± tan−𝟏
𝐁𝐌
IT
But BM = , 𝐈𝐓 =72x103m3 , 𝛁 = Δ/ ρ = 12000/1.025
𝛁
BM = 6.15m 96
Prepared by Dr. Sudhir Sindagi

Problems
2GM
Angle of Loll 𝛗 = ± tan−𝟏
𝐁𝐌

−𝟏 2∗𝟎.𝟎𝟓
𝛗 = ± tan
𝟔.𝟏𝟓
𝛗 = ±7.268 degrees
Now in the second case 𝛗) = 130
2∗𝐆𝐌
𝟏𝟑 = ± tan−𝟏
𝟔.𝟏𝟓
GM= -0.1638m

97
Prepared by Dr. Sudhir Sindagi

GZ Curve
The easiest and handiest tool for analysing a surface ship’s stability, is
by graphs or curves.
Since the stability of a ship can be directly commented on by the nature
and value of its metacentric height (GM), a direct method to track the
stability of a ship for a range of heel angles would be, to generate a
curve that relates this parameter to the angle of heel.
Since metacentric height is directly related to the righting lever (GZ) and
angle of heel, the curve of statical stability is a plot between the righting
lever and angle of heel.

98
Prepared by Dr. Sudhir Sindagi

GZ Curve

99
Prepared by Dr. Sudhir Sindagi

GZ Curve

100
Prepared by Dr. Sudhir Sindagi

GZ Curve
MAIN FEATURES OF THE GZ CURVE
Slope at the origin. For small angles of heel, the righting lever is
proportional to the angle of inclination, the metacentre being effectively
a fixed point.
Initial GM: The tangent to the GZ curve at the origin represents the
metacentric height at the angle of heel equal to 1 rad = 57.3 degrees.
Maximum GZ. This is proportional to the largest steady heeling moment
that the ship can sustain without capsizing, and its value and the angle
at which it occurs are both important.

101
Prepared by Dr. Sudhir Sindagi

GZ Curve
MAIN FEATURES OF THE GZ CURVE
Range of stability. At some angle, often greater than 90 degrees, the GZ
value reduces to zero and becomes negative for larger inclinations. This
angle is known as the angle of vanishing stability and the range of angle
for which GZ is positive is known as the range of stability. For angles
less than this, a ship will return to the upright state when the heeling
moment is removed.
Angle of deck edge immersion. For most ship forms, there is a point of
inflexion in the curve corresponding roughly to the angle at which the
deck edge becomes immersed. In general, of course, the angle at which
the deck edge is immersed varies along the length, but is within a fairly
narrow band for the larger sections amidships which exert most
influence upon the stability. 102
Prepared by Dr. Sudhir Sindagi

GZ Curve
MAIN FEATURES OF THE GZ CURVE
Area under the curve. The area under the curve represents the ability of
the ship to absorb energy imparted to it by winds, waves or any other
external agency.
The dynamical stability of a ship at any inclination is defined as the
work done in heeling the vessel to that inclination.
Dynamical stability = Δ* Area under the stability curve
The dynamic stability of a ship is the area enclosed within its static
stability curve. It gives us the magnitude of external heeling energy that
the ship can absorb before capsizing.

103
Prepared by Dr. Sudhir Sindagi

Problems
A ship displacing 5000t has GZ values as follows when KG= 6m. Draw
the Statical Stability Curve for this condition and for KG= 5m. Discuss
the effect on Stability of lowering the CG of ship.
Angle of 0 15 30 45 60 75 90
Heel (θ)
GZ(m) 0 0.14 0.36 0.73 0.67 0 -0.65
Given Data
Δ=5000 tonne, KG1=6m, KG2=5m,
To draw the statical stability curve(GZ Curve), need to select suitable
scale to fit on the graph paper.
Suggested scales on X and y axis of the graph papers are
104
Prepared by Dr. Sudhir Sindagi

Problems
On X axis, which is longer side of the graph paper
1cm = 5 degree (always)
On Y axis, which is shorter side of the graph paper
1cm ≈ (Difference of maxima and minima of GZ values)/15
In this case, it will be 1cm≈(0.73-(-0.65)/18≈0.08m
Write these scales on the graph paper(Top right location)
Position location of the X axis in such way that, you can locate the
negative value of GZ if given on the graph paper.
Plot the graph by locating the points with values in the table. Join these
points with smooth curves and from graphs obtain following
Angle of Vanishing Stability, Range of Stability, GZ max and Initial GM.
105
Prepared by Dr. Sudhir Sindagi

Problems
To draw GZ curve at different
value of KG
Need to calculate values of GZ
for new KG values as per the
following formula.
G1Z1 = GZ ± GG1* sinϕ
When G goes down new values
of GZ increases and Vice versa.
With these new values of GZ
values for different values of
angles of heel, one can plot
similar GZ curve as per the 106
procedure mentioned earlier.
Prepared by Dr. Sudhir Sindagi

Problems
A ship of 5000 tonnes displacement has righting levers as follows.

Angle of Heel (θ) 10 20 30 40


GZ(m) 0.21 0.33 0.40 0.43

Calculate the dynamical stability to 40 degrees heel


We know that, Dynamical stability = Δ* Area under the stability curve
To calculate the area under the GZ curve, one can make use Simpson’s
rule.
Here, n=5 (Including value of GZ=0 at 0 degree angle of heel)
Hence using Simpson’s 1st rule
h= 10 degrees=0.1745 rad (Always use angles in degrees 107during
multiplication or division)
Prepared by Dr. Sudhir Sindagi

Problems
Col(1) Col (2) Col (3) Col(4)= (2)X(3)

Simpson’s
Angle GZ For Area
Multiplier
0 0 1 0
10 0.21 4 0.84
20 0.33 2 0.66
30 0.40 4 1.6
40 0.43 1 0.43
ΣA=3.53 rad-m
𝐡
Total Area under the GZ curve=A= * ΣA=0.2053 rad-m
𝟑
Dynamical stability = 1026.5 tonnes-m- rad = 1026.5* 9.81 kJ
108
Dynamical stability = 10069.965 kJ
Prepared by Dr. Sudhir Sindagi

IMO Criteria for GZ Curve


The area under the righting lever curve (GZ curve) shall not be less than
– 0.055 metre-radians up to 30° angle of heel and
– not less than 0.09 metre-radians up to 40° or the angle of down-
flooding, whichever is smaller.
– Additionally, the area under the righting lever curve (GZ curve)
between the angles of heel of 30° and 40° or between 30° and angle
of down-flooding, whichever is smaller, shall not be less than 0.03
metre-radians.
The righting lever GZ shall be at least 0.2 m at an angle of heel equal to
or greater than 30°.
The maximum righting lever shall occur at an angle of heel not less than
25°. 109

The initial metacentric height GM shall not be less than 0.15 m.


Prepared by Dr. Sudhir Sindagi

IMO Criteria for GZ Curve

110
Prepared by Dr. Sudhir Sindagi

IMO Criteria for GZ Curve


Severe wind and rolling criterion (weather criterion)
The ability of a ship to withstand the combined effects of beam wind
and rolling shall be demonstrated,

111
Prepared by Dr. Sudhir Sindagi

IMO Criteria for GZ Curve


Severe wind and rolling criterion (weather criterion)
The ship is subjected to a steady wind pressure acting perpendicular to
the ship’s centreline which results in a steady wind heeling lever (lwl1)
𝐏∗𝐀∗𝐙
𝐥𝐰𝐥𝟏 = & 𝐥𝐰𝐥𝟐 = 1.5 𝐥𝐰𝐥𝟏
𝟏𝟎𝟎𝟎∗𝐠∗∆
P = 504 N/m2 (wind speed = 29 m/s).
A = projected lateral area of the portion of the ship and deck cargo
above the waterline in m2.
Z = vertical distance from the centre of A to the centre of the underwater
lateral area or approximately to a point at one half the draught in m.
𝐥𝐰𝐥𝟐 = a gust wind heeling lever
112
Prepared by Dr. Sudhir Sindagi

IMO Criteria for GZ Curve


Severe wind and rolling criterion (weather criterion)
From the resultant angle of equilibrium (ϕ0), the ship is assumed to roll
owing to wave action to an angle of roll (ϕ1) to windward.
The angle of heel under action of steady wind (ϕ0) should be limited to a
certain angle to the satisfaction of the Classification Society. As a
guide, 16° or 80% of the angle of deck edge immersion, whichever is
less, is suggested.
The ship is then subjected to a gust wind pressure which results in a
gust wind heeling lever lwl2
Under these circumstances, area “b” should be equal to or greater than
area “a”
113
Prepared by Dr. Sudhir Sindagi

IMO Criteria for GZ Curve


Severe wind and rolling criterion (weather criterion)
ϕ0 = angle of heel under action of steady wind
ϕ1 =angle of roll to windward due to wave action
ϕ2 =angle of down-flooding (ϕf) or 500 or ϕc, whichever is less,
where:
ϕf = angle of heel at which openings in the hull, superstructures or
deckhouses which cannot be closed weathertight immerse.
ϕc = angle of second intercept between wind heeling lever 𝐥𝐰𝐥𝟐 and GZ
curves.

114
Prepared by Dr. Sudhir Sindagi

IMO Criteria for GZ Curve


International Code for the Safe Carriage of Grain in Bulk
The angle of heel due to the shift of grain shall not be greater than 12°
or in the case of ships constructed on or after 1 January 1994 the angle
at which the deck edge is immersed, whichever is the lesser;
In the statical stability diagram, the net or residual area between the
heeling arm curve and the righting arm curve up to the angle of heel of
maximum difference between the ordinates of the two curves, or 40° or
the angle of down flooding whichever is the least, shall in all conditions
of loading be not less than 0.075 metre-radians; and
The initial metacentric height, after correction for the free surface
effects of liquids in tanks, shall be not less than 0.30 m
115
Prepared by Dr. Sudhir Sindagi

IMO Criteria for GZ Curve


International Code for the Safe Carriage of Grain in Bulk

116
Prepared by Dr. Sudhir Sindagi

IMO Criteria for GZ Curve


International Code for the Safe Carriage of Grain in Bulk

117
Prepared by Dr. Sudhir Sindagi

IMO Criteria for GZ Curve


Additional criteria are recommended for passenger ships
1. The angle of heel on account of crowding of passengers to one side
should not exceed 10°.
2. The angle of heel on account of turning should not exceed 10° when
calculated using the following formula:
V2 T
MR = 0.02 Δ KG −
L 2
MR : heeling moment in metre-tons
V : service speed in m/sec.,
L : length of ship at waterline in m.,
 : displacement in metric tons,
T : mean draught in m. , 118
KG : height of centre of gravity above keel in m.
Prepared by Dr. Sudhir Sindagi

Cross Curves of Stability – KN Curves


The vertical position of centre of gravity (G) of the ship is not always
fixed. It changes with every voyage, depending on the loading
conditions and the amount of ballast.
The CG of the ship also changes when the ship is in transit.
This varying nature of CG makes it difficult for the designer to assume
the loading conditions at which GZ curves should be obtained, because
different values of KG would result in different metacentric heights
(GM), and righting lever (GZ), the stability curves for each of the loading
condition would be different.
The cross curves of stability were developed, so that, for any loading
condition (where KG is already known), values of righting lever (GZ) can
be obtained for all angles of heel.
119
Prepared by Dr. Sudhir Sindagi

Cross Curves of Stability – KN Curves


The point ‘K’ represents the
keel of the ship.
If at any angle of heel (ϕ), a
line parallel to that of GZ is
drawn from ‘K’, then the point
of intersection of this line with
the line of action of buoyancy,
is represented as ‘N’.
GZ = KN- KG* sinϕ
The only unknown in the above
expression is the value of KN
which are obtained from KN
120
curves.
Prepared by Dr. Sudhir Sindagi

Cross Curves of Stability – KN Curves


Once the KN
curves are
obtained, it is
now possible for
the designer to
obtain the
stability curve /
GZ curve for any
loading
condition

121
Prepared by Dr. Sudhir Sindagi

Hydrostatic Curves
A series of graphs drawn to a vertical scale of draught and a base of
length, which gives values such as, Vertical, Longitudinal and
Transverse Center of Buoyancy, Mass Displacement (Δ), Volume
Displacement (∇), Longitudinal and Transverse Centre of Floatation,
Metacentric Radius, Moment to Change Trim 1 cm (MCT), Tonnes per
cm Immersion (TPC).
All the hydrostatic parameters are calculated by a stability analysis and
are plotted on a graph against different drafts. This graph is collectively
called hydrostatic curves.
In practice tables with hydrostatic parameters calculated for different
draughts are used.

122
Prepared by Dr. Sudhir Sindagi

Hydrostatic Curves

123
Prepared by Dr. Sudhir Sindagi

Hydrostatic Curves

124
Prepared by Dr. Sudhir Sindagi

Summary
Introduction – Types of Equilibrium
Conditions for the stability of the ship and submarine
Stiff and Tender Ship
Stability for the small angles of heel
– Derivation for the Metacentric radius
– Free Surface Effect
– Inclining Experiment
Stability at the large angles of heel
– Derivation for the wall sided formula
– Angle of Loll
– GZ Curve
– Dynamical Stability of the ship 125
– Requirements of the IMO for the GZ Curve
Prepared by Dr. Sudhir Sindagi

Longitudinal and Damaged stability

By
Dr. Sudhir Sindagi 1
Prepared by Dr. Sudhir Sindagi

Chapter Content
Introduction- Trim
Trimming Moment, MCT 1cm, IL, BML, GML
Trim due to
– Shifting of existing weight
– Addition or removal of weight
Damaged Stability
– Deterministic damage stability
• Added Weight Method
• Lost Buoyancy Method
– Permeability
– Probabilistic damage stability
– Floodable length, Margin Line, Permissible Length, Factor of 2
Subdivision, Floodable Length curve
Prepared by Dr. Sudhir Sindagi

Introduction
Trim may be considered as the longitudinal equivalent of list.
Trim is also known as ‘longitudinal stability’. It is in effect transverse
stability turned through 90°.
Instead of trim being measured in degrees it is measured as the
difference between the drafts forward and aft.
If difference is zero then the ship is said to be on even keel.
Trim =0 , if TA=TF

3
Prepared by Dr. Sudhir Sindagi

Introduction
If forward draft is greater than aft draft, the vessel is said to be trimming
by the bow (forward). Trim by forward (t) = TF-TA

If aft draft is greater than the forward draft, the vessel is said to be
trimming by the stern (aft). Trim by aft (t) = TA-TF

4
Prepared by Dr. Sudhir Sindagi

Introduction
In case of Trim by aft, if TM is the mean draft at F, then x is the increase
in the draft at AP above F (mean draft)
x= increase in the draft at AP above F (mean draft)
𝐓𝐀 −𝐓𝐅 𝐭
x=LCF* tan θ = LCF∗ = 𝐋𝐂𝐅 ∗
𝐋 𝐋
y= reduction in the draft at FP below F= t-x

5
Prepared by Dr. Sudhir Sindagi

Introduction
𝐓𝐀 +𝐓𝐅
Mean draft at F = TM= Applicable when F lies at Midship
𝟐
𝐭
TM= 𝐓𝐀 -x = 𝐓𝐀 - 𝐋𝐂𝐅 ∗ Applicable when F lies anywhere.
𝐋
TM= 𝐓𝐅 +y = 𝐓𝐅 + (𝐭 − 𝐱)

6
Prepared by Dr. Sudhir Sindagi

Introduction
In case of Trim by forward, if TM is the mean draft at F, then x is the
reduction in the draft at AP below F (mean draft)
x= reduction in the draft at AP below F (mean draft)
𝐓𝐅 −𝐓𝐀 𝐭
x=LCF* tan θ = LCF∗ = 𝐋𝐂𝐅 ∗
𝐋 𝐋
y= increase in the draft at FP above F= t-x

7
Prepared by Dr. Sudhir Sindagi

Introduction
𝐓𝐀 +𝐓𝐅
Mean draft at F = TM=
𝟐
𝐭
TM= 𝐓𝐀 +x = 𝐓𝐀 + 𝐋𝐂𝐅 ∗
𝐋
TM= 𝐓𝐅 −y = 𝐓𝐅 - (𝐭 − 𝐱)

8
Prepared by Dr. Sudhir Sindagi

Trimming Moment, MCTC, IL, BML, GML

9
Prepared by Dr. Sudhir Sindagi

Trimming Moment, MCTC, IL, BML, GML

10
Prepared by Dr. Sudhir Sindagi

Trimming Moment, MCTC, IL, BML, GML


Consider a ship to be floating at rest in still water and on an even keel
as shown in Figure 1
Now let a weight ‘w’, already on board, be shifted aft through a distance
‘d’, . This causes the centre of gravity of the ship to shift from G to G1,
parallel to the shift of the centre of gravity of the weight shifted, so that:
wxd
GG1 =
Δ
Δ*GG1 = w x d
Trimming Moment =Δ*GG1 = w x d is generated
The ship will now trim until the centres of gravity and buoyancy are
again in the same vertical line as shown in the Figure 2.
11
Prepared by Dr. Sudhir Sindagi

Trimming Moment, MCTC, IL, BML, GML


When trimmed, the wedge of buoyancy LFL1 emerges and the wedge
WFW1 is immersed.
Since the ship, when trimmed, must displace the same weight of water
as when on an even keel, the volume of the immersed wedge must be
equal to the volume of the emerged wedge and F, the point about which
the ship trims, is the centre of gravity of the water-plane area. The point
F is called the ‘centre of flotation’ or ‘tipping centre’.
A vessel with a rectangular water-plane has its centre of flotation on the
centre line amidships but, on a ship, it may be a little forward or abaft
amidships, depending on the shape of the water-plane. In trim
problems, unless stated otherwise, it is to be assumed that the centre of
flotation is situated amidships.
12
Prepared by Dr. Sudhir Sindagi

Trimming Moment, MCTC, IL, BML, GML


Trimming moments are taken about the centre of flotation since this is
the point about which rotation takes place.
The longitudinal metacentre (ML) is the point of intersection between the
verticals through the longitudinal positions of the centres of buoyancy.
The vertical distance between the centre of gravity and the longitudinal
metacentre (GML) is called the longitudinal metacentric height.
BML is the height of the longitudinal metacentre above the CB
IL = Longitudinal Moment of inertia of the ship’s waterplane about the
Transverse axis passing through the centroid of the waterplane.
𝐁𝐋𝟑 IL
IL = , BML =
𝟏𝟐 𝛁
𝐋𝟐
Longitudinal Metacentric Radius = BML = for rectangular barge13
𝟏𝟐𝐓
Prepared by Dr. Sudhir Sindagi

Trimming Moment, MCTC, IL, BML, GML

14
Prepared by Dr. Sudhir Sindagi

Trimming Moment, MCTC, IL, BML, GML


Righting Moment in the longitudinal direction = Δ * GZL
𝐆𝐙𝐋
sin θ =
𝐆𝐌𝐋
Since the angle of trim is very small sin θ = tan θ
𝐆𝐙𝐋
tan θ =
𝐆𝐌𝐋
𝐆𝐙𝐋 = 𝐆𝐌𝐋 tan θ
Righting Moment in the longitudinal direction = Δ * GZL
Righting Moment in the longitudinal direction = Δ * 𝐆𝐌𝐋 tan θ
t
Righting Moment in the longitudinal direction = Δ * 𝐆𝐌𝐋
𝐋
This is the moment which acts on the ship, to float it on the even keel.
15
Prepared by Dr. Sudhir Sindagi

Trimming Moment, MCTC, IL, BML, GML


If Trim (t) = 1cm
1 cm
Moment required to change the trim by 1cm= Δ * 𝐆𝐌𝐋
𝐋
Δ ∗ 𝐆𝐌𝐋
MCT1cm=
𝟏𝟎𝟎𝐋
Since the angle of trim is very small, hence the distance GML is very
high as compared to the distance BG. In the view of above, in case no
data is available, it is assumed that
GML≈BML
Δ ∗ 𝐁𝐌𝐋
MCT1cm≈
𝟏𝟎𝟎𝐋

16
Prepared by Dr. Sudhir Sindagi

Steps to solve Problems of Trim


1. Calculate the Trimming moment using
1. w*d- when weight is shifted by a distance d
2. Tabular method – when weights are added or removed from and to the ship
2. For the trimming moment all distances are to be taken from F.
3. If MCT 1cm is not given then calculate it using
Δ ∗ 𝐁𝐌𝐋
4. MCT1cm≈
𝟏𝟎𝟎𝐋
5. Calculate the trim using Trimming moment and MCT1cm using
Trimming Moment
6. Trim in cm=
MCT1cm
7. Calculate change in draft at aft using
t
8. x=LCF* tan θ = x=LCF* 17
𝐋
Prepared by Dr. Sudhir Sindagi

Steps to solve Problems of Trim


9. Calculate change in draft at forward using
10. y= t – x
11. In the case, weight is added or removed from or to the ship, then
calculate the change in draft using TPC as follows
weight added or removed
12. Change in draft in cm (ΔT)=
𝐓𝐏𝐂
13. Calculate final drafts at AP and FP using tabular method.
Drafts AP (m) FP (m)
Original
Change in draft due to Trimming Moment ±x ±y
Change in draft due to weight addition or removal ±ΔT ±ΔT
18
Final Drafts
Prepared by Dr. Sudhir Sindagi

Problems
A vessel displacing 30000 tonnes is floating at drafts of 8.3m at FP and
9.6m at AP. MCT1 is 300. Center of flotation is 109m forward of AP,
Length of the ship is 210m. Find the drafts at FP and AP if 1000 tonnes
of ballast is moved from a tank CG of 175m forward of AP to a tank with
CG 205m forward of AP.
Given Data
Δ = 30000 tonnes, 𝐓𝐅 = 8.3m, 𝐓𝐀 = 9.6m, MCT1=300, LCF=109m forward
of AP, L=210m, w=1000 tonnes, d=(205-175)=30m
𝐓𝐅 = ?, 𝐓𝐀 = ?
Trimming moment= w*d = 30000 t-m
Trimming Moment 30000
Trim in cm= =
MCT1cm 300 19
Trim in cm (t)= 100 cm =1m
Prepared by Dr. Sudhir Sindagi

Problems
Since the weight is being shifted from aft to forward, hence draft at AP
will reduce and will increase at FP.
Reduction in draft at AP = x=
t 100
x=LCF* = 109*
𝐋 𝟐𝟏𝟎
Reduction in draft at AP =x =51.904cm=0.51904m
Increase in draft at FP = y = t - x
Increase in draft at FP = y = 0.4809m
Since, weight is not added and not removed hence there is no need to
calculate the change in draft using TPC.

20
Prepared by Dr. Sudhir Sindagi

Problems
Making table for Final draft calculations.

Drafts AP (m) FP (m)


Original
Change in draft due to
Trimming Moment
Change in draft due to
weight addition or removal
Final Drafts

21
Prepared by Dr. Sudhir Sindagi

Problems
Making table for Final draft calculations.

Drafts AP (m) FP (m)


Original 9.6 8.3
Change in draft due to
( - ) 0.51903 ( + ) 0.4809
Trimming Moment
Change in draft due to
0 0
weight addition or removal
Final Drafts 9.0810 8.7809

22
Prepared by Dr. Sudhir Sindagi

Problems
A ship 100 m in length floats at draughts forward 7.00 m and aft 6.80 m.
Calculate the final draughts if 150 tonnes is loaded 20 m forward of aft
perpendicular given that TPC is 15 and MCTC is 150 tm and LCF is 45 m
forward of aft perpendicular.
Given Data
L=100m, 𝐓𝐅 = 7.0m, 𝐓𝐀 = 6.8m, MCT1=150, LCF=45m forward of AP,
w=150 tonnes, TPC=15
𝐓𝐅 = ?, 𝐓𝐀 = ?
Here, since weight is added, hence to calculate the trimming moment, it
will be calculated from F
d= (45-20) = 25m
Trimming moment= w*d = 3750 t-m 23
Prepared by Dr. Sudhir Sindagi

Problems
Trimming Moment 3𝟕𝟓𝟎
Trim in cm= =
MCT1cm 150
Trim in cm= 25 cm =0.25m
Since the weight is added aft of CF, hence draft at AP will increase and
will decrease at FP.
Increase in draft at AP = x=
t 0.25
x=LCF* = 45*
𝐋 𝟏𝟎𝟎
Increase in draft at AP =x =0.1125m
Reduction in draft at FP = y = t - x
Reduction in draft at FP = y = 0.1375m
24
Prepared by Dr. Sudhir Sindagi

Problems
Here, weight is added to the ship, hence the change in draft using TPC
will be
weight added or removed
Change in draft in cm =
𝐓𝐏𝐂
150
Change in draft in cm =
𝟏𝟓
Change in draft in cm = 10 cm = 0.1m
Drafts AP (m) FP (m)
Original 6.8 7.0
Change in draft due to Trimming Moment + 0.1125 - 0.1375
Change in draft due to weight addition or removal 0.1 0.1
25
Final Drafts 7.0125 6.9625
Prepared by Dr. Sudhir Sindagi

Problems
A ship 100 m long arrives in port with draughts of 3m at FP and 4.3 m at
AP. The hydrostatic particulars are TPC=10, MCTC= 120 tm, LCF= 3m aft
of amidships. 80 tonnes of cargo is now loaded at a position of 24 m
ford of amidships and 40 tonnes of cargo is discharged from 12m aft of
amidships. Find out the new draughts.
Given Data
L=100m, 𝐓𝐅 = 3.0m, 𝐓𝐀 = 4.3m, MCT1=120, LCF=3m aft of Midship, w1=80
tonnes, and w2=40 tonnes, TPC=10
𝐓𝐅 = ?, 𝐓𝐀 = ?
Trimming Moment = w1*d1+w2*d2 (forward trim)
Trimming Moment = 80*27+40*9
Trimming Moment = 2520 t-m (forward trim) 26
Prepared by Dr. Sudhir Sindagi

Problems
Trimming Moment 2520
Trim in cm= =
MCT1cm 120
Trim in cm= 21 cm =0.21m
Since it is creating forward trim, hence draft at AP will decrease and will
increase at FP.
Reduction in draft at AP = x=
t 0.21
x=LCF* = 47*
𝐋 𝟏𝟎𝟎
Reduction in draft at AP =x =0.09827m
Increase in draft at FP = y = t - x
Increase in draft at FP = y = 0.1113m
27
Prepared by Dr. Sudhir Sindagi

Problems
Here, one weight is added to the ship and the other is removed, hence
the change in draft using TPC will be
weight added or removed
Change in draft in cm =
𝐓𝐏𝐂
(80−40)
Change in draft in cm =
𝟏𝟎
Change in draft in cm = 4 cm = 0.04m
Drafts AP (m) FP (m)
Original 4.3 3.0
Change in draft due to Trimming Moment - 0.0987 + 0.1113
Change in draft due to weight addition or removal 0.04 0.04
28
Final Drafts 4.2413 3.1513
Prepared by Dr. Sudhir Sindagi

Problems
A vessel floating at drafts forward 9.84m and aft 10.62m. She loads
following weights. TPC=26, MCT1=148 tonnes-m/cm, LCF=64m forward
of AP, Length=120m. Find final drafts. Is it trimmed by aft or forward? .
Weight (Tonnes) LCG from AP (m)
Loads 450 25
Loads 320 100
Discharges 140 110

Given Data
L=120m, 𝐓𝐅 = 9.84m, 𝐓𝐀 = 10.62m, MCT1=148, LCF=64m forward of AP,
TPC=26
𝐓𝐅 = ?, 𝐓𝐀 = ?
29
Prepared by Dr. Sudhir Sindagi

Problems
Weight (Tonnes) Lever from F (m)
Loads 450 39
Loads 320 36
Discharges 140 46

Trimming Moment = w1*d1+w2*d2 +w3*d3(Aft trim)


Trimming Moment = 450*39-320*36+140*46
Trimming Moment = 12470 t-m (aft trim)
Trimming Moment 12470
Trim in cm= =
MCT1cm 148
Trim in cm= 0.8425m
Since it is creating aft trim, hence draft at AP will increase and will
30
reduce at FP.
Prepared by Dr. Sudhir Sindagi

Problems
Increase in draft at AP = x=
t 0.8425
x=LCF* = 64*
𝐋 𝟏𝟐𝟎
Increase in draft at AP =x =0.4493m
Reduction in draft at FP = y = t - x
Reduction in draft at FP = y = 0.3931m
weight added or removed
Change in draft in cm =
𝐓𝐏𝐂
(450+320−140)
Change in draft in cm =
𝟐𝟔
Change in draft in cm = 0.2423m
31
Prepared by Dr. Sudhir Sindagi

Problems

Drafts AP (m) FP (m)


Original 10.62 9.84
Change in draft due to Trimming Moment + 0.4493 - 0.3931
Change in draft due to weight addition or removal 0.2423 0.2423
Final Drafts 11.3116 9.6896

32
Prepared by Dr. Sudhir Sindagi

Problems
A vessel has following details
TF=10.8m, TA=11.4m, TPC=28 tonnes/cm, L=138m, LCF=65m forward of
AP. It has to complete loading with final drafts as TF=10.9m, TA=11.4m.
Find the amount of cargo load to achieve above.
Given Data
L=138m, 𝐓𝐅 = 10.8m, 𝐓𝐀 = 11.4m, LCF=65m forward of AP, TPC=28
To achieve 𝐓𝐅 = 10.9, 𝐓𝐀 = 11.4 what load to be added?
Here, no MCT1 is provided and as no other required data is provided to
calculate it, hence calculating required values from mean draft, initial &
final trim and TPC.
Initial Trim = ti = 𝐓𝐀 - 𝐓𝐅 = 0.6m
Final Trim = tf = 𝐓𝐀 - 𝐓𝐅 = 0.5m 33
Prepared by Dr. Sudhir Sindagi

Problems
𝐭𝐢
TMi= 𝐓𝐀𝐢 - 𝐋𝐂𝐅 ∗
𝐋
TMi= 11.1173m
𝐭𝐟
TMf= 𝐓𝐀𝐢 - 𝐋𝐂𝐅 ∗
𝐋
TMf= 11.1644m
Change in mean draft expected = ΔTmean= TMf- TMi= 0.0471m
Weight to be added = ΔTmean* TPC
Weight to be added =131.88 tonnes

34
Prepared by Dr. Sudhir Sindagi

Problems
A ship 150 m long floats at draughts of 8.20 m Forward and 8.9 m Aft.
MCT 1 cm 260 tonne-m TPC 28 and LCF 1.5m Aft of midship. It is
necessary to bring the ship to an even keel and a double bottom tank 60
m forward of midships is available. Calculate the mass of water required
and the final draughts.
Given Data
L=150m, 𝐓𝐅 = 8.2m, 𝐓𝐀 = 8.9m, MCT 1 cm= 260, LCF= 1.5m Aft of
midship, TPC=28
To bring the ship to an even keel calculate mass of water required to be
added?
𝐓𝐅 = ?, 𝐓𝐀 = ?
35
Prepared by Dr. Sudhir Sindagi

Problems
Here, the initial trim and the trimming moment needs to be nullified.
Initial Trim = t = 𝐓𝐀 - 𝐓𝐅 = 0.7m
Trimming Moment = trim * MCT1 cm
Trimming Moment = 18200 t-m
Trimming Moment = weight to added * distance from F
18200= w* (1.5+60)
w= 295.93 tonnes.
Since weight is added at the forward end, hence it will cause increase in
the draft at FP and the reduction in draft at AP.
Reduction in draft at AP = x=
t 0.7
x=LCF* = 73.5* 36
𝐋 𝟏𝟓𝟎
Prepared by Dr. Sudhir Sindagi

Problems
Reduction in draft at AP =x =0.343m
Increase in draft at FP = y = t - x
Increase in draft at FP = y = 0.357m
weight added
Change in draft in cm =
𝐓𝐏𝐂
(295.93)
Change in draft in cm =
𝟐𝟖
Change in draft in cm = 0.1056m

37
Prepared by Dr. Sudhir Sindagi

Problems

Drafts AP (m) FP (m)


Original 8.9 8.2
Change in draft due to Trimming Moment - 0.343 + 0.357
Change in draft due to weight addition or removal 0.1056 0.1056
Final Drafts 8.6626 8.6626

38
Prepared by Dr. Sudhir Sindagi

Change in trim due to change in density


When a ship passes from sea water into river water, or vice versa
without change in displacement, there is a change in trim in addition to
the change in mean draught. This change in trim is always very smaIl.
Consider a ship of displacement Δ floats in sea water wherein G and B
are in the same vertical line.
If the vessel moves into river water of lesser density there is a bodily
sinkage resulting in increase in draught.

39
Prepared by Dr. Sudhir Sindagi

Change in trim due to change in density


The volume of displacement has been increased by a layer of volume v,
whose centroid is assumed to be at the centre of flotation F, resulting in
the total shift as FB.
This causes the centre of buoyancy to move from B to B1 and the centre
of gravity remaining at G.
Let 𝛁𝐒 & 𝛁𝐑 be the volumetric displacements in SW and RW, wherein,
change in underwater volume= v= 𝛁𝐑 - 𝛁𝐒

40
Prepared by Dr. Sudhir Sindagi

Change in trim due to change in density


Shift in Center of Buoyancy of the ship due to additional sinkage in the
𝐯∗𝐝 𝐯∗𝐅𝐁
ship is BB1= =
𝛁 𝛁𝐑
(𝛁𝐑 −𝛁𝐒 )∗𝐅𝐁
BB1=
𝛁𝐑
(𝛒𝐒 −𝛒𝐑 )∗𝐅𝐁
BB1=
𝛒𝐒
Since B1 is no longer in line with G, a trimming moment of Δ* BB1 acts
on the ship causing a change in trim by the bow
Trimming Moment Δ∗ BB1
Trim in cm= =
MCT1cm MCT1cm
Δ∗FB (𝛒𝐒 −𝛒𝐑 )
Trim in cm=
MCT1cm 𝛒𝐒
41
If the ship moves from the RW into SW, It will change trim by the stern.
Prepared by Dr. Sudhir Sindagi

Change in trim due to change in density


Four important cases:
If F lies aft of B
Ship moves from SW to FW or RW (moves from higher density to lower
density), the Ship will trim by the Bow.
Ship moves from RW or FW to SW (moves from lower density to higher
density), the Ship will trim by the stern.
If F lies forward of B
Ship moves from SW to FW or RW (moves from higher density to lower
density), the Ship will trim by the stern.
Ship moves from RW or FW to SW (moves from lower density to higher
density), the Ship will trim by the bow.
42
Prepared by Dr. Sudhir Sindagi

Change in trim due to change in density


A ship 120 m long and 9100 tonne displacement floats at a level keel
draught of 6.50 m in fresh water of 1.000 t/m3 MCT1=130 tonnes-m, TPC
in sea water 16.5. LCB 2.30 m forward of midships. LCF 0.6 m aft of
midships. Calculate the new draughts if the vessel moves into sea water
of 1.024 t/m3 without change in displacement
Given Data
L=120m, Δ=9100 tonnes 𝐓𝐅 = 6.5m, 𝐓𝐀 = 6.5m, LCF=0.6m aft of midship,
TPCSW=16.5, LCB 2.30 m forward of midships
New drafts=? When it moves from FW to SW of ρ= 1.024 t/m3.
When the ship moves from FW to SW, the draft reduces, which can be
calculated using FWA as
Δ ρSW – ρRW 43
FWA =
TPCsw ρRW
Prepared by Dr. Sudhir Sindagi

Change in trim due to change in density


FWA =13.2cm =0.132m
Change in trim to change in density of water is
Δ∗FB (𝛒𝐒 −𝛒𝐑 )
Trim in cm=
MCT1cm 𝛒𝐒
Since the ship moves from the RW into SW & LCF is aft of LCB, It will
change trim by the stern.
Trim in cm (t)=4.87cm by stern = 0.0487 by stern
t 𝟎.𝟎𝟒𝟖𝟕
Increase in draft at aft =x= LCF* = 59.4*
𝐋 𝟏𝟐𝟎
Increase in draft at aft =x = 0.0241m (+ve)
Reduction in draft at forward = y = t-x = 0.0246m (-ve)
44
Prepared by Dr. Sudhir Sindagi

Problems

Drafts AP (m) FP (m)


Original 6.5 6.5
Change in draft due to Trimming Moment + 0.0241 - 0.0246
Change in draft due to change in density -0.132 -0.132
Final Drafts 6.3921 6.3434

45
Prepared by Dr. Sudhir Sindagi

Change in trim due to change in density


A vessel with forward draft 8.72m and aft draft =9m is floating in SW.
She enters in DW of RD=1.004. Find the initial and the mean drafts and
her drafts in DW. Given that, MCTC=162, LCF=82m forward of AP,
LCB=90m forward of AP, L=170m, TPC=29.8 and Δ=2700 tonnes
Given Data
Follow the similar procedure to solve the question.

46
Prepared by Dr. Sudhir Sindagi

Determination of trim, due to the


addition of large masses
When a large mass is added to a ship, the resultant increase in draught
is sufficient to cause changes in all the hydrostatic details.
It then becomes necessary to calculate the final draughts from first
principles. Such a problem exists every time a ship loads or discharges
the major part of its deadweight.
The principle is that after loading or discharging, the vessel is in
equilibrium and the final G is in the same vertical line as the final B.
For any given condition of loading, it is possible to calculate the
displacement Δ and the longitudinal position of the centre of gravity G
relative to midships.
From the hydrostatic curves or data, the mean draught may be obtained
at this displacement, and hence the value of MCT1 and the distance 47
of
the LCB and LCF from midship.
Prepared by Dr. Sudhir Sindagi

Determination of trim, due to the


addition of large masses
These values are calculated for the level keel condition and it is unlikely
that the LCB will be in the same vertical line as G.
Thus a trimming moment acts on the ship which is the product of the
displacement and the longitudinal distance between B and G, known as
the trimming lever.
The trimming moment, divided by the MCT1 cm gives the change in trim
from the level keel condition.
The vessel changes trim about the LCF and hence it is possible to
calculate the end draughts. When the vessel has changed trim in this
manner, the new centre of buoyancy B1 lies in the same vertical line as
G.
48
Prepared by Dr. Sudhir Sindagi

Problems
A ship 125 m long has a light displacement of 4000 tonne with LCG 1.60
m aft of midships. The following items are now added:
– Cargo 8500 tonne LCG 3.9 m forward of midships
– Fuel 1200 tonne LCG 3.1 m aft of midships
– Water 200 tonne LCG 7.6 m aft of midships
– Stores 100 tonne LCG 30.5 m forward of midships.
At 14000 tonne displacement the mean draught t is 7.80 m, MCT1cm 160
tonne m, LCB 2.00 m forward of midships and LCF 1.5m aft of midships.
Calculate the final draughts.
Given Data
L=125m, Δ=4000 tonnes,𝐓𝐦𝐞𝐚𝐧 = 7.8m at 14000 tonnes, LCF=1.5m aft of
midship, LCB 2m forward of midships, LCG 1.60 m aft of midships
49
New drafts=? When large weights are added
Prepared by Dr. Sudhir Sindagi

Problems
Weight Lever from Moment
(Tonnes) Midship
Cargo 8500 3.9 33150
Fuel 1200 -3.1 -3720
Water 200 -7.6 -1520
Stores 100 30.5 3050
Lightweight 4000 -1.6 -6400
ΣW=14000 ΣM=24560
LCG= ΣM/ ΣW =1.754m forward of midship
LCB =2.0 forward of midship
Trimming Lever = 2.0- 1.745 =0.246m
Trimming Moment = 14000*0.246 50

Trimming Moment =3444 tm ---It will be trim by aft


Prepared by Dr. Sudhir Sindagi

Change in trim due to change in density


Trimming Moment
Trim in cm=
MCT1cm
3444
Trim in cm=(t)= = 21.525cm
160
t 𝟎.𝟐𝟏𝟓𝟐𝟓
Increase in draft at aft =x= LCF* = 61*
𝐋 𝟏𝟐𝟓
Increase in draft at aft =x = 0.1050m (+ve)
Reduction in draft at forward = y = t-x = 0.1102m (-ve)
𝐓𝐀 = 𝐓𝐦𝐞𝐚𝐧 +x =7.905m
𝐓𝑭 = 𝐓𝐦𝐞𝐚𝐧 − 𝐲=7.6898m

51
Prepared by Dr. Sudhir Sindagi

Stability of Ship during Docking


The process of docking and undocking of ships might not seem like an
important operation.
However, it’s a process that is carried out more than once, not only by
shipbuilding yards during the construction of a ship, but also as regular
part of the ship’s lifetime.
The understanding of the process of docking is specialised, and hence,
not many naval architects or engineers are thorough with the inner
details of docking.
From time to time, it becomes important to carry out repairs in the
underwater portion of the hull, such as renewal of the sacrificial anodes,
refit of the propellers, overhauling of the propulsion shafts, repair of
rudders, underwater hull blasting to remove fouling, etc.
52
Prepared by Dr. Sudhir Sindagi

Stability of Ship during Docking


In order to carry out these repairs, the underwater portion of the hull
needs to be made accessible, which is the purpose served by a dry
dock.
It has also become a common practice in large shipyards to build their
ships on dry docks, and float it out when ready for trials.
For such procedures, the docking plans need to be prepared taking into
consideration the increase in weight of the ship structure along the
building time.
Once the ship has been built, the dry dock is flooded and the ship is
undocked.
The calculations for undocking also play a major role in the process
because it is during undocking that the ship is at a risk of capsizing.
53
Prepared by Dr. Sudhir Sindagi

Stability of Ship during Docking


When the ship enters a dry dock, it must have a positive metacentric
height; and is usually trimmed by stern.
The floor of the dry dock is lined with keel blocks, which are so
arranged such that they can bear the weight of the ship.
The dock gates are then closed and the water is pumped out of the dock
in stages.
Since the ship has a trim by stern, the stern of the ship will first sit on
the keel blocks. The rate of pumping out water is reduced as the stern is
almost about to touch the keel blocks

54
Prepared by Dr. Sudhir Sindagi

Stability of Ship during Docking


When the ship’s stern just touches the keel blocks, part of the ship’s
weight is being borne by the keel blocks.
The contact between the stern and the keel block creates a normal
reaction or upthrust. The magnitude of this upward normal reaction
increases as the water level in the dry dock reduces.

55
Prepared by Dr. Sudhir Sindagi

Stability of Ship during Docking


It is this upthrust that creates a virtual reduction in the metacentric
height of the ship.
Hence it is very crucial to maintain sufficient positive metacentric height
before docking, lacking which, the ship may heel over to either side, or
even slip off the keel blocks and capsize.
Since the location of the stern is a known point, its distance from the
center of floatation (l) can be calculated instantly

56
Prepared by Dr. Sudhir Sindagi

Stability of Ship during Docking


Figure shows the longitudinal section of a ship during the critical
period.
‘P’ is the upthrust at the stern and ‘l’ is the distance of the centre of
flotation from aft. The trimming moment is given by P*l.
But the trimming moment is also equal to MCTC * change of trim (t)
P*l = MCTC * change of trim (t)

57
Prepared by Dr. Sudhir Sindagi

Stability of Ship during Docking

58
Prepared by Dr. Sudhir Sindagi

Stability of Ship during Docking


Now consider Figure which shows a transverse section of the ship
during the critical period after she has been inclined to a small angle (Ɵ
degrees) by a force external to the ship.
For the sake of clarity the angle of heel has been magnified. The weight
of the ship (W) acts downwards through the centre of gravity (G).
The force P acts upwards through the keel (K) and is equal to the weight
being borne by the blocks.
For equilibrium the force of buoyancy must now be (W - P) and will act
upwards through the initial metacentre (M).
There are, thus, three parallel forces to consider when calculating the
effect of the force P on the ship’s stability.
Two of these forces may be replaced by their resultant in order 59to find
the effective metacentric height and the moment of statical stability
Prepared by Dr. Sudhir Sindagi

Stability of Ship during Docking


Righting Moment = (W - P) * GZ – P * GZ’
Righting Moment = (W - P) * GM sin Ɵ – P * KG sin Ɵ
Righting Moment = W * GM sin Ɵ - P * GM sin Ɵ – P * KG sin Ɵ
Righting Moment = W * GM sin Ɵ – (GM + KG) * P sin Ɵ
Righting Moment = W * GM sin Ɵ – KM * P sin Ɵ
𝐏
Righting Moment = W * (GM – 𝐊𝐌) sin Ɵ

The above equation is equivalent, Righting Moment = ∆ ∗ 𝐆𝐌 sin θ
𝐏
New GM during docking = GM - 𝐊𝐌

𝐏
Loss of GM during docking = 𝐊𝐌

𝐏 60
If 𝐊𝐌 > GM, then the ship becomes unstable.

Prepared by Dr. Sudhir Sindagi

Problems
Just before entering drydock, a ship of 5000 tonnes floats at draft of
2.7m forward and 4.2m aft. The length between perpendiculars is 150m.
Assuming blocks are horizontal and based on following hydrostatic
data, find the force on the heel of the stern frame, which is at the AP,
when the ship is just about to settle on the dock blocks and the GM at
that instant. KG=8.5m, KM=9.3m, MCT1=107 t-m/cm, LCF=2.7m aft of
amidships.
Given Data
L=150m, Δ=5000 tonnes,𝐓𝐀 = 4.2m, 𝐓𝐅 = 2.7m, LCF=2.7m aft of midship,
KG=8.5m, KM=9.3m, MCT1=107 t-m/cm
P=? GM during Docking=?
61
Prepared by Dr. Sudhir Sindagi

Problems
Initial Trim = 𝐓𝐀 -𝐓𝐅 = t=1.5m by aft
Distance of CF from the point of resting the ship= l = 72.3m
Using below equation, the upthrust(P) can be calculated
P*l = MCTC * change of trim (t), we get
P= 222 tonnes
𝐏
Loss of GM during docking = 𝐊𝐌, we get

Loss of GM= 0.41m
Final GM during Docking = Initial GM- Loss of GM
Final GM during Docking = KM- KG – loss of GM
Final GM during Docking = 0.39m
62
Prepared by Dr. Sudhir Sindagi

Problems
As ship of 3000 tonnes displacement and 100m length has KM=6m,
KG=5.5m, LCF= 2m aft of amidships. MCT1=40 t-m/cm. Find the
maximum trim for the ship to enter a drydock of the GM at the critical
instant before the ship takes blocks forward and aft is not less than
0.3m.
Given Data
L=100m, Δ=3000 tonnes,𝐓𝐀 = 4.2m, 𝐓𝐅 = 2.7m, LCF=2m aft of midship,
KG=5.5m, KM=6.0m, MCT1=40 t-m/cm
t=? When GM during Docking>0.3m
Loss of GM permitted = Initial GM- Final GM
Loss of GM permitted = 6.0 – 5.5 – 0.3 = 0.2m
𝐏 63
Loss of GM during docking = 𝐊𝐌 = 0.2

Prepared by Dr. Sudhir Sindagi

Problems
We get, P= 100 tonnes
Now we know that
P*l = MCTC * change of trim (t), we get
Here, l=48m
Permitted trim < 1.2m

64
Prepared by Dr. Sudhir Sindagi

Damaged Stability
The term damage stability deals with the ability of a ship to float in
water and regain its upright equilibrium position when some sort of
structural damage has occurred.
Generally, following an accident, the damage is hull fracture leading to
flooding of ship compartments.
If so many compartments are flooded that there is not enough buoyancy
available to keep the vessel afloat, the ship may sink.
Another critical scenario due to hull breach is ship capsizing due to
loss of transverse stability as it can happen very quickly.
To assess the behaviour of a ship after some damage two methods are
considered:
– Deterministic damage stability 65
– Probabilistic damage stability
Prepared by Dr. Sudhir Sindagi

Damaged Stability
Deterministic Damage Stability
This is a traditional method of assessment of the stability of a ship
when it is flooded. In this process, the ship is divided into several
subdivisions along its length with the help of transverse watertight
bulkheads. Now the stability of the ship is calculated when one or more
compartments get flooded due to a breach of hull.
The changes in draft and stability when a compartment becomes
flooded due to damage can be investigated by either of two methods:
• Lost Buoyancy method
• Added weight method

66
Prepared by Dr. Sudhir Sindagi

Damaged Stability
Deterministic Damage Stability: Lost Buoyancy method
The damaged compartment(s) is considered open to the sea and
therefore, does not contribute to the buoyancy of the ship.
So, the lost buoyancy must be compensated by sinkage of the vessel
and the moment due to change in LCB of the vessel is manifested
through the heel or trim of the vessel.
The assumptions considered in this method are that the flooded
compartment does not provide buoyancy anymore and hence, there is
no change in displacement or KG of the vessel and no free surface
effect is observed.

67
Prepared by Dr. Sudhir Sindagi

Damaged Stability
Deterministic Damage Stability: Added Weight method
This method considers that water ingresses in the damaged
compartments up to the new water level and the weight of the ingressed
water augments the displacement of the vessel that is compensated by
the sinkage of the vessel.
Consequently, the KG of the vessel changes due to the weight of
ingressed water and Free Surface Effects has to be taken into account,
if the compartment is partially filled with water.
The weight added shifts the CG of the vessel that might lead to list or
trim of the vessel.
Both methods will give identical answers for final draughts, trim and
RM, despite different values for GM. However, IMO/SOLAS recommends
68
the use of Lost buoyancy method for all calculations
Prepared by Dr. Sudhir Sindagi

Lost Buoyancy Method


Bilging amidships compartments
When a vessel floats in still water it
displaces its own weight of water.
Figure shows a box-shaped vessel
floating at the waterline, WL.
Now let an empty compartment
amidships be holed below the waterline
to such an extent that the water may flow
freely into and out of the compartment.
A vessel holed in this way is said to be
‘bilged’.
69
Prepared by Dr. Sudhir Sindagi

Lost Buoyancy Method


Bilging amidships compartments
Figure shows the vessel in the bilged condition. The buoyancy provided
by the bilged compartment is lost. The draft has increased and the
vessel now floats at the waterline W1L1, where it is again displacing its
own weight of water. ‘X’ represents the increase in draft due to bilging.

70
Prepared by Dr. Sudhir Sindagi

Lost Buoyancy Method


Bilging amidships compartments
The volume of lost buoyancy (v) is made good by the volumes ‘y’ and
‘z’.
v= y + z
Let ‘A’ be the area of the water-plane before bilging, and let ‘a’ be the
area of the bilged compartment. Then
y + z = A*x –a*x
v = x* (A - a)
𝐯
Increase in draft = x =
𝐀 −𝐚
𝐕𝐨𝐥𝐮𝐦𝐞 𝐨𝐟 𝐥𝐨𝐬𝐭 𝐛𝐮𝐨𝐲𝐚𝐧𝐜𝐲
x=
𝐀𝐫𝐞𝐚 𝐨𝐟 𝐢𝐧𝐭𝐚𝐜𝐭 𝐰𝐚𝐭𝐞𝐫𝐩𝐥𝐚𝐧𝐞
71
Prepared by Dr. Sudhir Sindagi

Lost Buoyancy Method - Problems


A box-shaped vessel is 50 metres long and is floating on an even keel at
4 metres draft. A amidships compartment is 10 metres long and is
empty. Find the increase in draft if this compartment is bilged.
Given Data:
L=50m, T=4m, l=10m
Volume of the compartment =l*B*T =10*B*4 =40B
𝐯
Increase in draft = x =
𝐀 −𝐚
40B
Increase in draft = x =
𝟓𝟎∗𝐁 −𝟏𝟎𝐁
Increase in draft = x =1m

72
Prepared by Dr. Sudhir Sindagi

Lost Buoyancy Method - Problems


A box-shaped vessel is 150 metres long x 24 metres wide x 12 metres
deep, and is floating on an even keel at 5 metres draft. GM = 0.9 metres.
A compartment amidships is 20 metres long and is empty. Find the new
GM if this compartment is bilged.
Given Data:
L=150m, B= 24m, TOLD=5m, GMOLD= 0.9m, l=20m, GMnew=?
Volume of the compartment =l*b*T =20*24*5 =2400m3
𝐯
Increase in draft = x =
𝐀 −𝐚
2400
Increase in draft = x =
𝟏𝟓𝟎∗𝟐𝟒−𝟐𝟎∗𝟐𝟒
Increase in draft = x =0.7692m≈0.77m
73
New Draft≈ 5.77m
Prepared by Dr. Sudhir Sindagi

Lost Buoyancy Method - Problems


Since, in this problem, KG value is not provided, one need to estimate
its KG value based on the old GM value, using
GMOLD= KBOLD+BMOLD-KG
Here KG remains same, as in the lost buoyancy method, it is assumed
that KG does not changes.
KBOLD= TOLD/2 =2.5m
𝐁𝟐
BMOLD- = =9.6m
𝟏𝟐𝐓𝐎𝐋𝐃
Putting these values in the equation for GMOLD= , we get
KG= 11.2m
To calculate, GMNEW,will use following equation
GMNEW= KBNEW+BMNEW-KG 74
Prepared by Dr. Sudhir Sindagi

Lost Buoyancy Method - Problems


KBNEW= TNEW/2 =2.885m
𝐁𝟐
BMNEW- = =8.3188m
𝟏𝟐𝐓
NEW
Putting these values in the equation for GMNEW= , we get
GMNEW= KBNEW+BMNEW-KG
GMNEW= 0.003m

75
Prepared by Dr. Sudhir Sindagi

Lost Buoyancy Method


Permeability (µ):
Permeability is the amount of water that can enter a compartment or
tank after it has been bilged. When an empty compartment is bilged, the
whole of the buoyancy provided by that compartment is lost.
Typical values for permeability, are as follows:
– Empty compartment - 100%
– Engine room - 80% to 85%
– Grain-filled cargo hold - 60% to 65%
– Coal-filled compartment - 36% approximately
– Filled water ballast tank (when ship is in salt water) - 0%
Consequently, the higher the value of the permeability for a bilged
compartment, the greater will be a ship’s loss of buoyancy when the
76
ship is bilged.
Prepared by Dr. Sudhir Sindagi

Lost Buoyancy Method


Permeability (µ):
When a bilged compartment contains cargo, the formula for finding the
increase in draft must be amended to allow for the permeability.
If ‘µ’ represents the permeability, expressed as a fraction, then the
volume of lost buoyancy will be ‘µ*v’ and the area of the intact water-
plane will be ‘A - µ*a’ square metres. The formula then reads.
𝛍𝐯
Increase in draft = x =
𝐀 −𝛍𝐚

77
Prepared by Dr. Sudhir Sindagi

Permeability (µ) - Problems


A box-shaped vessel is 64 metres long and is floating on an even keel at
3 metres draft. A compartment amidships is 12 m long and contains
cargo having a permeability of 25 per cent. Calculate the increase in the
draft if this compartment be bilged.
Given Data:
L=64m, T=3m, l=12m, µ=0.25, x=?
Volume of water entering the compartment =µ*l*b*T =0.25*12*B*3 =9B
µ𝐯
Increase in draft = x =
𝐀 −µ𝐚
9B
Increase in draft = x =
𝟔𝟒𝐁−𝟎.𝟐𝟓∗𝟏𝟐∗𝐁
Increase in draft = x =0.1475m≈0.15m
78
Prepared by Dr. Sudhir Sindagi

Permeability (µ) - Problems


A box barge 60 m long and 10 m wide floats at an even keel draught of 4
m. It has a compartment amidships 12 m long. Calculate the new
draughts if this compartment is laid open to the sea when permeability
is (i) 100% (ii) 85% (iii) 60%.
Given Data:
L=60m, B= 10m, T=4m, l=12m, x=? When µ=1.0, 0.85 and 0.6.
X= 1m when µ=1.0
X= 0.8192m when µ=0.85
X= 0.5454m when µ=0.6

79
Prepared by Dr. Sudhir Sindagi

Permeability (µ) - Problems


A box barge of 58m long and 8m wide floats a draught of 5m and has a
mid- length compartment 9m long containing coal (relative density 1.28)
which stows at 1.22 m³/t. Calculate the new draught if this compartment
is bilged.
Given Data:
L=58m, B= 8m, T=5m, l=9m, x=? When coal (relative density 1.28) which
stows at 1.22 m³/t.
Stowage factor is space occupied by the cargo in m³ per tonnes of it.
Total weight of the cargo = volume of the compartment / stowage factor.
Total weight of the cargo = 9*8*5 / 1.22=295.08 tonnes
Volume of the cargo occupied by the cargo = weight of the cargo /
Density of the cargo 80
Prepared by Dr. Sudhir Sindagi

Permeability (µ) - Problems


Volume of the cargo occupied by the cargo = weight of the cargo /
Density of the cargo
Volume of the cargo occupied by the cargo = 295.08 / 1.28
Volume of the cargo occupied by the cargo =230.53 m3
Volume available for the water to enter in it once is flooded = Total
volume of the compartment - Volume of the cargo in the compartment
Volume available for the water to enter in it once is flooded = 360 -
230.53 = 129.467 m3
Permeability of the compartment = Volume available for the water to
enter in it once is flooded / Total volume of the compartment
µ = 0.3596
81
Prepared by Dr. Sudhir Sindagi

Permeability (µ) - Problems


µ𝐯
Increase in draft = x =
𝐀 −µ𝐚
𝟎.𝟑𝟓𝟗𝟔∗𝟑𝟔𝟎
Increase in draft = x =
𝟓𝟖∗𝟖 −𝟎.𝟑𝟓𝟗𝟔∗𝟗∗𝟖
Increase in draft = x = 0.2954m
New Draft = 5.2954m

82
Prepared by Dr. Sudhir Sindagi

Permeability (µ) - Problems


A box-shaped vessel 75m long, 10m wide and 6m deep is floating in salt
water on an even keel at a draft of 4.5m. Find the new drafts if a forward
compartment 5m long is bilged.
Given Data:
L=75m, B= 10m, T=4.5m, l=5m, 𝐓𝐅 = ?, 𝐓𝐀 = ?
When the bilged compartment is situated in a position away from
amidships, the vessel’s mean draft will increase to make good the lost
buoyancy but the trim will also change.
There will be a horizontal component of the shift of the centre of
buoyancy (B1B2) equal to half the length of the compartment flooded.
A trimming moment of W * B1B2 by the head is produced and the vessel
will trim about the centre of flotation (F). 83
Prepared by Dr. Sudhir Sindagi

Permeability (µ) - Problems


Trimming moment = W * B1B2 = w * d
It can therefore be seen that the effect on trim is similar to that which
would be produced if a mass equal to the lost buoyancy were loaded in
the bilged compartment.
Such questions shall be solved in a similar method of Trim problems.

84
Prepared by Dr. Sudhir Sindagi

Permeability (µ) - Problems


Trimming moment = w * d
Weight of the water = 5*10*4.5*1.025= 230.625 tonnes
d = Lever from new LCF. Here, due to flooding, the CF will shift aft
wards from midship by a distance of half the length of the compartment
and the weight acts at the centroid of the compartment.
d = 37.5m
Trimming moment = 230.625 * 37.5 = 8648.4375 t-m.
Trimming Moment
Trim in cm=
MCT1cm
Here, MCT1 cm is not given and hence to be calculated for the box
barge.
85
Prepared by Dr. Sudhir Sindagi

Permeability (µ) - Problems


Δ ∗ 𝐁𝐌𝐋
MCT1cm≈
𝟏𝟎𝟎𝐋
Δ = Volume * density = 75 * 10 * 4.5 *1.025
Δ = 3459.375 tonnes.
𝐈𝐋
𝐁𝐌𝐋 =
𝛁
𝐁𝐋𝐍𝐄𝐖 𝟑
IL = , here the new length of the ship = 70m and 𝛁 will remain
𝟏𝟐
same.
𝐁𝐌𝐋 = 84.6914m
𝟑𝟒𝟓𝟗.𝟑𝟕𝟓∗𝟖𝟒.𝟔𝟗𝟏𝟒
MCT1cm≈ , here in the denominator L= 75m as drafts at
𝟏𝟎𝟎 ∗𝟕𝟓
AP and FP are to be calculated which are separated by a distance of
86
75m
Prepared by Dr. Sudhir Sindagi

Permeability (µ) - Problems


MCT1cm≈ 𝟑𝟗. 𝟎𝟔𝟑𝟗
Trimming Moment 8648.4375
Trim in cm= =
MCT1cm 39.0639
Trim in cm=221.39cm=2.21m by forward.
Since the forward compartment is flooded, hence the ship will trim by
forward.
t 221.39
Reduction in draft at AP = x= x=LCF* = 35* , here LCF= 35 at
𝐋 𝟕𝟓
new location of F from AP
Reduction in draft at AP =x =103.32cm=1.033m
Increase in draft at FP = y = t - x
Increase in draft at FP = y = 1.1807m 87
Prepared by Dr. Sudhir Sindagi

Permeability (µ) - Problems


weight added or removed
Change in draft in cm =
𝐓𝐏𝐂
𝛒 ∗𝐀𝐖𝐏 𝟏.𝟎𝟐𝟓 ∗𝟕𝟎∗𝟏𝟎
TPC = = = 7.175, here for Area, new length is taken
𝟏𝟎𝟎 𝟏𝟎𝟎
230.625
Change in draft in cm =
𝟕.𝟏𝟕𝟓
Change in draft in cm = 32.143 cm = 0.3214m

Drafts AP (m) FP (m)


Original 4.5 4.5
Change in draft due to Trimming Moment - 1.033 + 1.1807
Change in draft due to weight addition or removal 0.3214 0.3214
88
Final Drafts 3.7884 6.0021
Prepared by Dr. Sudhir Sindagi

Permeability (µ) - Problems


A box shaped vessel 75m long, 10m wide and 6m deep is floating in SW
on an even keel at a draft of 4.5. Find the new drafts if a forward
compartment 7m long is bilged.
Given Data:
L=75m, B= 10m, T=4.5m, l=7m, 𝐓𝐅 = ?, 𝐓𝐀 = ?
Trimming moment = w * d
Weight of the water = 322.875 tonnes
d = 37.5m
Trimming moment = 12107.8125 t-m.
Trimming Moment
Trim in cm=
MCT1cm
89
Prepared by Dr. Sudhir Sindagi

Permeability (µ) - Problems


Δ ∗ 𝐁𝐌𝐋
MCT1cm≈
𝟏𝟎𝟎𝐋
Δ = 3459.375 tonnes.
𝐈𝐋
𝐁𝐌𝐋 =
𝛁
𝐁𝐋𝐍𝐄𝐖 𝟑
IL = , here the new length of the ship = 68m and 𝛁 will remain
𝟏𝟐
same.
𝐁𝐌𝐋 = 77.64m
MCT1cm≈ 35.81
Trimming Moment
Trim in cm=
MCT1cm
Trim in cm=3.381 m by forward. 90
Prepared by Dr. Sudhir Sindagi

Permeability (µ) - Problems


t
Reduction in draft at AP = x= x=LCF*
𝐋
Reduction in draft at AP =x =1.5329m
Increase in draft at FP = y = t - x
Increase in draft at FP = y = 1.8484m
weight added or removed
Change in draft in cm =
𝐓𝐏𝐂
𝛒 ∗𝐀𝐖𝐏
TPC = = 6.97, here for Area, new length is taken
𝟏𝟎𝟎
Change in draft in cm = 0.4632m

91
Prepared by Dr. Sudhir Sindagi

Permeability (µ) - Problems

Drafts AP (m) FP (m)


Original 4.5 4.5
Change in draft due to Trimming Moment - 1.5329 + 1.8484
Change in draft due to weight addition or removal 0.4632 0.4632
Final Drafts 3.4303 6.8116

92
Prepared by Dr. Sudhir Sindagi

Permeability (µ) - Problems


A box-shaped vessel 100 metres long, 20 metres wide and 12 metres
deep is floating in salt water on an even keel at 6 metres draft. A forward
compartment is 10 metres long, 12 metres wide and extends from the
outer bottom to a watertight flat, 4 metres above the keel. The
compartment contains cargo of permeability 25 per cent. Find the new
drafts if this compartment is bilged.
Given Data:
L=100m, B= 20m,
T=6m, l=10m, b=12m,
𝐓𝐅 = ?, 𝐓𝐀 = ?
When µ=0.25.
93
Prepared by Dr. Sudhir Sindagi

Permeability (µ) - Problems


In this question, as the compartment depth is limited to the height of 4m
and hence is not touching the WL, hence there wont be changes in
length of WL after flooding.
Trimming moment = w * d
Weight of the water = 123 tonnes
d = Lever from LCF. Here, the CF will not shift aft wards.
d = 45m
Trimming moment = 5535 t-m.
Trimming Moment
Trim in cm=
MCT1cm
Here, MCT1 cm is not given and hence to be calculated for the box
barge. 94
Prepared by Dr. Sudhir Sindagi

Permeability (µ) - Problems


Δ ∗ 𝐁𝐌𝐋
MCT1cm≈
𝟏𝟎𝟎𝐋
Δ = Volume * density
Δ = 12300 tonnes.
𝐈𝐋
𝐁𝐌𝐋 =
𝛁
𝐁𝐋𝐍𝐄𝐖 𝟑
IL = , here the new length of the ship = old length and 𝛁 will
𝟏𝟐
remain same as usual.
𝐁𝐌𝐋 = 138.889m
MCT1cm≈ 170.467

95
Prepared by Dr. Sudhir Sindagi

Permeability (µ) - Problems


Trimming Moment
Trim in cm=
MCT1cm
Trim in cm=0.324m by forward.
Since the forward compartment is flooded, hence the ship will trim by
forward.
t
Reduction in draft at AP = x= x=LCF*
𝐋
Reduction in draft at AP =x =0.162m
Increase in draft at FP = y = t - x
Increase in draft at FP = y = 0.162m

96
Prepared by Dr. Sudhir Sindagi

Permeability (µ) - Problems


Volume of the compartment =µ*l*b*T =0.25 * 10 * 12 * 4 =120 m3
µ𝐯
Increase in draft = x =
𝐀 −µ𝐚
120
Increase in draft = x =
𝟏𝟎𝟎∗𝟐𝟎−𝟎.𝟐𝟓∗𝟏𝟐∗𝟏𝟎
Increase in draft = x =0.0609m

Drafts AP (m) FP (m)


Original 6 6
Change in draft due to Trimming Moment -0.162 + 0.162
Change in draft due to weight addition or removal 0.0609 0.0609
97
Final Drafts 5.8989 6.2229
Prepared by Dr. Sudhir Sindagi

Damaged Stability
Probabilistic Damage Stability
Damage stability calculations by probabilistic damage assessment is
required by SOLAS Chapter II-1, part B. This is required for cargo ships
80 m in length and upwards and to all passenger ships regardless of
length.
This approach is based on probability of survival after certain
compartment or group of compartments are damaged.
Attained index is a measure of ship's safety after damage or Collision.
Two ship's with same index are assumed to have same level of safety
for damage or collision, irrespective of location of damage.
Probability of survival is calculated as sum of probability of damage of a
space or group spaces multiplied by probabilities of survival98 after
corresponding space damage.
Prepared by Dr. Sudhir Sindagi

Damaged Stability
Probabilistic Damage Stability
This approach uses the concept of probability to ensure that ships can
survive damage to its compartment(s). There are two probability factors
that are used in this approach.
Probability that a particular compartment(s) will damage in an incident
(factor “p”)
the probability that ship will survive if that compartment(s) is flooded
(Factor “s”)
Used as the requirement for the cargo ships and passenger ships.
Multiplying these two factors (p x s) will give the probability of surviving
that damage case.
99
Prepared by Dr. Sudhir Sindagi

Damaged Stability
Probabilistic Damage Stability
The value of S in all these will either be 0 or 1. This is because when we
have considered a damage, the ship will either survive (probability 1) or
not survive (probability 0).
So if this ship is three compartment ship, there is no need to consider
the probability of survival for four and more compartments because it
will be zero.
But there is still one thing to consider. At what drafts we need to
consider all these damages?
SOLAS requires that these should be considered at three drafts.

100
Prepared by Dr. Sudhir Sindagi

Damaged Stability
Probabilistic Damage Stability
Deepest subdivision draught (ds): Which corresponds to the Summer
Load Line draught of the ship.
Light service draught (dl): Service draught corresponding to the lightest
anticipated loading and associated tankage, including, however, such
ballast as may be necessary for stability and/or immersion.
Partial subdivision draught (dp): light service draught plus 60% of the
difference between the light service draught and the deepest
subdivision draught..

101
Prepared by Dr. Sudhir Sindagi

Damaged Stability
Probabilistic Damage Stability: Deepest subdivision draft (ds) will have

102
Prepared by Dr. Sudhir Sindagi

Damaged Stability
Probabilistic Damage Stability: Light service draught (dl) will have

103
Prepared by Dr. Sudhir Sindagi

Damaged Stability
Probabilistic Damage Stability: Partial subdivision draught (dp) will
have

104
Prepared by Dr. Sudhir Sindagi

Damaged Stability
Probabilistic Damage Stability: Attained Subdivision Index
Finally the bottom line. How would a ship comply with the damage
stability requirements?
As per SOLAS Chapter II-1, part B-1, Regulation 6, the ship complies
with damage stability when
Attained Subdivision Index > Required subdivision index
As per SOLAS, attained subdivision index is calculated by the formula.

105
Prepared by Dr. Sudhir Sindagi

Damaged Stability
Probabilistic Damage Stability: Required subdivision index
SOLAS chapter II-1, Reg 7 gives the formula to calculate the required
subdivision index for a ship. These formulas are different for different
type and size of the ship.
This would be the minimum required value of subdivision index.
If the actual value of subdivision index (Attained value) is less than the
required, the subdivisions need to be re-arranged or increased to have
attained subdivision index to be more than required subdivision index.

106
Prepared by Dr. Sudhir Sindagi

Damaged Stability

107
Prepared by Dr. Sudhir Sindagi

Damaged Stability
Floodable length and factor of subdivision
This is an old approach in which, the number of subdivisions required
is calculated by knowing the floodable length along the ship.
Floodable length is the length of the compartment which if flooded will
cause the ship to sink up to the margin line.

108
Prepared by Dr. Sudhir Sindagi

Damaged Stability
Floodable length and factor of subdivision
Margin line is the imaginary line drawn at least 76mm below the upper
surface of the bulkhead deck at side, such that, in any condition of
damage/flooding of the compartment, new waterline does not intersect
the margin line.
For a ship one needs to put subdivisions (bulkheads) to divide the ship
into compartments.
One such compartment is AB. The length of this compartment (Length
AB) need to such that if this compartment is flooded, the ship will sink
to a point where margin line is just submerged.

109
Prepared by Dr. Sudhir Sindagi

Damaged Stability
Floodable length and factor of subdivision
Now we want to place another bulkhead aft of midship. Again this
bulkhead needs to be at a location (C) such that if compartment AC is
flooded, the ship will sink to a point where margin line is just
submerged. And with this same approach, we can decide the location of
other bulkheads along the ship’s length.

110
Prepared by Dr. Sudhir Sindagi

Damaged Stability
Permissible length and factor of subdivision
The maximum permissible length of a compartment having its centre at
any point in the ship’s length is obtained from the floodable length by
multiplying the latter by an appropriate factor called factor of
subdivision.
The factor of subdivision shall be:
FOR NEW CLASS B, C AND D SHIPS AND EXISTING CLASS B RO-RO
PASSENGER SHIPS:
1.0 when the number of persons the ship is certified to carry is less than
400,
and 0.5 when the number of persons the ship is certified to carry is 400
or more. 111
Prepared by Dr. Sudhir Sindagi

Damaged Stability
Floodable length Curve
Floodable length curve represents the maximum floodable length of the
ship along the ship’s length. This curve is obtained by vertically plotting
the floodable length along the ship’s length.

112
Prepared by Dr. Sudhir Sindagi

Topics to study from the Internet


– Factor of Subdivision
– Floodable length Curve
– Permissible Length

113
Prepared by Dr. Sudhir Sindagi

Summary
Introduction- Trim
Trimming Moment, MCTC, IL, BML, GML
Trim due to
– Shifting of existing weight
– Addition or removal of weight
Damaged Stability
– Deterministic damage stability
• Added Weight Method
• Lost Buoyancy Method
– Permeability
– Probabilistic damage stability
– Floodable length, Margin Line, Permissible Length, Factor of 114
Subdivision, Floodable Length curve
Prepared by Dr. Sudhir Sindagi

Strength of Bulkhead and Ship

By
Dr. Sudhir Sindagi 1
Prepared by Dr. Sudhir Sindagi

Chapter Content
Introduction
Force and Center of Pressure for
– Regular shaped bulkhead
– Irregular shaped bulkhead
Strength of Ship
– Sagging
– Hogging
– Ship as a girder
– SFD & BMD for the ship
– Calculation of stresses in Main deck and Keel due to Bending Moment

2
Prepared by Dr. Sudhir Sindagi

Introduction
The problem of calculating the necessary strength of ships is made
difficult by the many and varied forces to which the ship structure is
subjected during its lifetime. These forces may be divided into two
groups, namely statical forces and dynamical forces.
The statical forces are due to:
– The weight of the structure which varies throughout the length of the ship.
– Buoyancy forces, which vary over each unit length of the ship and are constantly
varying in a seaway.
– Direct hydrostatic pressure.
– Concentrated local weights such as machinery, masts, derricks, winches, etc.

3
Prepared by Dr. Sudhir Sindagi

Introduction
The dynamical forces are due to:
– Pitching, heaving and rolling.
– Wind and waves.
These forces cause bending in several planes and local strains are set
up due to concentrated loads. The effects are aggravated by structural
discontinuities.
A stress is the mutual actual between the parts of a material to preserve
their relative positions when external loads are applied to the material.
Thus, whenever external loads are applied to a material stresses are
created within the material.

4
Prepared by Dr. Sudhir Sindagi

Introduction
When an external load is applied to a
material in such a way as to cause an
extension of the material it is called a
‘tensile’ load, whilst an external load
tending to cause compression of the
material is a ‘compressive’ load and
corresponding stresses are called as
‘tensile’ and Compressive stresses,
respectively.
A shearing stress is a stress within a
material which tends to break or shear the
material across tending to cause
deformation of a material by slippage along 5

a plane
Prepared by Dr. Sudhir Sindagi

Force and CP of Bulkhead


The water pressure at any depth is due to the weight of water above the
point in question and increases uniformly with depth below the surface.
From this, it can be seen that the pressure at any depth will vary with
the depth below the surface and the density of the water.
Force acting on the bulkhead is given by the Area under the pressure
diagram* width of the bulkhead.
Lets consider a rectangular shaped bulkhead with dimension B X D,
immersed completely into the water of density ρ in such a way that, WL
is coinciding with top of the bulkhead.
Pressure at any point on the bulkhead is proportional to the depth of
water, hence it will be 0 at the top and maximum at the bottom of the
bulkhead, as shown in the figure. 6
Prepared by Dr. Sudhir Sindagi

Force and CP of Bulkhead


Force = Area under the pressure diagram* width of
the bulkhead
𝟏
Force = 𝛒𝐠𝐃 ∗ 𝐃 ∗ 𝐁
𝟐
𝟏
Force = 𝛒𝐠𝐁𝐃𝟐
𝟐
This can be rearranged as follows
𝐃
Force = 𝛒𝐠 ∗ (𝐁 ∗ 𝐃) ∗
𝟐
Force = 𝛒𝐠 * Area of the Bulkhead * distance of the
centroid from WL
Force = 𝛒𝐠 * First moment of Area about WL
7
Prepared by Dr. Sudhir Sindagi

Force and CP of Bulkhead


Center of Pressure is the point on the bulkhead
where resultant force is assumed to be acting. Its
distance from the WL is mentioned as CP and is
calculated using
𝐅𝐢𝐫𝐬𝐭 𝐌𝐨𝐦𝐞𝐧𝐭 𝐨𝐟 𝐅𝐨𝐫𝐜𝐞 𝐚𝐛𝐨𝐮𝐭 𝐖𝐋
𝐂𝐏 =
𝐓𝐨𝐭𝐚𝐥 𝐅𝐨𝐫𝐜𝐞
𝛒𝐠∗ 𝐒𝐞𝐜𝐨𝐧𝐝 𝐌𝐨𝐦𝐞𝐧𝐭 𝐨𝐟 𝐚𝐫𝐞𝐚 𝐚𝐛𝐨𝐮𝐭 𝐖𝐋
𝐂𝐏 =
𝛒𝐠∗𝐅𝐢𝐫𝐬𝐭 𝐌𝐨𝐦𝐞𝐧𝐭 𝐨𝐟 𝐚𝐫𝐞𝐚 𝐚𝐛𝐨𝐮𝐭 𝐖𝐋
𝐒𝐞𝐜𝐨𝐧𝐝 𝐌𝐨𝐦𝐞𝐧𝐭 𝐨𝐟 𝐚𝐫𝐞𝐚 𝐚𝐛𝐨𝐮𝐭 𝐖𝐋
𝐂𝐏 =
𝐅𝐢𝐫𝐬𝐭 𝐌𝐨𝐦𝐞𝐧𝐭 𝐨𝐟 𝐚𝐫𝐞𝐚 𝐚𝐛𝐨𝐮𝐭 𝐖𝐋

8
Prepared by Dr. Sudhir Sindagi

Force and CP of Bulkhead


Consider a arbitrary shape of the bulkhead, immersed completely in the
water.
Consider a small elemental area of depth dh with width b at the depth h.
Force = 𝛒𝐠 * Area of the Bulkhead * distance of the centroid from WL
𝐝𝐡
Force on the elemental area = 𝛒𝐠 ∗ 𝐛𝐝𝐡 ∗ (𝐡 + )
𝟐
Since dh is very small, hence dh/2 can be neglected.
Force on small area= 𝛒𝐠 ∗ 𝐛𝐡 𝐝𝐡
𝐃
Total Force = 𝛒𝐠 ‫𝐡𝐝 𝐡𝐛 𝟎׬‬
𝐒𝐞𝐜𝐨𝐧𝐝 𝐌𝐨𝐦𝐞𝐧𝐭 𝐨𝐟 𝐚𝐫𝐞𝐚 𝐚𝐛𝐨𝐮𝐭 𝐖𝐋
𝐂𝐏 =
𝐅𝐢𝐫𝐬𝐭 𝐌𝐨𝐦𝐞𝐧𝐭 𝐨𝐟 𝐚𝐫𝐞𝐚 𝐚𝐛𝐨𝐮𝐭 𝐖𝐋
𝐃
‫𝐡𝐝 𝟐𝐡𝐛 𝟎׬‬ 9
𝐂𝐏 = 𝐃
‫𝐡𝐝 𝐡𝐛 𝟎׬‬
Prepared by Dr. Sudhir Sindagi

Force and CP of Bulkhead


Lets verify above expressions for the rectangular
shaped bulkhead
𝐃
Total Force = 𝛒𝐠 ‫𝐡𝐝 𝐡𝐛 𝟎׬‬
Here b=B is constant, hence
𝐃
Total Force = 𝛒𝐠𝐁 ‫𝐡 𝟎׬‬ 𝐝𝐡
𝟏
Force = 𝛒𝐠𝐁𝐃𝟐
𝟐
𝐃 𝐃 𝐃 𝐃𝟑
‫𝐡𝐝 𝟐𝐡𝐛 𝟎׬‬ 𝐁 ‫𝐡𝐝 𝟐𝐡 𝟎׬‬ ‫𝐡𝐝 𝟐𝐡 𝟎׬‬
𝐂𝐏 = 𝐃 = 𝐃 = 𝐃 = 𝟑
𝐃𝟐
‫𝐡𝐝 𝐡𝐛 𝟎׬‬ 𝐁 ‫𝐡𝐝 𝐡 𝟎׬‬ ‫𝐡𝐝 𝐡 𝟎׬‬
𝟐
𝟐𝐃
𝐂𝐏 = 10
𝟑
Prepared by Dr. Sudhir Sindagi

Force and CP of Bulkhead


Derive the expression for the force and CP for the
triangular shaped bulkhead shown in the figure
𝐃
Total Force = 𝛒𝐠 ‫𝐡𝐝 𝐡𝐛 𝟎׬‬
𝐛 𝐡 𝐡
Here b≠B is not constant, but = , b= B
𝐁 𝐃 𝐃
𝐁 𝐃 𝟐
Total Force = 𝛒𝐠 ‫𝐡𝐝 𝐡 𝟎׬‬
𝐃
𝟏
Force = 𝛒𝐠𝐁𝐃𝟐
𝟑
𝐃
‫𝐡𝐝 𝟐𝐡𝐛 𝟎׬‬
𝐂𝐏 = 𝐃
‫𝐡𝐝 𝐡𝐛 𝟎׬‬
𝟑𝐃
𝐂𝐏 = 11
𝟒
Prepared by Dr. Sudhir Sindagi

Force and CP of Bulkhead


Derive the expression for the force and CP for the
triangular shaped bulkhead shown in the figure
𝐃
Total Force = 𝛒𝐠 ‫𝐡𝐝 𝐡𝐛 𝟎׬‬
𝐛 𝐃−𝐡 𝐃−𝐡
Here b≠B is not constant, but = , b= B
𝐁 𝐃 𝐃
𝐃 𝐃−𝐡
Total Force = 𝛒𝐠 ‫ 𝟎׬‬B 𝐃 𝐡 𝐝𝐡
𝟏
Force = 𝛒𝐠𝐁𝐃𝟐
𝟔
𝐃
‫𝐡𝐝 𝟐𝐡𝐛 𝟎׬‬
𝐂𝐏 = 𝐃
‫𝐡𝐝 𝐡𝐛 𝟎׬‬
𝐃
𝐂𝐏 = 12
𝟐
Prepared by Dr. Sudhir Sindagi

Force and CP of Bulkhead


Calculate the force acting and its CP for a rectangular shaped bulkhead
of dimensions 12m X 10m (B X D), if SW is completely filled on one side
of the bulkhead and on the other side, the cargo of density 0.88 t/m3 is
half filled.
Force = 𝛒𝐠 * Area of the Bulkhead * distance of the centroid from WL
𝐃
FSW = 𝛒𝐠 ∗ 𝐁𝐃 ∗
𝟐
𝟏𝟎
FSW = 𝟏. 𝟎𝟐𝟓 ∗ 𝟗. 𝟖𝟏 ∗ 𝟏𝟐 ∗ 𝟏𝟎 ∗
𝟐
FSW = 6033.15 kN
𝟓
FC = 𝟎. 𝟖𝟖 ∗ 𝟗. 𝟖𝟏 ∗ 𝟏𝟐 ∗ 𝟓 ∗
𝟐
FC = 1294.92 kN 13
FResultant = FSW - FC = 4738.23 kN
Prepared by Dr. Sudhir Sindagi

Force and CP of Bulkhead


To calculate the CP value for the resultant force, one needs to estimate
either reaction at top or the bottom of the bulkhead.
Taking moment about Top
𝟐𝟎 𝟏𝟎
FSW * = FC * (5 + ) + FB * 10
𝟑 𝟑
After putting values of FSW & FC we get
FB =2943.63 kN
Again, Taking moment about Top
FR *CP= FB * 10
After putting values of FR we get
CP= 6.213m from top.
14
Prepared by Dr. Sudhir Sindagi

Strength of Ship
In a seaway, the ship is subjected to both static and dynamic forces
which cause it to bend in a longitudinal vertical plane.
For the purposes of structural design and for comparison, ship and the
ship problem is considered as a static one, so that it resolves into the
ship being poised statically on a wave and the resulting forces and
moments acting on the ship are calculated.
In order to determine the forces acting on a ship, it is treated as a
girder. It is necessary to determine the distribution of weight and the
buoyancy.
The total weight must equal to the total buoyancy and the fore and aft
position of the centre of gravity must be in the same athwartship
section as the centre of buoyancy.
15
Prepared by Dr. Sudhir Sindagi

Strength of Ship
For the purpose of investigating the longitudinal bending of a ship
certain assumptions are made. The calculations are, in general, carried
out for two standard conditions, Hogging and Sagging.
The assumptions are as follows:
– a) the ship is head on to the waves and is poised statically on a wave;
– b) the wave has a trochoidal profile of length equal to the length of the ship and a
height as described below.
– c) the wave crest is at amidships for the hogging condition;
– d) the wave crest is at the ends for the sagging condition.
A long accepted practice has been to take the height of the wave (h) as
1/20th of the length in strength calculations.
However, observation of sea waves has shown that longer waves tend
to be less steep than shorter waves. 16
Prepared by Dr. Sudhir Sindagi

Strength of Ship
Lloyd's Register suggested that a better approximation to the height
would be
h = 0.607 𝑳 meters
Later Murray of Lloyd's Register suggested that wave height should
vary as L0.3 instead of L0.5 and Muckle has derived from data that the
wave height can be written
h = 1·632*L0.3 where L is in metres.

17
Prepared by Dr. Sudhir Sindagi

Strength of Ship
Consider the case of a homogeneous log of rectangular section floating
freely at rest in still water.
The total weight of the log is balanced by the total force of buoyancy
and the weight (W) of any section of the log is balanced by the force of
buoyancy (B) provided by that section. There is therefore no bending
moment longitudinally which would cause stresses to be set up in the
log.

18
Prepared by Dr. Sudhir Sindagi

Strength of Ship
Now consider the case of a ship floating at rest in still water, on an even
keel, at the light draft.
Although the total weight of the ship is balanced by the total force of
buoyancy, neither is uniformly distributed throughout the ship’s length

19
Prepared by Dr. Sudhir Sindagi

Strength of Ship
Although the sections in the ship are not free to move in this way,
bending moments, and consequently longitudinal stresses, are created
by the variation in the longitudinal distribution of weight and buoyancy,
and these must be allowed for in the construction of the ship.
When a ship encounters waves at sea the stresses created differ greatly
from those created in still water. The maximum stresses are considered
to exist when the wavelength is equal to the ship’s length and either a
wave crest or trough is situated amidships

20
Prepared by Dr. Sudhir Sindagi

Strength of Ship
In this case, although once more the total weight of the ship is balanced
by the total buoyancy, there is an excess of buoyancy over the weight
amidships and an excess of weight over buoyancy at the bow and the
stern.
This situation creates a tendency for the ends of the ship to move
downwards and the section amidships to move upwards.
Under these conditions the ship is said to be subjected to a ‘hogging’
stress.

21
Prepared by Dr. Sudhir Sindagi

Strength of Ship
Consider the effect after the wave crest has moved onwards and the
ship is now supported by wave crests at the bow and the stern and a
trough amidships.
There is now an excess of buoyancy over weight at the ends and an
excess of weight over buoyancy amidships.
The situation creates a tendency for the bow and the stern to move
upwards and the section amidships to move downwards

22
Prepared by Dr. Sudhir Sindagi

Strength of Ship
Under these conditions a ship is said to be subjected to a sagging
stress.
Sagging and hogging of ship occurs even when the ship in calm water.
If the weight distribution is such that, more weights are distributed at
the midship and lesser weights are distributed at the ends, then ship
will Sag.
However, the ship will hog, when the weights are distributed more at the
ends rather than at the midship.

23
Prepared by Dr. Sudhir Sindagi

Strength of Ship

24
Prepared by Dr. Sudhir Sindagi

Strength of Ship
As shown in the figure, under the static condition, weight of structure,
machinery, cargo etc. acts downwards on the ship, while pressure from
the surrounding fluid acts upwards or sideways depending on the
position.
This situation becomes more complex when the ship is moving in
waves. Hence it becomes paramount to design ship’s structure so that
it will withstand such loading conditions.
In order to find scantlings(thicknesses of every structural member) of
the ship, it is very important to calculate Bending Moment acting on the
ship, which is estimated as per the procedure

25
Prepared by Dr. Sudhir Sindagi

Strength of Ship
1. Distribution of weights per unit length
known as the weight curve is plotted
against the length of the ship.
2. Distribution of buoyancy force per
unit length known as the Buoyancy
curve is plotted against the length of
the ship, which varies as per the
sectional area of the ship.
3. Hence each and every point on the
ship is subjected to a resultant load
which can be calculated as
4. Load / length = Buoyancy Force /
26
Length (b) – Weight Force / Length (w)
Prepared by Dr. Sudhir Sindagi

Strength of Ship
5. These values of load / length are
plotted against to get the Load Curve.
6. Due to the variation in the values of
load at each and every point, these
points are subjected to a shear force
𝐋
7. Shear Force (SFx) = ‫ 𝐛 𝟎׬‬− 𝐱 𝐝𝐱
8. Variation in the values of shear forces
are plotted against length to obtain
Shear Force Diagram (SFD)
9. Similar to the values of SFx, Bending
moment values are calculated as
𝐋 27
10. Bending Moment (BMx)=‫𝐱𝐝 𝐱𝐅𝐒 𝟎׬‬
Prepared by Dr. Sudhir Sindagi

Strength of Ship
11. Variation in the values of BMx are
plotted to generate Bending Moment
Diagram (BMD).
12. Maximum value of Bending moment is
then used to calculate the minimum
values of section modulus of the ship
required to withstand it using
𝐌𝐦𝐚𝐱 𝛔
13. 𝐈 =
𝐲
14. Minimum section Modulus
𝐈 𝐌𝐦𝐚𝐱
15. (Z) min =( )min=
𝐲 𝛔
28
Prepared by Dr. Sudhir Sindagi

Strength of Ship

29
Prepared by Dr. Sudhir Sindagi

Problems
A rectangular barge of 250m length 35m beam and 20m depth floats in
SW at a draft of 2m when it is empty. The lightship weight may be
assumed to be uniformly distributed over the barge length. It has five
holds each 50m long. The barge floats in SW loaded with cargo as
shown below. Cargo weights within each hold are to be assumed as
uniformly distributed over the length of the hold. Calculate and plot
diagrams of distributions of weight, buoyancy, load, shear and bending.
Determine the values of the curves at each bulkhead and at their
maximum points.
Hold 1 Hold 2 Hold 3 Hold 4 Hold 5
14000t 17000t 28000t 17000t 14000t
50m length 50m length 50m length 50m length 50m length
30
Prepared by Dr. Sudhir Sindagi

Strength of Ship
Given Data:
L=250m, B=35m, T=2m floats in SW.
Weight in lightship condition = Volume * Density
Lightweight = 250*35*2*1.025 =17937.5 tonnes
Weight of Cargo = 14000+17000+28000+17000+14000
Weight of Cargo = 90000 tonnes
Displacement of ship = 17937.5 + 90000
Displacement of ship = 107937.5
Since it is given that, the lightship weight may be assumed to be
uniformly distributed over the barge length
𝐋𝐢𝐠𝐡𝐭 𝐰𝐞𝐢𝐠𝐡𝐭 17937.5
= = 71.75 t/m 31
𝐋𝐞𝐧𝐠𝐭𝐡 𝟐𝟓𝟎
Prepared by Dr. Sudhir Sindagi

Strength of Ship
Now, cargo weight may be assumed to be uniformly distributed over the
length of the cargo hold.
𝐂𝐚𝐫𝐠𝐨 𝐰𝐞𝐢𝐠𝐡𝐭 14000
for the hold 1 = = 280 t/m
𝐋𝐞𝐧𝐠𝐭𝐡 𝟓𝟎
𝐂𝐚𝐫𝐠𝐨 𝐰𝐞𝐢𝐠𝐡𝐭 17000
for the hold 2 = = 340 t/m
𝐋𝐞𝐧𝐠𝐭𝐡 𝟓𝟎
𝐂𝐚𝐫𝐠𝐨 𝐰𝐞𝐢𝐠𝐡𝐭 28000
for the hold 3 = = 510 t/m
𝐋𝐞𝐧𝐠𝐭𝐡 𝟓𝟎
𝐂𝐚𝐫𝐠𝐨 𝐰𝐞𝐢𝐠𝐡𝐭 17000
for the hold 4 = = 340 t/m
𝐋𝐞𝐧𝐠𝐭𝐡 𝟓𝟎
𝐂𝐚𝐫𝐠𝐨 𝐰𝐞𝐢𝐠𝐡𝐭 14000
for the hold 5 = = 280 t/m
𝐋𝐞𝐧𝐠𝐭𝐡 𝟓𝟎

32
Prepared by Dr. Sudhir Sindagi

Strength of Ship
Now, since it is a rectangular barge with rectangular sectional area
remaining constant throughout the length, hence buoyancy force may
be assumed to be uniformly distributed over the length of the ship.
𝐁𝐮𝐨𝐲𝐚𝐧𝐜𝐲 𝐅𝐨𝐫𝐜𝐞 Δ 107937.5
= b= = = 431.75 t/m
𝐋𝐞𝐧𝐠𝐭𝐡 𝐋 𝟐𝟓𝟎
𝐓𝐨𝐭𝐚𝐥 𝐰𝐞𝐢𝐠𝐡𝐭 𝐋𝐢𝐠𝐡𝐭 𝐰𝐞𝐢𝐠𝐡𝐭 𝐂𝐚𝐫𝐠𝐨 𝐰𝐞𝐢𝐠𝐡𝐭
at any point = + for the hold
𝐋𝐞𝐧𝐠𝐭𝐡 𝐋𝐞𝐧𝐠𝐭𝐡 𝐋𝐞𝐧𝐠𝐭𝐡
𝐓𝐨𝐭𝐚𝐥 𝐰𝐞𝐢𝐠𝐡𝐭 𝐋𝐢𝐠𝐡𝐭 𝐰𝐞𝐢𝐠𝐡𝐭 𝐂𝐚𝐫𝐠𝐨 𝐰𝐞𝐢𝐠𝐡𝐭
for the Hold 1 = + for the hold 1
𝐋𝐞𝐧𝐠𝐭𝐡 𝐋𝐞𝐧𝐠𝐭𝐡 𝐋𝐞𝐧𝐠𝐭𝐡
𝐓𝐨𝐭𝐚𝐥 𝐰𝐞𝐢𝐠𝐡𝐭
for the Hold 1 (w1) = 71.75 t/m + 280 t/m = 351. 75 t/m
𝐋𝐞𝐧𝐠𝐭𝐡
𝐓𝐨𝐭𝐚𝐥 𝐰𝐞𝐢𝐠𝐡𝐭
for the Hold 2 (w2) = 71.75 t/m + 340 t/m = 411. 75 t/m
𝐋𝐞𝐧𝐠𝐭𝐡
33
Prepared by Dr. Sudhir Sindagi

Strength of Ship
𝐓𝐨𝐭𝐚𝐥 𝐰𝐞𝐢𝐠𝐡𝐭
for the Hold 3 (w3) = 71.75 t/m + 510 t/m = 631. 75 t/m
𝐋𝐞𝐧𝐠𝐭𝐡
𝐓𝐨𝐭𝐚𝐥 𝐰𝐞𝐢𝐠𝐡𝐭
for the Hold 4 (w4) = 71.75 t/m + 340 t/m = 411. 75 t/m
𝐋𝐞𝐧𝐠𝐭𝐡
𝐓𝐨𝐭𝐚𝐥 𝐰𝐞𝐢𝐠𝐡𝐭
for the Hold 5 (w5) = 71.75 t/m + 280 t/m = 351. 75 t/m
𝐋𝐞𝐧𝐠𝐭𝐡
Hence each and every point on the ship is subjected to a resultant load
which can be calculated as
Load / length = Buoyancy Force / Length (b) – Weight Force / Length (w)
𝐋𝐨𝐚𝐝 Buoyancy Force Weight Force
for the Hold 1 = - for the hold 1
𝐋𝐞𝐧𝐠𝐭𝐡 𝐋𝐞𝐧𝐠𝐭𝐡 𝐋𝐞𝐧𝐠𝐭𝐡
𝐋𝐨𝐚𝐝
for the Hold 1 = 431.75 – 351.75 = 80 t/m
𝐋𝐞𝐧𝐠𝐭𝐡 34
Prepared by Dr. Sudhir Sindagi

Strength of Ship
𝐋𝐨𝐚𝐝
for the Hold 2 = 431.75 – 411.75 = 20 t/m
𝐋𝐞𝐧𝐠𝐭𝐡
𝐋𝐨𝐚𝐝
for the Hold 3 = 431.75 – 631.75 = -200 t/m
𝐋𝐞𝐧𝐠𝐭𝐡
𝐋𝐨𝐚𝐝
for the Hold 4 = 431.75 – 411.75 = 20 t/m
𝐋𝐞𝐧𝐠𝐭𝐡
𝐋𝐨𝐚𝐝
for the Hold 5 = 431.75 – 351.75 = 80 t/m
𝐋𝐞𝐧𝐠𝐭𝐡
With the available data we can get the weight curve, buoyancy curve
and then the load curve.

35
Prepared by Dr. Sudhir Sindagi

Strength of Ship

36
Prepared by Dr. Sudhir Sindagi

Strength of Ship
Shear Force at any point of consideration is the area under the load
curve till that point.
SFA= 0
SFB= 80 * 50 = 4000 t
SFC= 4000+ 20*50 = 5000 t
SFD= 5000 – 200*50 = -5000 t
SFE= -5000 + 20*50 = -4000 t
SFF= -4000 + 50*40 = 0
SFO= 0

37
Prepared by Dr. Sudhir Sindagi

Strength of Ship
Bending Moment at any point of consideration is the area under the SFD
till that point.
BMA= 0
BMB= ½ * 50 * 4000 = 100000 tm
BMC= 100000 + ½ (5000+4000)* 50 = 325000 tm
BMO= 325000 + ½ *25 *5000 = 387500 tm
BMD= 387500 – ½ *25*5000 = 325000 tm
BME= 325000 - ½ (5000+4000)* 50 = 100000 t-m
BMF= 100000 - ½ * 50 * 4000 = 0

38
Prepared by Dr. Sudhir Sindagi

Strength of Ship

39
Prepared by Dr. Sudhir Sindagi

Strength of Ship
Summary of Curves ( Relationship between strength Curves):
The area under the weight curve and that under the buoyancy curve are
equal.
The centroids of the areas of weight and buoyancy are in the same
athwartship section.
For the load curve the areas above and below the base line are equal.
The maximum values of the shearing force occur where the load curve
crosses the base line.
The maximum bending moment occurs where the shearing force curve
crosses the base line.
The shearing force and bending moment curves must close. The value
at the ends is zero. 40
Prepared by Dr. Sudhir Sindagi

Problems
A box shaped barge of uniform construction is 32m long and displaces
352tonnes when empty. It is divided by transverse bulkheads into four
equal compartments. Cargo is loaded into each compartments as below
No 1 Hold -192 tonnes No 2 Hold- 224tonnes
No 3 hold- 272 tonnes No 4 hold- 176 tonnes
Given Data:
L=32m, Lightweight = 352 tonnes.
Weight of Cargo = 192+224+272+176
Weight of Cargo = 864 tonnes
Displacement of ship = 352 + 864
Displacement of ship = 1216 t
41
Prepared by Dr. Sudhir Sindagi

Problems
Since it is given that, the lightship weight may be assumed to be
uniformly distributed over the barge length
𝐋𝐢𝐠𝐡𝐭 𝐰𝐞𝐢𝐠𝐡𝐭 352
= = 11 t/m
𝐋𝐞𝐧𝐠𝐭𝐡 𝟑𝟐
Now, cargo weight may be assumed to be uniformly distributed over the
length of the cargo hold.
𝐂𝐚𝐫𝐠𝐨 𝐰𝐞𝐢𝐠𝐡𝐭 192
for the hold 1 = = 24 t/m
𝐋𝐞𝐧𝐠𝐭𝐡 𝟖
𝐂𝐚𝐫𝐠𝐨 𝐰𝐞𝐢𝐠𝐡𝐭 224
for the hold 2 = = 28 t/m
𝐋𝐞𝐧𝐠𝐭𝐡 𝟖
𝐂𝐚𝐫𝐠𝐨 𝐰𝐞𝐢𝐠𝐡𝐭 272
for the hold 3 = = 34 t/m
𝐋𝐞𝐧𝐠𝐭𝐡 𝟖
𝐂𝐚𝐫𝐠𝐨 𝐰𝐞𝐢𝐠𝐡𝐭 176 42
for the hold 4 = = 22 t/m
𝐋𝐞𝐧𝐠𝐭𝐡 𝟖
Prepared by Dr. Sudhir Sindagi

Strength of Ship
Now, since it is a rectangular barge with rectangular sectional area
remaining constant throughout the length, hence buoyancy force may
be assumed to be uniformly distributed over the length of the ship.
𝐁𝐮𝐨𝐲𝐚𝐧𝐜𝐲 𝐅𝐨𝐫𝐜𝐞 Δ 1216
= b= = = 38 t/m
𝐋𝐞𝐧𝐠𝐭𝐡 𝐋 𝟑𝟐
𝐓𝐨𝐭𝐚𝐥 𝐰𝐞𝐢𝐠𝐡𝐭 𝐋𝐢𝐠𝐡𝐭 𝐰𝐞𝐢𝐠𝐡𝐭 𝐂𝐚𝐫𝐠𝐨 𝐰𝐞𝐢𝐠𝐡𝐭
at any point = + for the hold
𝐋𝐞𝐧𝐠𝐭𝐡 𝐋𝐞𝐧𝐠𝐭𝐡 𝐋𝐞𝐧𝐠𝐭𝐡
𝐓𝐨𝐭𝐚𝐥 𝐰𝐞𝐢𝐠𝐡𝐭 𝐋𝐢𝐠𝐡𝐭 𝐰𝐞𝐢𝐠𝐡𝐭 𝐂𝐚𝐫𝐠𝐨 𝐰𝐞𝐢𝐠𝐡𝐭
for the Hold 1 = + for the hold 1
𝐋𝐞𝐧𝐠𝐭𝐡 𝐋𝐞𝐧𝐠𝐭𝐡 𝐋𝐞𝐧𝐠𝐭𝐡
𝐓𝐨𝐭𝐚𝐥 𝐰𝐞𝐢𝐠𝐡𝐭
for the Hold 1 (w1) = 24 t/m + 11 t/m = 35 t/m
𝐋𝐞𝐧𝐠𝐭𝐡
𝐓𝐨𝐭𝐚𝐥 𝐰𝐞𝐢𝐠𝐡𝐭
for the Hold 2 (w2) = 39 t/m
𝐋𝐞𝐧𝐠𝐭𝐡
43
Prepared by Dr. Sudhir Sindagi

Strength of Ship
𝐓𝐨𝐭𝐚𝐥 𝐰𝐞𝐢𝐠𝐡𝐭
for the Hold 3 (w3) = 45 t/m
𝐋𝐞𝐧𝐠𝐭𝐡
𝐓𝐨𝐭𝐚𝐥 𝐰𝐞𝐢𝐠𝐡𝐭
for the Hold 4 (w4) = 33 t/m
𝐋𝐞𝐧𝐠𝐭𝐡
Hence each and every point on the ship is subjected to a resultant load
which can be calculated as
Load / length = Buoyancy Force / Length (b) – Weight Force / Length (w)
𝐋𝐨𝐚𝐝 Buoyancy Force Weight Force
for the Hold 1 = - for the hold 1
𝐋𝐞𝐧𝐠𝐭𝐡 𝐋𝐞𝐧𝐠𝐭𝐡 𝐋𝐞𝐧𝐠𝐭𝐡
𝐋𝐨𝐚𝐝
for the Hold 1 = 38 - 35= 3 t/m
𝐋𝐞𝐧𝐠𝐭𝐡

44
Prepared by Dr. Sudhir Sindagi

Strength of Ship
𝐋𝐨𝐚𝐝
for the Hold 2 = 38 – 39 = -1 t/m
𝐋𝐞𝐧𝐠𝐭𝐡
𝐋𝐨𝐚𝐝
for the Hold 3 = 38 - 45= -7 t/m
𝐋𝐞𝐧𝐠𝐭𝐡
𝐋𝐨𝐚𝐝
for the Hold 4 = 38 - 33= 5 t/m
𝐋𝐞𝐧𝐠𝐭𝐡
With the available data we can get the weight curve, buoyancy curve
and then the load curve.

45
Prepared by Dr. Sudhir Sindagi

Strength of Ship

46
Prepared by Dr. Sudhir Sindagi

Strength of Ship
Shear Force at any point of consideration is the area under the load
curve till that point.
SFA= 0
SFB= 3 * 8 = 24 t
SFC= 24 – 1*8= 16 t
SFD= 16 – 7*8= -40 t
SFE= -40 + 5*8= 0 t
To find the location of 0
𝒙 𝟖−𝒙
=
𝟏𝟔 𝟒𝟎
x = 2.285m from C
47
Prepared by Dr. Sudhir Sindagi

Strength of Ship
Bending Moment at any point of consideration is the area under the SFD
till that point.
BMA= 0
BMB= ½ * 8 * 24 = 96 tm
BMC= 96 + ½ (24+16)* 8 = 256 tm
BMO= 256 + ½ *2.285 *16 = 274.28 tm
BMD= 274.28 – ½ *(8-2.285)*40 = 160 tm
BME= 160 - ½*40*8= 0 t-m

48
Prepared by Dr. Sudhir Sindagi

Problems

49
Prepared by Dr. Sudhir Sindagi

Problems
A deck beam is in the form of an H-girder as shown in the
accompanying Figure. If the bending moment at the middle of its length
is 15 tonnes metres, find the maximum stress in the steel.

To find Maximum stress, we will use the equation


𝐌 𝛔
= 50
𝐈 𝐲
Prepared by Dr. Sudhir Sindagi

Problems
Since the section is has uniform thicknesses at the top and the bottom,
hence, the neutral axis lies at the mid point of the section.
Moment of Inertia of the area about the neutral axis will be
𝟎.𝟑∗𝟎.𝟑𝟑 𝟎.𝟏𝟒∗𝟎.𝟐𝟓𝟑
𝐈𝐍𝐀 = − *2
𝟏𝟐 𝟏𝟐
𝐈𝐍𝐀 =3.1042 * 10-4 m4
𝐌 𝛔
=
𝐈 𝐲
𝟏𝟓 ∗𝟏𝟎𝟎𝟎∗𝟗.𝟖𝟏 𝛔
=
3.1042 ∗ 10−4 𝟎.𝟏𝟓
𝛔 = 71.108 MN/m2

51
Prepared by Dr. Sudhir Sindagi

Problems
A Midship Section Drawing is shown here. (See Fig.1).
(a) Calculate the Moment of Inertia of the Ship Section.
(b) Calculate the Section Modulus at Deck & Section Modulus at Keel

𝐈𝐍𝐀
Section Modulus = Z=
𝒚
In this case, need to find
location of NA
MI about NA
Need to use tabular method for NA

52
Prepared by Dr. Sudhir Sindagi

Problems
(1) (2) (3) (4) =(2) * (3) (5) =(3) * (4) (6)
Area of the Lever 1st Moment 2nd Moment INA about their
Item
section from Keel about Keel about Keel neutral axis

Deck

02 Side
shells
Bottom
Shell
Total ΣA=00 m2 ΣM1=00 m3 ΣM2=00 m4 ΣINA =00 m4

53
Prepared by Dr. Sudhir Sindagi

Problems
(1) (2) (3) (4) =(2) * (3) (5) =(3) * (4) (6)
Area of the Lever 1st Moment 2nd Moment INA about their
Item
section from Keel about Keel about Keel neutral axis
20∗0.0253
Deck 20*0.025 = 0.5 13 6.5 84.5 ≈0
12

02 Side 2∗0.014∗133
13*0.014*2 = 0.364 6.5 2.366 15.379 = 5.1263
shells 12

Bottom 20∗0.023
20*0.02 =0.4 0.01 0.004 0.00004 ≈0
Shell 12

ΣM1=8.87 ΣM2=99.879
Total ΣA=1.264 m2 ΣINA =5.1263 m4
m3 m4

54
Prepared by Dr. Sudhir Sindagi

Problems
Distance of NA from the keel
𝐅𝐢𝐫𝐬𝐭 𝐌𝐨𝐦𝐞𝐧𝐭 𝐨𝐟 𝐭𝐡𝐞 𝐚𝐫𝐞𝐚 𝐚𝐛𝐨𝐮𝐭 𝐤𝐞𝐞𝐥 ΣM1
Distance of NA from the keel = ==
𝐀𝐫𝐞𝐚 ΣA
Distance of NA from the keel =7.017 m from keel
We need to estimate the MI about NA.
Hence, using parallel axis theorem
MI about Keel= Σ(INA)self +Σ(A*d2)
MI about Keel= 5.1263 + 99.879
MI about Keel= 105.005 m4
MI about Keel= INA+A*d2
105.005 = INA+ 1.264*7.0172
55
INA=42.767m4
Prepared by Dr. Sudhir Sindagi

Problems
Section Modulus (Z) is given by
𝐈𝐍𝐀
Z=
𝐲
𝐈𝐍𝐀
(Z)Main Deck =
𝐲𝐌𝐃
42.767
(Z)Main Deck =
(𝟏𝟑−𝟕.𝟎𝟏𝟕)
(Z) Main Deck =7.148 m3
𝐈𝐍𝐀
(Z) Keel =
𝐲Keel
42.767
(Z) Keel =
𝟕.𝟎𝟏𝟕
(Z) Keel = 6.094 m3 56
Prepared by Dr. Sudhir Sindagi

Safe Stress
Shear Stress at any point is given by
𝐅𝐀𝐘ഥ
𝐪=
𝐛𝐈
F= The shearing Force at the section under consideration
𝐀𝐘ഥ= Moment of area about the Neutral axis above or below the surface under
consideration.
ഥ=Distance of the centroid of the area under consideration from NA
𝐘
I= Total Moment of Inertia about the NA
b= Total thickness of material resisting shear
The maximum allowable stress in terms of the length of a ship using
P=77+0.25*L
𝟏
𝐏 = 𝟐𝟑 ∗ 𝐋𝟑
𝐌𝐍 57
𝐖𝐡𝐞𝐫𝐞 𝐏 𝐢𝐬 𝐢𝐧 𝐚𝐧𝐝 𝐋 𝐢𝐬 𝐢𝐧 𝐌𝐞𝐭𝐞𝐫𝐬.
𝐦𝟐
Prepared by Dr. Sudhir Sindagi

Problems
In a ship the maximum shearing force is 44.76MN. 𝐀𝐘 ത about the NA is
151000mcm2. I about NA is 2758000m2cm2 and the thickness of the shell
plating at the neutral axis is 2.14cm. Determine the shear stress induced
at the neutral axis.
Given Data:
F= 44.7 MN, 𝐀𝐘 ത = 151000mcm2, I about NA is 2758000m2cm2, b= 2.14cm
q=?
𝐅𝐀𝐘ത
𝐪=
𝐛𝐈
𝐌𝐍∗m∗cm2
𝐪=
𝐜𝐦 ∗m2cm2
𝐌𝐍
𝐪= *100
m2 58
Prepared by Dr. Sudhir Sindagi

Problems
𝐅𝐀𝐘ത
𝐪=
𝐛𝐈
𝟒𝟒.𝟕𝟔∗𝟏𝟓𝟏𝟎𝟎𝟎 𝐌𝐍
𝐪= *100
𝟐.𝟏𝟒∗𝟐𝟕𝟓𝟖𝟎𝟎𝟎 m2
𝐌𝐍
𝐪 = 𝟏. 𝟏𝟒
m2

59
Prepared by Dr. Sudhir Sindagi

Problems
The midship section of a ship of breadth 16.5m and depth 11m can be
assumed as shown in the figure. All the material has a thickness of
1.25cm. Determine the MI of the section about NA. Moments are taken
about the base. What are the section modulus at the upper deck and at
the keel? Determine bending stress (stating whether tensile or
compressive) for a sagging BM of 44500 t-m. If the ships length is 200m,
is it safe in bending stress?

60
Prepared by Dr. Sudhir Sindagi

Problems
(1) (2) (3) (4) =(2) * (3) (5) =(3) * (4) (6)
Area of the Lever 1st Moment 2nd Moment INA about their
Item
section from Keel about Keel about Keel neutral axis

UD

2nd
Deck
Tank
Top
Keel
2 Side
Shells
Central
Girder 61

Total ΣA=00 m2 ΣM =00 m3 ΣM =00 m4 ΣI =00 m4


Prepared by Dr. Sudhir Sindagi

(1) (2) (3) Problems


(4) =(2) * (3) (5) =(3) * (4) (6)
Area of the Lever 1st Moment 2nd Moment INA about their
Item
section from Keel about Keel about Keel neutral axis
16.5*0.0125 16∗0.01253
UD 11 2.2687 24.9557 ≈0
=0.20625 12

2nd 16∗0.01253
0.20625 8.4 1.7329 14.5521 ≈0
Deck 12

Tank 16∗0.01253
0.20625 1.2 0.2475 0.297 ≈0
Top 12
16∗0.01253
Keel 0.20625 0 0 0 ≈0
12
2 Side 2∗0.0125∗113
2*11*0.0125=0.275 5.5 1.5125 8.3187 = 2.7729
Shells 12

Central 0.0125∗1.23
1.2*0.0125=0.015 0.6 0.009 0.0054 = 0.0018
Girder 12

ΣM1=5.7702 ΣM2=48.0134 62
Total ΣA=1.115 m2 ΣINA =2.7747 m4
m3 m4
Prepared by Dr. Sudhir Sindagi

Problems
Distance of NA from the keel
𝐅𝐢𝐫𝐬𝐭 𝐌𝐨𝐦𝐞𝐧𝐭 𝐨𝐟 𝐭𝐡𝐞 𝐚𝐫𝐞𝐚 𝐚𝐛𝐨𝐮𝐭 𝐤𝐞𝐞𝐥 ΣM1
Distance of NA from the keel = ==
𝐀𝐫𝐞𝐚 ΣA
Distance of NA from the keel=5.1751 m from keel
We need to estimate the MI about NA.
Hence, using parallel axis theorem
MI about Keel= Σ(INA)self +Σ(A*d2)
MI about Keel= 50.9051 m4
MI about Keel= INA+A*d2
50.9051 = INA+ 1.115*5.17512
INA=21.053m4
63
Prepared by Dr. Sudhir Sindagi

Problems
Section Modulus (Z) is given by
𝐈𝐍𝐀
Z=
𝐲
𝐈𝐍𝐀
(Z)Main Deck =
𝐲𝐌𝐃
𝟐𝟏.𝟎𝟓𝟑
(Z)Main Deck =
(𝟏𝟏−𝟓.𝟏𝟕𝟓𝟏)
(Z) Main Deck =3.6125 m3
𝐈𝐍𝐀
(Z) Keel =
𝐲Keel
𝟐𝟏.𝟎𝟓𝟑
(Z) Keel =
𝟓.𝟏𝟕𝟓𝟏
(Z) Keel = 4.066m3
64
Prepared by Dr. Sudhir Sindagi

Problems
Stresses at Main Deck and the keel are calculated as follows
𝐌
σ=
𝐙
𝐌
(σ)Main Deck =
𝐙𝐌𝐃
𝟒𝟒𝟓𝟎𝟎∗𝟏𝟎𝟎𝟎∗𝟗.𝟖𝟏
(σ)Main Deck =
𝟑.𝟔𝟏𝟐𝟓
(σ)Main Deck = =120.84 MN/m2 (Compressive)
𝐌
(σ) Keel =
𝐙Keel
𝟒𝟒𝟓𝟎𝟎∗𝟏𝟎𝟎𝟎∗𝟗.𝟖𝟏
(σ) Keel =
𝟒.𝟎𝟔𝟔
(σ) Keel = 107.36 MN/m2 (Tensile)
65
Prepared by Dr. Sudhir Sindagi

Problems
We know that, Safe stress is calculated using following
P=77+0.25*L
P=127 MN/m2
and
𝟏
𝐏 = 𝟐𝟑 ∗ 𝐋𝟑
𝐏 = 𝟏𝟑𝟒. 𝟓 MN/m2
Since induced stresses are lesser than the allowable stresses hence,
the structure of the ship is safe.

66
Prepared by Dr. Sudhir Sindagi

Summary
Introduction
Force and Center of Pressure for
– Regular shaped bulkhead
– Irregular shaped bulkhead
Strength of Ship
– Sagging
– Hogging
– Ship as a girder
– SFD & BMD for the ship
– Calculation of stresses in Main deck and Keel due to Bending Moment

67

You might also like